vi clinical case presentations - wiley · clinical case presentations ... case 17: dog with vitamin...

135
Clinical Case Presentations VI

Upload: truongphuc

Post on 27-May-2018

226 views

Category:

Documents


0 download

TRANSCRIPT

Page 1: VI Clinical Case Presentations - Wiley · Clinical Case Presentations ... Case 17: Dog with vitamin K antagonist (rodenticide) toxicity, 643 ... PT Prothrombin time

Clinical Case Presentations VI

Page 2: VI Clinical Case Presentations - Wiley · Clinical Case Presentations ... Case 17: Dog with vitamin K antagonist (rodenticide) toxicity, 643 ... PT Prothrombin time
Page 3: VI Clinical Case Presentations - Wiley · Clinical Case Presentations ... Case 17: Dog with vitamin K antagonist (rodenticide) toxicity, 643 ... PT Prothrombin time

617

CA

SES

Clinical Case Presentations

Veterinary Hematology and Clinical Chemistry, Second Edition. Edited by Mary Anna Thrall, Glade Weiser, Robin W. Allison,

and Terry W. Campbell.

© 2012 John Wiley & Sons, Inc. Published 2012 by John Wiley & Sons, Inc.

Hematology Case 1: Cat with hemolytic anemia due to Mycoplasma

haemofelis , 620 Case 2: Dog with immune - mediated hemolytic anemia,

622 Case 3: Cat with myeloproliferative disorder M6 (E), 624 Case 4: Cat with iron defi ciency anemia, 625 Case 5: Dog with chronic infl ammatory disease and iron

defi ciency, 626 Case 6: Dog with chronic external blood loss, iron

defi ciency anemia, 627 Case 7: Dog with iron defi ciency anemia, 628 Case 8: Dog with hemangiosarcoma, acute blood loss,

631 Case 9: Dog with chronic ehrlichiosis and Mycoplasma

haemocanis , 632 Case 10: Dog with anemia of infl ammatory disease and

DIC, 633 Case 11: Dog with acute infl ammatory disease and sepsis,

634 Case 12: Dog with immune - mediated thrombocytopenia,

blood loss, 636 Case 13: Dog with lymphocytic leukemia, 637

Introduction This section presents a number of case studies taken from animal medical records. Each case is presented with its rel-evant clinicopathologic data. The cases are organized more or less by the primary disease or organ system involved in disease, with the understanding that many of them have multiple system abnormalities. For example, the fi rst 14 cases focus on common primary hematologic disease. However, abnormal hematologic processes are dispersed throughout the remaining cases. The following is a directory of cases classifi ed by the primary system abnormality or disease problem.

Clinical Case Presentations: Contents

Case 14: Dog with hyperproteinemia and monoclonal gammopathy due to myeloma, 638

Coagulation Case 15: Disseminated intravascular coagulation, 640 Case 16: Dog with hemophilia A, 642 Case 17: Dog with vitamin K antagonist (rodenticide)

toxicity, 643 Case 18: Disseminated intravascular coagulation associated

with pancreatitis, 644

Renal Disease Case 19: Dog with acute renal failure associated with

heatstroke, 646 Case 20: Dog with end - stage renal disease, 648 Case 21: Dog with glomerular disease, amyloidosis, 650 Case 22: Cat with acute renal disease associated with

thromboembolic disease, 652 Case 23: Dog with chronic renal failure associated with

infi ltrative neoplastic disease, 655 Case 24: Horse with hypercalcemic chronic renal failure,

and salmonellosis, 657 Case 25: Cat with kaliopenic polymyopathy/nephropathy

syndrome, 659 Case 26: Dog with congenital Fanconi ’ s syndrome, 661 Case 27: Dog with hypercalcemic nephropathy, 664 Case 28: Dog with polydipsia associated with

hypercalcemia, 666 Case 29: Cat with renal disease and severe acidemia, 667 Case 30: Cat with acute renal failure due to ethylene

glycol toxicity, 668 Case 31: Dog with atypical ethylene glycol toxicity, 670 Case 32: Foal with uroabdomen, 671

Fluid and Electrolyte and Acid - Base Disturbances Disorders of electrolytes and acid base disturbances are not primary disease problems. Therefore, most electrolyte and

Page 4: VI Clinical Case Presentations - Wiley · Clinical Case Presentations ... Case 17: Dog with vitamin K antagonist (rodenticide) toxicity, 643 ... PT Prothrombin time

SECTION VI Clinical Case Presentations

618

CA

SES

acid - base disturbances are best considered with their primary disease process(es). Therefore, for more broad exposure to this category, also see: Electrolyte abnormalities Cases: # ’ s 11, 16, 24, 25, 30, 32, 34, 35, 41, 45, 47, 53, 54,

56, 57, 58, 63, 64, 65, 66 Acid base disturbances Cases: # ’ s 19, 29, 31, 33, 40, 54, 56, 57, 58, 65

Case 33: Cat with compensated metabolic acidosis associated with diabetes, 673

Case 34: Cat with sodium chloride depletion due to meningomyelocele, 674

Case 35: Horse with colic, 676

Hepatic Disease Case 36: Dog with cholangiohepatitis and cholestasis, 677 Case 37: Dog with hepatitis and cholestasis associated with

steroid hepatopathy, 679 Case 38: Dog with chronic liver disease due to metastatic

neoplasia, 680 Case 39: Dog with liver failure associated with chronic

diaphragmatic hernia, 682 Case 40: Dog with liver failure associated with severe

hepatic infi ltrate with lymphoma, 684 Case 41: Dog with bile peritonitis, 686 Case 42: Dog with end - stage liver disease, cirrhosis, 688 Case 43: Dog with portosystemic shunt, 689 Case 44: Dog with hepatocellular injury associated with

trauma, 692 Case 45: Dog with cirrhosis and cholestasis, 694 Case 46: Dog with portosystemic shunt, 696 Case 47: Dog with hepatic lipidosis and cellular injury

associated with diabetes, 697 Case 48: Cat with hepatic lipidosis syndrome, 698 Case 49: Alpaca with hepatic lipidosis and serum lipid

abnormalities, 699 Case 50: Dog with end - stage liver disease and

hepatocutaneous syndrome, 701

Pancreatic & Gastrointestinal Disease and Glucose Metabolism Case 51: Dog with hyperlipidemia syndrome, 703 Case 52: Dog with hyperlipidemia and pancreatitis, 705 Case 53: Cat with diabetic non - ketotic hyperosmolar

syndrome, 707 Case 54: Cat with diabetic ketoacidosis, 709 Case 55: Dog with hypoglycemia associated with insulin

secreting neoplasia, 711 Case 56: Calf with secretory diarrhea and hypotonic

dehydration, 712 Case 57: Bull with upper gastrointestinal obstruction, 714 Case 58: Calf with non - secretory diarrhea and hypertonic

dehydration, 716

Case 59: Dog with protein - losing enteropathy, 718 Case 60: Dog with exocrine pancreatic insuffi ciency, 721

Endocrine Disorders Case 61: Dog with hypothyroidism, 723 Case 62: Dog with diabetes and secondary

hypothyroidism, 724 Case 63: Dog with hypoadrenocorticism, 726 Case 64: Dog with hypoadrenocorticism, 728 Case 65: Dog with hypoadrenocorticism, 730 Case 66: Dog with hypoadrenocorticism, 732 Case 67: Dog with hyperadrenocorticism, 734 Case 68: Dog with iatrogenic Cushing ’ s disease, 736 Case 69: Dog with pituitary adenoma secreting ACTH, 738 Case 70: Dog with pituitary dependent

hyperadrenocorticism, 740 Case 71: Dog with hyperparathyroidism, 742 Case 72: Dog with primary hyperparathyroidism, 744 Case 73: Dog with humoral hypercalcemia of malignancy,

746 Case 74: Dog with hypoparathyroidism, 748

Perspective on use of this section Interpretation of laboratory data is an art that is developed through accumulated experience. The interactions and pat-terns of data related to disease diagnosis are complex. One also must develop an appreciation for magnitudes of abnor-mality that infl uence interpretation of each measurement. This case discussion appendix is designed to provide the reader with both experience and guidance in beginning to learn the art of interpretation. This art is then continually cultivated through real - time experience in the clinical setting.

The laboratory data are presented for each case in a form that allows the reader to learn from making his or her own effort at describing and interpreting data. Please note the following formatting: 1. Data are presented in conventional units. In some areas, international (SI) units are given; these are shown in italics. 2. Laboratory values that are abnormal and central to the interpretation are given in bold type.

Following each data set, an interpretive discussion is pre-sented. These narratives may be used by the reader for self - assessment of profi ciency in interpretation of data.

List of abbreviations The following list of abbreviations identifi es test results that are typically used in laboratory reports. The user should refer to these as needed while learning the content of labora-tory reports.

Alb Albumin ALP Alkaline phosphatase ALT Alanine aminotransferase

Page 5: VI Clinical Case Presentations - Wiley · Clinical Case Presentations ... Case 17: Dog with vitamin K antagonist (rodenticide) toxicity, 643 ... PT Prothrombin time

619

CA

SESClinical Case Presentations: Contents

An. Gap Anion gap aPTT Activated partial thromboplastin time AST Aspartate aminotransferase Bands Band neutrophils BUN Blood urea nitrogen Ca Calcium Calc. Osmolality Calculated osmolality Chol Cholesterol CK Creatine kinase CL Chloride Creat Creatinine Eos Eosinophils Epith cells Epithelial cells GGT Gamma glutamyl transferase Glob Globulin Gluc Glucose Hgb Hemoglobin Hpf High power fi elds Lymphs Lymphocytes MCHC Mean corpuscular hemoglobin

concentration MCV Mean cell volume Meas. Osmolality Measured osmolality

Metas Metamyelocytes Monos Monocytes Na Sodium NCC Total nucleated cell count (Also

commonly called WBC for “ white blood cell count ” .)

PCV Packed cell volume Phos Phosphorus PT Prothrombin time RBC Red blood cells Retics Reticulocytes SDH Sorbitol dehydrogenase Segs Segmented neutrophils Sp. Gr. Specifi c gravity T. Bili Total bilirubin TCO 2 Total CO 2 TP Total protein TP (P) Total plasma protein TP (S) Total serum protein Trig Triglyceride UN Urea nitrogen (same as BUN) WBCs White blood cells WRI Within reference interval(s)

Page 6: VI Clinical Case Presentations - Wiley · Clinical Case Presentations ... Case 17: Dog with vitamin K antagonist (rodenticide) toxicity, 643 ... PT Prothrombin time

SECTION VI Clinical Case Presentations

620

CA

SES

Case 1 Signalment and History: 11 - year - old male cat. Lethargy and polydipsia. One month ago PCV was 38%.

Hematology Reference Interval

PCV (%) 13 25 – 45

RBC ( × 10 6 / μ L) 1.55 5 – 11

Hgb (g/dL) 4.0 8 – 15

MCV (fL) 84 39 – 50

MCHC (g/dL) 31 33 – 37

Retics ( × 10 3 / μ L) 155,000 0 – 60,000

NCC ( × 10 3 / μ L) 20.6 5.5 – 19.5

Metas ( × 10 3 / μ L) 0.4 0

Bands ( × 10 3 / μ L) 0.8 0 – 0.3

Segs ( × 10 3 / μ L) 9.9 2.5 – 12.5

Lymphs ( × 10 3 / μ L) 1.4 1.5 – 7.0

Monos ( × 10 3 / μ L) 3.1 0 – 0.8

Eos ( × 10 3 / μ L) 0.2 0 – 1.5

Nucleated RBCs ( × 10 3 / μ L) 4.8 0

Platelets ( × 10 3 / μ L) Adequate 150 – 700

TP (P) (g/dL) 8.9 6.0 – 8.5

Hemopathology: Many Hemobartonella felis ( Mycoplasma

haemofelis ) organisms on erythrocytes. Occasional reactive

lymphocyte.

Histogram Solid line = patient; Dashed line = normal.

Rela

tive

Num

ber

24

45 84

Cell Volume (fl)300

Biochemical Profi le Reference Interval

Gluc (mg/dL) 249 67 – 124

BUN (mg/dL) 96 17 – 32

Creat (mg/dL) 6.6 0.9 – 2.1

Ca (mg/dL) 10.2 8.5 – 11

Phos (mg/dL) 7.9 3.3 – 7.8

TP (g/dL) 8.4 5.9 – 8.1

Alb (g/dL) 3.3 2.3 – 3.9

Glob (g/dL) 5.1 2.9 – 4.4

T. Bili (mg/dL) 0.3 0 – 0.3

Chol (mg/dL) 386 60 – 220

ALT (IU/L) 53 30 – 100

ALP (IU/L) 19 6 – 106

Na (mEq/L) 150 146 – 160

K (mEq/L) 4.9 3.7 – 5.4

CL (mEq/L) 127 112 – 129

TCO 2 (mEq/L) 10 14 – 23

An. gap (mEq/L) 18 10 – 27

Calc. osmolality (mOsm/kg) 337 290 – 310

Urinalysis (cystocentesis)

Color Yellow Urine Sediment

Transparency Cloudy WBCs/hpf 6 – 8

Sp. gr. 1.020 RBCs/hpf 1 – 2

Protein Negative Epith cells/hpf 1 – 3 transitional

Gluc 2 + Casts/lpf 0

Bilirubin Negative Crystals 0

Blood Negative Bacteria 0

pH 5.0 Other fat droplets

Ketones Negative

Page 7: VI Clinical Case Presentations - Wiley · Clinical Case Presentations ... Case 17: Dog with vitamin K antagonist (rodenticide) toxicity, 643 ... PT Prothrombin time

Case 1

621

CA

SES

Interpretive discussion

Hematology Packed cell volume, Hemoglobin, Red blood cell count: The cat is severely anemic; all measurements of red blood cell mass are decreased.

MCV: The mean cell volume is increased, which one would expect with a regenerative anemia. However, the increase is greater than can be accounted for by the reticu-locytes, suggesting that agglutination is causing the increase, since doublets are being counted as one erythrocyte. This is confi rmed by the histogram tailing to the right.

Reticulocytes are increased, indicating that the anemia is regenerative. Regenerative anemia is due to blood loss or blood destruction. The protein concentration suggests hemo-lysis and tends to rule out blood loss.

Nucleated RBCs are increased due to early marrow release, and are often present in a regenerative anemia.

Erythrocyte morphology: The presence of Hemobartonella felis ( Mycoplasma haemofelis ) organisms explains the anemia (blood destruction). Agglutination is likely due to the pres-ence of antibodies against the organisms.

Monocytosis and increased immature (band) neutrophils are indicative of an infl ammatory leukogram.

Lymphopenia is indicative of a stress component in the leukogram.

Total protein: Total protein is increased. In this patient, it is due to hyperglobulinemia (see biochemical profi le inter-pretation below).

Biochemical profi le The serum glucose concentration is moderately increased. This could be due to stress (glucocorticoid release), as the lymphopenia suggests, but could also be due to diabetes mellitus.

The BUN and serum creatinine concentrations are increased, and in the face of a urine specifi c gravity of only 1.020, is indicative of renal azotemia.

The serum phosphorus concentration is mildly increased, and is compatible with decreased glomerular fi ltration rate.

The serum total protein concentration is increased due to an increase in the globulin concentration. The increase in globulin should trigger protein electrophoresis to determine if gammopathy is monoclonal or polyclonal.

The serum cholesterol concentration is moderately increased. This may be due to metabolic disorders associated with diabetes mellitus.

Serum total CO 2 is decreased, suggesting metabolic acidosis.

The increased calculated serum osmolality is primarily due to hyperglycemia and increased BUN.

Urinalysis As evidenced by the relatively low urine specifi c gravity in the face of azotemia, the animal is not concentrating ade-quately, indicating renal dysfunction. The presence of glucose indicates that the renal threshold of glucose has been exceeded.

Summary

This animal had been previously diagnosed with diabetes mellitus and was not being controlled adequately. Hemobar-tonella felis (Mycoplasma haemofelis ) is often an opportunist in cats that are immunosuppressed. The hyperglobulinemia was polyclonal, indicating antigenic stimulation, possibly due to Mycoplasma felis.

Page 8: VI Clinical Case Presentations - Wiley · Clinical Case Presentations ... Case 17: Dog with vitamin K antagonist (rodenticide) toxicity, 643 ... PT Prothrombin time

SECTION VI Clinical Case Presentations

622

CA

SES

Case 2 Signalment: 5 - year - old spayed female cocker spaniel History: Acutely lethargic Physical Examination: Pale, slightly icteric mucous membranes

Hematology Reference Interval

PCV (%) 12 37 – 55

Hgb (g/dL) 3.6 12 – 18

RBC ( × 10 6 / μ L) 0.95 5.5 – 8.5

MCV (fL) 114 60 – 72

MCHC (g/dL) 30 34 – 38

Retics ( × 10 3 / μ L) 123 < 60

NCC ( × 10 3 / μ L) 96.1 6 – 17

Segs ( × 10 3 / μ L) 69.1 3 – 11.5

Bands ( × 10 3 / μ L) 6.7 0 – 0.3

Metas ( × 10 3 / μ L) 1.0 0

Monos ( × 10 3 / μ L) 5.8 0.1 – 1.3

Lymphs ( × 10 3 / μ L) 0 1 – 4.8

Eos ( × 10 3 / μ L) 0 0.1 – 1.2

NRBCs ( × 10 3 / μ L) 13.5 0

Platelets ( × 10 3 / μ L) 284 200 – 500

TP (P) (g/dL) 6.8 6 – 8

Hemopathlogy: Polychromasia increased, agglutination present, many

spherocytes present. Occasional Howell - Jolly body.

Histogram Solid line = patient; Dashed line = normal.

Biochemical Profi le Reference Interval

Gluc (mg/dL) 143 75 – 130

BUN (mg/dL) 39 7 – 28

Creat (mg/dL) 1.3 0.9 – 1.7

Ca (mg/dL) 9.0 9.0 – 11.2

Phos (mg/dL) 4.4 2.8 – 6.1

TP (g/dL) 6.5 5.4 – 7.4

Alb (g/dL) 3.3 2.7 – 4.5

Glob (g/dL) 3.2 1.9 – 3.4

T. Bili (mg/dL) 4.7 0 – 0.4

Chol (mg/dL) 269 130 – 370

ALT/ μ L (IU//L) 32 10 – 120

AST/ μ L (IU//L) 30 16 – 40

ALP (IU//L) 438 35 – 280

Na (mEq/L) 146 145 – 158

K (mEq/L) 5.0 4.1 – 5.5

CL (mEq/L) 118 106 – 127

TCO 2 (mEq/L) 14 14 – 27

An. gap (mEq/L) 19 8 – 25

Rela

tive

Num

ber

24

65 114

Cell Volume (fl)300

Urinalysis (catheterized)

Color Orange Urine Sediment

Transparency Turbid WBCs/hpf 0

Sp. Gr. 1.038 RBCs/hpf 10

Protein 1 + Epith Cells/hpf 0

Gluc Negative Casts/lpf 0

Bilirubin 2 + Crystals Numerous

Bilirubin

Blood 1 + Bacteria 0

pH 6.0

Page 9: VI Clinical Case Presentations - Wiley · Clinical Case Presentations ... Case 17: Dog with vitamin K antagonist (rodenticide) toxicity, 643 ... PT Prothrombin time

Case 2

623

CA

SES

BUN is mildly increased, suggesting decreased GFR or bleeding into the GI tract. Since the creatinine is within the reference interval, and the dog is concentrating urine, this is likely prerenal azotemia, due to GI hemorrhage (high protein diet) or decreased blood fl ow to the kidneys.

Bilirubin is increased, indicating either cholestasis or increased red blood cell destruction. Because the dog has immune - mediated hemolytic anemia, increased RBC destruction is most likely the cause.

Serum alkaline phosphatase activity is increased, which could be due to either cholestasis or previous treatment with corticosteroids.

Urinalysis Bilirubinuria and the presence of bilirubin crystals refl ect the increased serum bilirubin concentration. Conjugated bilirubin readily passes through glomeruli and is then excreted in the urine. Blood and protein may be present due to traumatic catheterization. The animal is concentrating, indicating that the increase in BUN is not due to renal dysfunction.

Summary

This is a typical case of immune - mediated hemolytic anemia. Dog was treated with prednisone and recovered. It had been previously treated with corticosteroids, accounting for the stress leukogram, hyperglycemia, and increased serum alka-line phosphatase activity.

Interpretive discussion

Hematology Packed cell volume, Hemoglobin, Red blood cell count: The dog is very anemic, as all measurement of red blood cells mass are decreased. The RBC count is likely erroneously decreased, due to erythrocyte agglutination, and groups of red blood cells being counted as one red blood cell.

MCV: The mean cell volume is erroneously increased due to agglutination. Although the anemia is regenerative, the MCV is much higher than can be accounted for by an increase in reticulocytes. As one can see on the histogram, erythrocytes that are agglutinated are being counted as one large erythrocyte (note histogram tailing to right).

Reticulocytes are increased, indicating that the anemia is regenerative, suggesting either blood loss or blood destruc-tion. The protein and RBC morphology fi ndings indicate hemolysis. The decreased MCHC may be attributed to regeneration.

Nucleated RBCs are increased, likely due to early release from bone marrow in response to marked anemia. However, it is also possible that the dog has decreased splenic function secondary to glucocorticosteroid administration.

Erythrocyte morphology: Presence of spherocytes and agglutination, in the absence of a previous blood transfu-sion, are indicative of immune - mediated hemolytic anemia.

Neutrophilia, increased immature neutrophils, and monocytosis are indicative of a markedly infl ammatory leukogram. The absence of lymphocytes is suggestive of stress or corticosteroids. Infl ammatory leukograms are com-monly seen in association with immune - mediated hemolytic anemia.

Biochemical profi le Glucose is mildly increased. Considering the lymphopenia, this may be due to stress or steroids.

Page 10: VI Clinical Case Presentations - Wiley · Clinical Case Presentations ... Case 17: Dog with vitamin K antagonist (rodenticide) toxicity, 643 ... PT Prothrombin time

SECTION VI Clinical Case Presentations

624

CA

SES

Interpretive discussion

Hematology Packed cell volume, Hemoglobin, Red blood cell count: Cat is markedly anemic. Reticulocytes are not increased, indicat-ing that the anemia is nonregenerative.

MCV is markedly increased, in the absence of reticulocy-tosis or agglutination. In a cat, this should trigger testing for feline leukemia virus, as the MCV may be increased as a result of viral - induced erythrodysplasia. Macrocytosis with widened histogram is often seen in FeLV positive cats with anemia.

Neutrophilia, increased band neutrophils and metamyelo-cytes, and monocytosis are indicative of infl ammation.

Nucleated red blood cells are increased in the absence of reticulocytes. Moreover, many of these are quite immature, indicating that the cat has leukemia involving the erythrocytes.

Summary

Myeloproliferative disorder, erythremic myelosis or M6(E).

Case 3 Signalment and History: 11 - year - old DSH spayed female cat presented for anorexia and lethargy

Hematology Reference Interval

PCV (%) 13 25 – 45

RBC ( × 10 6 / μ L) 1.84 5 – 11

Hgb (g/dL) 4.2 8 – 15

MCV (fL) 71 39 – 50

MCHC (g/dL) 32 33 – 37

Retics ( × 10 3 / μ L) 7,360 0 – 60,000

Nucleated cells ( × 10 3 / μ L) 71.3 5.5 – 19.5

Metas ( × 10 3 / μ L) 0.7 0

Bands ( × 10 3 / μ L) 2.1 0 – 0.3

Segs ( × 10 3 / μ L) 33.2 2.5 – 12.5

Lymphs ( × 10 3 / μ L) 2.8 1.5 – 7.0

Monos ( × 10 3 / μ L) 6.9 0 – 0.8

NRBCs ( × 10 3 / μ L) 24.9 0

Blasts ( × 10 3 / μ L) 0.7 0

Platelets ( × 10 3 / μ L) Adequate 150 – 700

TP (P) (g/dL) 8.0 6.0 – 8.5

Hemopathology: Blasts appear to be rubriblasts. Many prorubricytes

and rubricytes also present.

Histogram Solid line = patient; Dashed line = normal.

Rela

tive

Num

ber

24

45 71

Cell Volume (fl)300

Page 11: VI Clinical Case Presentations - Wiley · Clinical Case Presentations ... Case 17: Dog with vitamin K antagonist (rodenticide) toxicity, 643 ... PT Prothrombin time

Case 4

625

CA

SES

Case 4 Signalment and History: 17 - year - old male cat presented for lethargy and enlarged abdomen. Liver disease suspected, but biochemical profi le normal.

Interpretive discussion

Hematology Packed cell volume: The cat is mildly anemic.

MCV: The mean cell volume is prominently decreased. Decreased mean cell volume is almost always caused by iron defi ciency anemia, which in adults is almost always second-ary to chronic external blood loss. The borderline decrease in MCHC is not important diagnostically.

Erythrocyte morphology: Keratocytes are commonly asso-ciated with iron defi ciency anemia. Iron defi ciency anemia is not as common in cats as in dogs, and the few cases we have seen did not have increased central pallor.

The total leukocyte count and the mature neutrophil con-centration are within the reference interval, but the increase in band neutrophils is indicative of infl ammation. Lympho-penia is indicative of stress or previous corticosteroid administration.

Total protein is within the reference interval. Although one might expect total protein to be decreased with chronic blood loss, animals often compensate for this chronic loss of protein.

Summary

Owner declined further diagnostic evaluation. Chronic GI blood loss secondary to an intestinal tumor would be the most likely diagnosis in this aged patient with iron defi -ciency anemia.

Hematology Reference Interval

PCV (%) 24 25 – 45

MCV (fL) 33 39 – 50

MCHC (g/dL) 32 33 – 37

Retics ( × 10 3 / μ L) ND 0 – 60,000

Nucleated cells ( × 10 3 / μ L) 13.2 5.5 – 19.5

Bands ( × 10 3 / μ L) 4.5 0 – 0.3

Segs ( × 10 3 / μ L) 6.6 2.5 – 12.5

Lymphs ( × 10 3 / μ L) 0.5 1.5 – 7.0

Monos ( × 10 3 / μ L) 0.5 0 – 0.8

Eos ( × 10 3 / μ L) 0.3 0 – 1.5

Basophils ( × 10 3 / μ L) 0.8 rare

Platelets ( × 10 3 / μ L) Adequate 150 – 700

TP (P) (g/dL) 6.6 6.0 – 8.5

Hemopathology: Many keratocytes, schistocytes.

Histogram Solid line = patient; Dashed line = normal.

Rela

tive

Num

ber

24

33 45

Cell Volume (fl)300

Page 12: VI Clinical Case Presentations - Wiley · Clinical Case Presentations ... Case 17: Dog with vitamin K antagonist (rodenticide) toxicity, 643 ... PT Prothrombin time

SECTION VI Clinical Case Presentations

626

CA

SES

Interpretive discussion

Hematology Packed cell volume, Hemoglobin: Both are decreased, indi-cating that the dog is anemic. The red blood cell count is within the reference interval, suggesting erythrocytes are small.

MCV: The mean cell volume is decreased. Decreased mean cell volume is almost always caused by iron defi ciency anemia, which in adults is almost always secondary to chronic external blood loss.

Reticulocytes are not increased, indicating that the anemia is nonregenerative. While uncomplicated iron defi ciency anemia is usually regenerative, this anemia may be nonre-generative due to the presence of infl ammation (note infl ammatory leukogram).

Erythrocyte morphology: Keratocytes, erythrocyte frag-mentation, and increased central pallor are commonly asso-ciated with iron defi ciency anemia.

Neutrophilia and increased immature neutrophils are indicative of a marked infl ammatory leukogram. The infl am-matory leukogram is compatible with the presence of infl ammation in the peritoneal cavity, although one would usually expect to see more band neutrophils in dogs with peritonitis. The presence of this infl ammation may be the explanation for the lack of a regenerative response to the anemia, as an anemia of infl ammatory disease may be superimposed on the iron defi ciency anemia. The noted occasional granulated lymphocyte is interpreted as an inci-dental fi nding.

Platelets are increased. Approximately half of all animals with iron defi ciency anemia have increased platelets, prob-ably in response to cytokines and growth factors.

Total protein: Total protein is slightly decreased, probably as a result of chronic blood loss.

Histogram confi rms the presence of a population of micro-cytic cells (normal histogram represented by dashed line).

Abdominal fl uid analysis Suppurative septic infl ammation. The presence of different types of bacteria suggests a GI source of bacteria.

Summary

The dog died, and on necropsy had an intestinal perforation secondary to an ulcer, chronic diffuse peritonitis, pyogranu-lomatous lymphadenitis and amyloidosis of the spleen, liver, and kidney. Presumably, the dog had been chronically bleed-ing from this ulcer, resulting in iron defi ciency anemia.

Case 5 Signalment: 1 - year - old pointer History: Treated for neck or back pain with corticosteroids by referring veterinarian. Dog was thought to have GI parasites due to occult blood in feces, and was treated with anthelmintics. The dog returned 1 month later with a PCV of 15% and MCV of 40 fl . At that time the dog had an abdominal effusion. Physical Examination: Painful abdomen, pale mucous membranes

Hematology Reference Interval

PCV (%) 18 37 – 55

Hgb (g/dL) 3.76 12 – 18

RBC ( × 10 6 / μ L) 5.8 5.5 – 8.5

MCV (fL) 47 60 – 72

MCHC (g/dL) 33 33 – 38

Retics ( × 10 3 / μ L) 18 < 60

NCC ( × 10 3 / μ L) 40.1 6 – 17

Segs ( × 10 3 / μ L) 36.5 3 – 11.5

Bands ( × 10 3 / μ L) 0.4 0 – 0.3

Metas ( × 10 3 / μ L) 0.4 0

Monos ( × 10 3 / μ L) 1.2 0.1 – 1.3

Lymphs ( × 10 3 / μ L) 1.2 1 – 4.8

Eos ( × 10 3 / μ L) 0.4 0.1 – 1.2

Platelets ( × 10 3 / μ L) 623 200 – 500

TP (P) (g/dL) 5.9 6 – 8

Hemopathology: Numerous keratocytes, few schistocytes, some RBCs

appear hypochromic. Occasional lymphocyte with azurophilic

granules.

Histogram Solid line = patient; Dashed line = normal:

Rela

tive

Num

ber

24

47 65

Cell Volume (fl)300

Abdominal Fluid Analysis

NCC ( μ L) 90,000

TP (g/dL) 4.0

Cytology: All cells are degenerate neutrophils. Bacteria of various

types are present.

Page 13: VI Clinical Case Presentations - Wiley · Clinical Case Presentations ... Case 17: Dog with vitamin K antagonist (rodenticide) toxicity, 643 ... PT Prothrombin time

Case 6

627

CA

SES

Case 6

Hematology Reference Interval

PCV (%) 12 37 – 55

RBC ( × 10 6 / μ L) 2.76 5.5 – 8.5

Hgb (g/dL) 3.2 12 – 18

MCV (fL) 40 60 – 72

MCHC (g/dL) 29 34 – 38

Retics ( × 10 3 / μ L) 242,880 0 – 60,000

Nucleated cells ( × 10 3 / μ L) 33.4 6.0 – 17.0

Metas ( × 10 3 / μ L) — 0

Bands ( × 10 3 / μ L) — 0 – 0.3

Segs ( × 10 3 / μ L) 30.7 3.0 – 11.5

Lymphs ( × 10 3 / μ L) 1.0 1.0 – 4.8

Monos ( × 10 3 / μ L) 1.0 0.2 – 1.4

Eos ( × 10 3 / μ L) — 0.1 – 1.2

NRBCs ( × 10 3 / μ L) 0.7 0

Platelets ( × 10 3 / μ L) Adequate 200 – 500

TP (P) (g/dL) 6.3 6.0 – 8.0

Hemopathology: Increased central pallor, occasional keratocyte, giant

platelets.

Signalment and History: 9 year - old female beagle presented for lethargy and pale mucous membranes. Owner reported that the dog occasionally had blood in feces.

Interpretive discussion

Hematology Packed cell volume, Hemoglobin, Red blood cell count: The dog is markedly anemic; all measurements of red blood cell mass are decreased.

MCV: The mean cell volume is markedly decreased. Decreased mean cell volume is almost always caused by iron defi ciency anemia, which in adults is almost always second-ary to chronic external blood loss. The MCHC may be mildly decreased in iron defi ciency as it is here. The reticulocytosis may also contribute to the decreased MCHC.

Reticulocytes are increased, indicating that the anemia is regenerative, suggesting blood loss or blood destruction. In this case, the decreased MCV strongly suggests iron defi -ciency anemia secondary to chronic blood loss. The presence of nucleated red blood cells is compatible with this degree of regenerative response.

Erythrocyte morphology: Keratocytes and increased central pallor are commonly associated with iron defi ciency anemia.

Neutrophilia is indicative of infl ammation, even though no band neutrophils are present, since the neutrophil con-centration is greater than two fold upper reference interval. The lymphocyte count is in the low normal range, indicating that there may be a stress or steroid component to the leukogram.

Total protein is within the reference interval. Although one might expect total protein to be decreased with chronic blood loss, animals often compensate for this chronic loss of protein.

Summary

GI barium series performed and jejunal mass seen. At surgery, a mass in the mid - jejunum was resected and deter-mined to be a leiomyosarcoma with clean surgical margins. The regenerative response in this case is in contrast to the previous case to make the point that iron defi ciency anemia may be either regenerative or nonregenerative.

Page 14: VI Clinical Case Presentations - Wiley · Clinical Case Presentations ... Case 17: Dog with vitamin K antagonist (rodenticide) toxicity, 643 ... PT Prothrombin time

SECTION VI Clinical Case Presentations

628

CA

SES

Case 7 Signalment Five - year - old mixed breed dog History Lethargy Physical examination Lethargic, pale mucous membranes

Hematology 1/20 * 1/28 Reference Interval

Packed cell volume (%) (spun) 14 30 37 – 55

Hematocrit (%) (calculated) 12.4 21.3 37 – 55

Hgb (g/dL) 4.4 9.0 12 – 18

RBC (10 6 / μ L) 4.26 5.77 5.5 – 8.5

MCV (fL) 29 37 60 – 77

MCHC (g/dL) 35.2 42.1 31 – 34

RDW 23.8 31.7 12 – 15

Platelets(x10 3 / μ L) 803 883 200 – 500

Reticulocytes (x10 3 / μ L) 102 403 0 – 60

Total Protein (g/dL) 5.3 6.2 6 – 8

* sample moderately lipemic

Sample ID:

Patient ID:

Name:

Test time:

Mode:

WBCLYM 1.90 10ˆ9/l

MON 0.89 10ˆ9/l

GRA 9.63 10ˆ9/lLY% 15.3 %

MO% 7.2 + %

GR% 77.5 %

RBC 5.77 10ˆ12/l43 78103

WBC

VetScan HM2

11519818 Doctor: Gillespie

Fluffy Age/Sex: 5 years / Male

Dog Report date:

01.28.2010 10:36 AM

01.28.2010 10:36 AM

S/N: 270107

RBC

14

PLT11.13.9

200 500

31 34

19.5 24.560 77

37 55

12 18

5.5 8.5

62 87

12 30

3 12

0.2 1.5

1 4.8

6 17

2 4

14 50

200

400

10ˆ9/l12.41

HGB 9.0

HCT 21.30

MCV 37

MCH 15.6

MCHC 42.1

RDWc 31.7 %

PLT

PCTMPV

PDWc 36.6 %

8.0 fl

0.70 %

883 10ˆ9l

– g/dl

%

fl

– pg

+ g/dl

+

1/20 histogram and blood fi lm

Page 15: VI Clinical Case Presentations - Wiley · Clinical Case Presentations ... Case 17: Dog with vitamin K antagonist (rodenticide) toxicity, 643 ... PT Prothrombin time

Case 7

629

CA

SES

erythrocytes, as well as keratocytes and schistocytes, eryth-rocyte shape changes and fragmentation that are very char-acteristic of iron defi cient erythrocytes. The dog was treated with iron. Note that one week later on 1/28 the MCV increased, and the new normal - sized erythrocytes are evident in the erythrocyte histogram on 1/28. They are represented by the additional curve on the right side of the histogram as well as on the blood fi lm made on 1/28.

MCHC The MCHC is miscalculated to be high on 1/28 likely as a result of the undercounted RBCs. The MCHC is probably erroneously increased on 1/20 as well. The MCHC is calcu-lated from the hemoglobin concentration and the HCT using the following formula: HGB (g/dL) divided by the HCT (%) x 100 = MCHC (g/dL). It provides an index for the quantity of hemoglobin relative to the volume of packed erythro-cytes. It may be erroneously increased due to lipemia, hemo-lysis, or an erroneously low HCT. On 1/28, it is likely erroneously high due to the erroneously low HCT. On 1/20, it may also be erroneously high due to lipemia artifactually increasing the HGB.

RDW The increased variation in erythrocyte size (anisocytosis) is represented by the increased red cell distribution width (RDW) on 1/28, a numerical expression (coeffi cient of varia-tion) that correlates with the degree of anisocytosis and which the instrument determines by dividing the standard

Interpretive discussion

Hematology PCV and Hematocrit: Decreased, indicating anemia. Note the marked discrepancy in the calculated hematocrit (HCT) and the spun packed cell volume (PCV). The HCT is calculated by the instrument using the following formula: (MCV x RBC)/10 = HCT(%). In the case of the sample from 1/28/10, the RBC count may be under - reporting the sample ’ s RBC concentration since some small RBCs are likely appearing in the PLT channel. The lower reported RBC count is also likely due to the fact that two overlapping RBC peaks are now present in the RBC histogram from 1/28/10; the two over-lapping peaks will be underestimated since the software expects only one RBC peak to be present and will not apply curve fi tting algorithms here. Since the RBC count is under-estimated, the HCT calculated here is lower than the reported PCV.

RBC The RBCs are decreased as a result of anemia, but they are likely undercounted in this patient due to their small size, and are likely being counted in the platelet channel (see the platelet histogram below the erythrocyte histogram).

MCV The decreased MCV indicates a severe microcytic anemia. The degree of microcytosis is illustrated by the RBC histo-gram on 1/20. The blood fi lm contained many hypochromic

1/28 histogram and blood fi lm

Sample ID:

Patient ID:

Name:

Test time:

Mode:

WBCLYM 2.03 10ˆ9/l

MON 0.43 10ˆ9/l

GRA 6.40 10ˆ9/lLY% 22.9 %

MO% 4.8 + %

GR% 72.3 %

RBC 4.26 10ˆ12/l–

+

48 104115

WBC

VetScan HM2

11009818 Doctor: Gillespie

Fluffy Age/Sex: 5 years / Male

Dog Report date:

01.28.2010 09:31 AM

01.28.2010 09:31 AM

S/N: 270107

RBC

14

PLT

14 50

200

400

10ˆ9/l8.86

HGB 4.4 g/dl

HCT 12.39 %

MCV 29 fl

MCH 10.3 pg

MCHC 35.2 g/dl

RDWc 23.8 %

PLT

PCTMPV

PDWc 36.6 %

8.3 fl

0.67 %

803 + 10ˆ9/l

11.13.9

200 500

31 34

19.5 24.560 77

37 55

12 18

5.5 8.5

62 87

12 30

3 12

0.2 1.5

1 4.8

6 17

2 4

Page 16: VI Clinical Case Presentations - Wiley · Clinical Case Presentations ... Case 17: Dog with vitamin K antagonist (rodenticide) toxicity, 643 ... PT Prothrombin time

SECTION VI Clinical Case Presentations

630

CA

SES

deviation of the red cell size by the MCV. Both iron defi cient microcytic erythrocytes and young macrocytic erythrocytes can also be seen on the blood fi lm.

Platelets The platelet concentration is markedly increased, as is often the case in patients with iron defi ciency anemia. Some of the platelets are quite large. Also, some microcytic RBC are counted as PLT, falsely increasing the PLT concentration.

Reticulocytes The reticulocyte concentration is increased, indicating a regenerative anemia.

Total protein The total protein is slightly decreased on 1/20, likely due to chronic blood loss (protein is lost as well as erythrocytes).

Summary

Iron defi ciency anemia as indicated by marked microcytosis.

Outcome The diagnosis of iron defi ciency anemia was confi rmed by measuring serum iron, which was 27 μ g/dL (Reference interval = 98 – 220 μ g/dL). The source of chronic blood loss is usually the gastrointestinal tract. An occult blood test was performed on the feces on multiple occasions, and was con-sistently negative. An examination of feces for the eggs of parasites such as hookworms was also negative. Further history revealed that the owner had given the dog an anthel-mintic two weeks prior to presentation, presumably elimi-nating a hookworm infestation. The dog was treated with 15 mg/kg iron dextran IM, and the owner was instructed to give him 162.5 mg Ferrous sulfate orally once daily for 4 weeks. The hematocrit and MCV were normal at his follow - up examination 6 months later.

Page 17: VI Clinical Case Presentations - Wiley · Clinical Case Presentations ... Case 17: Dog with vitamin K antagonist (rodenticide) toxicity, 643 ... PT Prothrombin time

Case 8

631

CA

SES

Interpretive discussion

Hematology Packed cell volume, Hemoglobin, Red blood cell count: The dog is anemic; all measurements of red blood cell mass are decreased.

MCV: The mean cell volume is normal. However, it is surprising that it is not higher considering that the reticulo-cyte count is increased.

Reticulocytes are increased, indicating that the anemia is regenerative, and is thus due to blood loss or blood destruc-tion. The borderline low plasma protein suggests blood loss is likely; this will be confi rmed by additional fi ndings. Nucle-ated RBCs are increased due to early marrow release as part of regeneration.

Erythrocyte morphology: Acanthocytes are commonly seen in dogs with hemangiosarcoma. The schistocytes are suggestive of microangiopathy, which may also be associated with hemangiosarcoma.

Neutrophilia, increased immature (band) neutrophils, and monocytosis are indicative of an infl ammatory leukogram, although a component of the mature neutrophilia is likely due to stress or corticosteroids. Lymphopenia is indicative of stress.

Platelets are slightly decreased. Considering the presence of schistocytes, the animal may have DIC.

Total protein: Total protein is within the reference inter-val. However, considering that it was 8.2 g/dL 4 months previously, it is likely decreased due to blood loss within the abdominal cavity.

Abdominal fl uid analysis

Hemoabdomen.

Summary

The signalment (large breed, older dog), history (episodes of weakness), regenerative anemia, erythrocyte morphology, and the hemoabdomen are all suggestive of hemangiosar-coma. An exploratory was performed, and the dog had hem-angiosarcoma of the spleen and liver, which had ruptured. Previous episodes of weakness were likely due to previous ruptures of the tumor, which had subsequently sealed, then ruptured again.

Signalment: 10 - year - old castrated male Labrador retriever History: Four episodes of acute weakness over past 3 months. At time of wellness exam 4 months ago, dog had PCV of 44% and T.P. of 8.2 g/dL. Physical examination: Pale mucous membranes, abdomen slightly distended

Hematology Reference Interval

PCV (%) 16 37 – 55

Hgb (g/dL) 5.3 12 – 18

RBC ( × 10 6 / μ L) 2.48 5.5 – 8.5

MCV (fL) 63 60 – 72

MCHC (g/dL) 34 34 – 38

Retics ( × 10 3 / μ L) 342 < 60

NCC ( × 10 3 / μ L) 39.1 6 – 17

Segs ( × 10 3 / μ L) 33.2 3 – 11.5

Bands ( × 10 3 / μ L) 1.2 0 – 0.3

Monos ( × 10 3 / μ L) 3.1 0.1 – 1.3

Lymphs ( × 10 3 / μ L) 0.4 1 – 4.8

Eos ( × 10 3 / μ L) 0.4 0.1 – 1.2

NRBCs ( × 10 3 / μ L) 0.8 0

Platelets ( × 10 3 / μ L) 130 200 – 500

TP (P) (g/dL) 6.2 6 – 8

Hemopathology: Polychromasia increased, numerous acanthocytes

and schistocytes. Numerous Howell - Jolly bodies

Case 8

Biochemical Profi le

No abnormalities

Abdominal Fluid Analysis

PCV (%) 24

NCC ( × 10 3 / μ L) 34,000

Cytology: 95% nondegenerate neutrophils; 5% macrophages, many

of which have phagocytized erythrocytes

Page 18: VI Clinical Case Presentations - Wiley · Clinical Case Presentations ... Case 17: Dog with vitamin K antagonist (rodenticide) toxicity, 643 ... PT Prothrombin time

SECTION VI Clinical Case Presentations

632

CA

SES

Interpretive discussion

Hematology Packed cell volume, Hemoglobin, Red blood cell count: The dog is anemic; all measurements of red blood cell mass are decreased.

Reticulocytes are increased in January, indicating that the anemia is regenerative, suggesting either blood loss or blood destruction. Although the PCV is further decreased in October, the anemia is no longer regenerative, suggesting bone marrow dysfunction.

Neutrophilic leukocytosis with left shift and monocytosis are indicative of an infl ammatory leukogram (January).

Lymphocytosis in January is most suggestive of either lymphocytic leukemia or ehrlichiosis.

Platelets: The animal is markedly thrombocytopenic in October. The combination of thrombocytopenia and nonre-generative anemia should trigger a bone marrow aspirate examination and ehrlichia titer. Common causes of throm-bocytopenia include ehrlichiosis, immune - mediated throm-bocytopenia, and DIC.

Total protein: Total protein is increased. In this patient, it is due to hyperglobulinernia, which should trigger protein electrophoresis.

The presence of increased rouleaux is compatible with increased globulin. The presence of large granular lympho-cytes is suggestive of certain types of antigenic stimulation, commonly ehrlichiosis, or a leukemia of LGL cells. The pres-ence of Hemobartonella canis ( Mycoplasma haemocanis ) organ-isms in October suggests either a previous splenectomy or splenic dysfunction, since the erythrocyte parasite is rarely seen in dogs with functional spleens. The anemia is no longer regenerative in the face of anemia and this erythro-cyte parasite, suggesting marrow impairment of some type, and a bone marrow aspirate is indicated.

Summary

In January, the anemia was possibly due to blood loss associ-ated with a large hematoma of the spleen, and the dog was splenectomized. The lymphocytosis, hyperglobulinemia, and presence of large granular lymphocytes should have trig-gered an ehrlichia titer, but did not. The animal returned in October, severely anemic and thrombocytopenic. An ehrlichia titer was done at this time, and was strongly posi-tive. The dog was treated for ehrlichiosis and hemobartonel-losis, and recovered.

Case 9 Signalment: 15 - year - old Staffordshire terrier History: Lethargic Physical examination: Pale mucous membranes

Hematology January October Reference Interval

PCV (%) 30 28 37 – 55

RBC ( × 10 6 / μ L) 4.70 4.44 5.5 – 8.5

Hgb (g/dL) 10.1 9.5 12 – 18

MCV (fL) 61 64 60 – 72

MCHC (g/dL) 35 34 34 – 38

Retics ( × 10 3 / μ L) 178,000 13,200 0 – 60,000

NCC ( × 10 3 / μ L) 23.4 10.2 6.0 – 17.0

Bands ( × 10 3 / μ L) 0.5 0.2 0 – 0.3

Segs ( × 10 3 / μ L) 15.7 6.2 3.0 – 11.5

Lymphs ( × 10 3 / μ L) 6.1 1.5 1.0 – 4.8

Monos ( × 10 3 / μ L) 0.7 1.7 0.2 – 1.4

Eos ( × 10 3 / μ L) — 0.1 – 1.2

NRBCs ( × 10 3 / μ L) — 0

Platelets ( × 10 3 / μ L) 150 12 200 – 500

TP (P) (g/dL) 8.2 5.6 6.0 – 8.0

Alb 1.5 2.7 – 4.5

Glob 6.0 1.9 – 3.4

Hemopathology (January): Increased rouleaux, giant platelets, lymphs

contain azurophilic granules.

Hemopathology (October): Increased rouleaux, lymphs contain

azurophilic granules. numerous Hemobartonella canis ( Mycoplasma

haemocanis ) organisms present.

Page 19: VI Clinical Case Presentations - Wiley · Clinical Case Presentations ... Case 17: Dog with vitamin K antagonist (rodenticide) toxicity, 643 ... PT Prothrombin time

Case 10

633

CA

SES

Interpretive discussion

Packed cell volume and hemoglobin are slightly decreased, indicating mild anemia.

Reticulocytes are not increased, indicating that the anemia is nonregenerative. Considering the infl ammatory leuko-gram, this is most likely an anemia of infl ammatory disease.

Marked neutrophilia with increased immature neutro-phils and monocytosis is indicative of a chronic infl amma-tory leukogram.

Lymphopenia is indicative of a stress or steroid component to the leukogram.

Platelets are decreased. Thrombocytopenia is most com-monly due to ehrlichiosis, immune - mediated thrombocyto-penia or DIC. This should trigger other coagulation tests. The presence of giant platelets suggests that immature platelets are being released by the bone marrow, and the thrombo-cytopenia is not due to bone marrow dysfunction.

Summary

Anemia of infl ammatory disease. Site of infl ammation was a prostatic abscess. DIC was confi rmed.

Signalment and history: 9 - year - old male castrated dog presented for lethargy

Case 10

Hematology Reference Interval

PCV (%) 36 37 – 55

RBC ( × 10 6 / μ L) 5.42 5.5 – 8.5

Hgb (g/dL) 13.2 12 – 18

MCV (fL) 66 60 – 72

MCHC (g/dL) 37 34 – 38

Retics ( × 10 3 / μ L) 0 0 – 60,000

NCC ( × 10 3 / μ L) 96.4 6.0 – 17.0

Metas ( × 10 3 / μ L) — 0

Bands ( × 10 3 / μ L) 7.7 0 – 0.3

Segs ( × 10 3 / μ L) 82.9 3.0 – 11.5

Lymphs ( × 10 3 / μ L) 1.0 1.0 – 4.8

Monos ( × 10 3 / μ L) 4.8 0.2 – 1.4

Eos ( × 10 3 / μ L) — 0.1 – 1.2

NRBCs ( × 10 3 / μ L) — 0

Platelets ( × 10 3 / μ L) 39 200 – 500

TP (P) (g/dL) 6.2 6.0 – 8.0

Hemopathology: Decreased platelets, giant platelets, toxic neutrophils,

numerous echinocytes, occasional schistocyte.

Page 20: VI Clinical Case Presentations - Wiley · Clinical Case Presentations ... Case 17: Dog with vitamin K antagonist (rodenticide) toxicity, 643 ... PT Prothrombin time

SECTION VI Clinical Case Presentations

634

CA

SES

Case 11 Signalment: 4 - year - old Doberman History: Acutely ill, vomiting Physical examination: Pendulous abdomen

Hematology Day 1 *

Day 2 Reference Interval

PCV (%) 50 20 37 – 55

Hgb (g/dL) 18.3 7.5 12 – 18

RBC ( × 10 6 / μ L) 7.70 3.11 5.5 – 8.5

MCV (fL) 66 66 60 – 72

MCHC (g/dL) 36 37 34 – 38

Retics ( × 10 3 / μ L) ND 124 < 60

NCC ( × 10 3 / μ L) 6.6 14.7 6 – 17

Segs ( × 10 3 / μ L) 0.4 4.1 3 – 11.5

Bands ( × 10 3 / μ L) 3.1 7.9 0 – 0.3

Metas ( × 10 3 / μ L) 0.1 1.5 0

Monos ( × 10 3 / μ L) 0.5 0.3 0.1 – 1.3

Lymphs ( × 10 3 / μ L) 2.1 0.4 1 – 4.8

Eos ( × 10 3 / μ L) 0.1 0.1 0.1 – 1.2

Platelets ( × 10 3 / μ L) 193 90 200 – 500

TP (P) (g/dL) 5.9 4.0 6 – 8

Hemopathology: marked toxic neutropils on Days 1 and 2.

* Had abdominal exploratory surgery the evening of Day 1; treated

with fl uids between Day 1 and Day 2

Abdominal Fluid Analysis

NCC (/ μ L) 93,000

TP (g/dL) 1.5

Cytology: 100% degenerate neutrophils; various types of bacteria

phagocytized and extracellular.

Biochemical Profi le

Day 1 *

Day 2 Reference Interval

Gluc 26 36 75 – 130

BUN (mg/dL) 45 62 7 – 28

Creat (mg/dL) 0.6 1.8 0.9 – 1.7

Ca (mg/dL) 8.2 7.6 9.0 – 11.2

Phos (mg/dL) 5.9 11.0 2.8 – 6.1

TP (g/dL) 4.5 2.6 5.4 – 7.4

Alb (g/dL) 1.8 1.0 2.7 – 4.5

Glob (g/dL) 2.7 1.0 1.9 – 3.4

T. Bili (mg/dL) 0.1 3.0 0 – 0.4

Chol (mg/dL) 145 140 130 – 370

ALT (IU/L) 20 328 10 – 120

AST (IU/L) 77 775 16 – 40

ALP (IU/L) 208 440 35 – 280

GGT 1 1 0 – 6

Na (mEq/L) 136 143 145 – 158

K (mEq/L) 4.1 5.8 4.1 – 5.5

CL (mEq/L) 100 106 106 – 127

TCO 2 (mEq/L) 9.4 19.4 14 – 27

An. gap (mEq/L) 31 23 8 – 25

* Had abdominal exploratory surgery the evening of Day 1; treated

with fl uids between Day 1 and Day 2

Page 21: VI Clinical Case Presentations - Wiley · Clinical Case Presentations ... Case 17: Dog with vitamin K antagonist (rodenticide) toxicity, 643 ... PT Prothrombin time

Case 11

635

CA

SES

prerenal azotemia or renal azotemia. A urinalysis was not performed.

The serum calcium is decreased on both Days 1 and 2, and is due to the hypoalbuminemia.

The serum phosphorus concentration is increased on Day 2 and is compatible with decreased glomerular fi ltration rate.

The serum total protein concentration is decreased due to hypoalbuminemia on Day 1, and both hypoalbuminemia and hypoglobulinemia on Day 2 (see explanation above).

The serum bilirubin concentration is increased on Day 2, likely due to cholestasis related to septicemia.

The ALT and AST activity on Day 2 indicates hepatocel-lular injury, possibly related to anemia, shock, surgery, or septicemia.

The ALP activity is increased on Day 2, possibly related to endogenous corticosteroids or cholestasis.

Both sodium and chloride are decreased on Day 1, pos-sibly due to loss of electrolytes into abdominal effusion, or loss due to vomiting.

Serum total CO 2 is decreased on Day 1, suggesting meta-bolic acidosis. This has been corrected by Day 2, likely due to fl uid therapy.

The anion gap is increased on Day 1, likely due to lactic acid.

Abdominal fl uid The nucleated cell count is very high and all of the cells present are neutrophils, indicating suppurative infl amma-tion or peritonitis. The total protein may be low because the serum protein is decreased, or it may he diluted in the large volume of fl uid. The presence of different types of bacteria suggests that the source of bacteria is the gastrointestinal tract.

Summary

This dog had a consumptive infl ammatory leukogram and hypoglycemia due to sepsis. On exploratory, the abdominal cavity contained 1400 mL of fl uid, and a toothpick was found to have perforated the intestine. Dog died on the evening of Day 2 as a result of septic peritonitis.

Interpretive discussion

Hematology Packed cell volume, Hemoglobin, Red blood cell count: Within or near reference intervals on Day 1, markedly decreased on Day 2 following blood loss that occurred at the time of surgery. Dog seemed to bleed excessively during the surgery.

Reticulocytes are increased on Day 2, indicating that the anemia is regenerative. This regenerative response is earlier than is typically seen, in that reticulocytes don ’ t usually increase until 24 to 72 hours following the onset of anemia.

Neutropenia is present on Day 1, with an increase in immature neutrophils, indicating in this case that the mature neutrophils are being consumed in an infl ammatory process, and the marrow is not meeting the consumption demand. On Day 2, the mature neutrophils have increased, as have the immature neutrophils (bands and metamyelocytes). This indicates that the consumptive process has decreased (source of infl ammation) or that the marrow has increased produc-tion, or both.

Lymphopenia is indicative of a stress or steroid component to the leukogram.

Platelets are mildly decreased on Day 1, and more mark-edly decreased on Day 2. While some platelets may have been consumed in clotting process secondary to surgery related blood loss, it is also possible that the animal has DIC, particularly with the history of excessive bleeding during surgery. This should trigger additional tests such as FDPs, PT, APTT, and activated clotting time.

Total protein: Total protein is decreased on Day 1 and Day 2. In this patient, this is likely due to loss into the abdominal cavity on Day 1, compounded by blood loss on Day 2. Fluid administration may also be diluting the PCV and plasma protein on Day 2.

Biochemical profi le The serum glucose concentration is markedly decreased, both on Day 1 and Day 2. In this patient, considering the leukogram, this is most likely due to sepsis. Other differen-tials should include insulinoma, although this is a relatively young dog for an insulinoma.

The BUN is increased on both Day 1 and Day 2, and the serum creatinine is increased on Day 2. This may be either

Page 22: VI Clinical Case Presentations - Wiley · Clinical Case Presentations ... Case 17: Dog with vitamin K antagonist (rodenticide) toxicity, 643 ... PT Prothrombin time

SECTION VI Clinical Case Presentations

636

CA

SES

Interpretive discussion

The erythrocyte values indicate moderate anemia. The anemia is regenerative as indicated by reticulocytosis, along with a few nucleated red cells. While the plasma protein concentration is seemingly normal, there is a reasonable probability it is decreased for this patient. The reason is that older dogs tend to have higher protein concentrations and 6.2 is regarded as low normal; the dog may have had a protein between 7 and 8 g/dL before bleeding occurred. The triad of anemia, regeneration, and decreasing protein is clas-sical for blood loss.

The cause of the blood loss is thrombocytopenia. This magnitude of thrombocytopenia is expected to result in both petechial hemorrhages and blood loss that may not be physi-cally visible.

Examination of the data from the bone marrow perspec-tive indicates that the marrow is producing erythrocytes (regeneration) and neutrophils appropriately. The thrombo-cytopenia present is therefore a selective, specifi c cytopenia. When present at this magnitude, typically less than 20 × 10 3 / μ L, immune - mediated thrombocytopenia is by far the most likely cause or diagnosis. Furthermore, the increased mean platelet volume (MPV) corroborated by macroplatelets on the blood fi lm suggests accelerated marrow thrombopoiesis. This is the expected response to a consumptive thrombocytopenia.

Summary

The pattern present is characteristic of immune - mediated thrombocytopenia, with bone marrow response to hemorrhage.

Case 12

Hematology Reference Interval

Plasma protein g/dL 6.2 6.0 – 8.0

PCV (%) 24 40 – 55

Hgb (g/dL) 8.4 12.0 – 18.0

RBC ( × 10 6 / μ L) 3.34 5.5 – 8.5

MCV (fL) 72 62 – 73

MCHC (g/dL) 35 33 – 36

Reticulocytes ( × 10 3 / μ L) 149 < 60

NCC ( × 10 3 / μ L) 11.5 4.5 – 15.0

Bands ( × 10 3 / μ L) 0.1 0 – 0.2

Neutrophils ( × 10 3 / μ L) 9.0 2.6 – 11.0

Lymphocytes ( × 10 3 / μ L) 1.4 1.0 – 4.8

Monocytes ( × 10 3 / μ L) 0.7 0.2 – 1.0

Eosinophils ( × 10 3 / μ L) 0.1 0.1 – 1.2

NRBC ( × 10 3 / μ L) 0.2 0

Platelets ( × 10 3 / μ L) 7 200 – 500

MPV (fL) 22 7.5 – 14.6

Hemopathology noted on blood fi lm:

• Moderate polychromasia

• No platelet clumps found

• Few macroplatelets on scanning

Signalment 11 - year - old female spayed border terrier dog History Owner noticed intermittent nose bleeds for a few days Physical examination Few petechial hemorrhages noted on mucous membranes, otherwise normal

Page 23: VI Clinical Case Presentations - Wiley · Clinical Case Presentations ... Case 17: Dog with vitamin K antagonist (rodenticide) toxicity, 643 ... PT Prothrombin time

Case 13

637

CA

SES

Interpretive discussion

There is marked leukocytosis with the predominant abnor-mality being marked lymphocytosis. Further, the morphol-ogy indicates the presence of large lymphocyte forms. The magnitude of lymphocytosis is clearly interpreted as lym-phocytic leukemia, with morphologic features of a blastic form that some would term “ acute. ” There is a mild mature neutrophilia and monocytosis that is diffi cult to interpret. These could be related to steroid release, with neoplastic lymphocytosis masking the expected steroid - induced lym-phopenia. Alternatively, there may be a compensated infl ammatory stimulus. There is a disproportionate number of NRBC that may be related to either marrow and/or splenic injury associated with lymphoma/leukemia.

The erythrocyte values indicate mild anemia. The anemia is poorly regenerative as indicated by the reticulocyte con-centration. There is mild thrombocytopenia. The presence of two cell lines that potentially have decreased production along with evidence of leukemia suggests that marrow may be involved.

Summary

The pattern present is characteristic of lymphocytic leuke-mia. Cytometric analysis is recommended if treatment is considered.

Signalment Nine - year - old male castrated beagle dog History Owner complaint of lethargy Physical examination Mildly enlarged peripheral lymph nodes, somewhat thin

Hematology Reference Interval

Plasma protein g/dL 7.3 6.0 – 8.0

PCV (%) 32 40 – 55

Hgb (g/dL) 11.5 13.0 – 20.0

RBC ( × 10 6 / μ L) 4.80 5.5 – 8.5

MCV (fL) 69 62 – 73

MCHC (g/dL) 35 33 – 36

Reticulocytes ( × 10 3 / μ L) 49 0 – 100

NCC ( × 10 3 / μ L) 83.7 4.5 – 15.0

Bands ( × 10 3 / μ L) 0 0 – 0.2

Neutrophils ( × 10 3 / μ L) 16.7 2.6 – 11.0

Lymphocytes ( × 10 3 / μ L) 64.4 1.0 – 4.8

Monocytes ( × 10 3 / μ L) 1.6 0.2 – 1.0

Eosinophils ( × 10 3 / μ L) 0.1 0.1 – 1.2

NRBC ( × 10 3 / μ L) 0.8 0

Platelets ( × 10 3 / μ L) 139 200 – 500

MPV (fL) 11.4 7.5 – 14.6

Hemopathology noted on blood fi lm:

• Many of the lymphoid cells are large with fi ne chromatin

and classifi ed as prolymphocytes and lymphoblasts

• No platelet clumps found

Case 13

Page 24: VI Clinical Case Presentations - Wiley · Clinical Case Presentations ... Case 17: Dog with vitamin K antagonist (rodenticide) toxicity, 643 ... PT Prothrombin time

SECTION VI Clinical Case Presentations

638

CA

SES

Case 14

Hematology Reference Interval

Packed cell volume (%) 33 37 – 55

Hemoglobin (g/dL) 11.1 12 – 18

RBC (10 6 / μ L) 5.77 5.5 – 8.5

MCV (fL) 57 60 – 72

MCHC (g/dL) 34 34 – 38

Total nucleated cell count ( × 10 3 / μ L) 5.6 6 – 17

Segmented neutrophils ( × 10 3 / μ L) 4.6 3 – 11.5

Monocytes ( × 10 3 / μ L) 0.7 0.2 – 1.4

Lymphocytes ( × 10 3 / μ L) 0.6 1 – 4.8

Platelets ( × 10 3 / μ L) 190 200 – 500

Plasma protein (g/dL) 11.4 6 – 8

Hemopathology: Marked rouleaux

Signalment 14 - year - old CM mixed - breed dog History Polydipsia and polyuria, bloody diarrhea, weight loss Physical exam Lethargic, bilateral masses in perianal region

Alburmin

1

2

3

4

5

6

7

8

α–1 α–2 β–1 β–2 γ

Serum protein electrophoresis tracing

Biochemical Profi le

Glucose (mg/dL) 93 65 – 122

Blood Urea Nitrogen (mg/dL) 19 7 – 28

Creatinine (mg/dL) 1.2 0.6 – 1.5

Calcium (mg/dL) 14 9.0 – 11.2

Phosphorus (mg/dL) 6.0 2.8 – 6.1

Total Protein (g/dL) 12.0 5.4 – 7.4

Albumin (g/dL) 1.7 2.7 – 4.5

Globulin (g/dL) 10.3 1.9 – 3.4

Total Bilirubin (mg/dL) 0.4 0 – 0.4

Cholesterol 154 130 – 300

Alanine aminotransferase (IU/L) 49 10 – 120

Aspartate aminotransferase (IU/L) 41 16 – 40

Alkaline phosphatase (IU/L) 249 18 – 141

Gamma glutamyl transferase (IU/L) 5 0 – 6

Sodium (mEq/L) 144 145 – 158

Potassium (mEq/L) 4.2 4.1 – 5.5

Chloride (mEq/L) 117 106 – 127

HCO 3 (mEq/L) 13.1 14 – 27

Anion gap 18 8 – 25

Urinalysis (cystocentesis)

Color pale yellow Urine Sediment

Transparency clear WBCs/hpf 0 – 1

Specifi c Gravity 1.012 RBCs/hpf 0 – 1

pH 5.0 Epithelial cells/hpf 0

Glucose neg Casts/lpf 0

Bilirubin neg Crystals few amorphous

Blood 1 + Bacteria 0

Protein 3 +

Ketones neg

Interpretive discussion

Hematology There is a mild normochromic, microcytic anemia that appears nonregenerative (no increased polychromasia is noted on the blood fi lm). A reticulocyte count should be done to confi rm this. In a sick dog, anemia of chronic disease should be considered; however, anemia of chronic disease is not usually microcytic. Given that the dog has bloody diar-rhea, iron defi ciency due to GI blood loss should also be considered as a cause of the microcytosis.

Page 25: VI Clinical Case Presentations - Wiley · Clinical Case Presentations ... Case 17: Dog with vitamin K antagonist (rodenticide) toxicity, 643 ... PT Prothrombin time

Case 14

639

CA

SES

action in the renal tubules, preventing adequate urine con-centration and causing polyuria with polydipsia. The pH is acid, consistent with the metabolic acidosis. The 3 + protein is signifi cant given the urine specifi c gravity and inactive sediment. A urine protein : creatinine ratio could have been performed, but was not in this case. Possibilities for the proteinuria include a prerenal proteinuria due to glomerular overload (paraproteinuria associated with multiple myeloma) or renal proteinuria due to glomerular disease.

Serum protein electrophoresis There is a distinct monoclonal peak in the gamma region, suggestive of a neoplastic monoclonal gammopathy. However, some cases of canine ehrlichiosis have apparent monoclonal gammopathies, so further diagnostics are war-ranted to confi rm lymphoid neoplasia.

Summary

The hypercalcemia was consistent with the clinical suspicion of anal sac adenocarcinoma, but the hyperglobulinemia sug-gested a second pathologic process. Fine needle aspirates of the perianal masses were consistent with anal sac adenocar-cinoma, which was later confi rmed by surgical removal and histopathology. Radiographs of the thoracic cavity showed no evidence of pulmonary metastases, however pathologic fracture of the right 6th rib was identifi ed. Pending tickborne disease titers, a bone marrow aspirate was performed and was diagnostic for plasma cell myeloma (64% of marrow cells were plasma cells). An immunoelectrophoresis identi-fi ed the paraprotein as IgA. Ehrlichial titers were negative. This dog was treated with melphalan and prednisone, and did well at home for about one year.

The lymphopenia suggests a stress/steroid response, although the expected neutrophilia is not observed. The slight thrombocytopenia is not clinically signifi cant. Marked rouleaux seen on the blood fi lm is related to the markedly increased plasma protein concentration (discussed further below).

Biochemical profi le The increased total protein concentration is due to marked hyperglobulinemia. This degree of hyperglobulinemia is usually caused by lymphoid neoplasia (such as multiple myeloma), but can also be seen with ehrlichiosis in dogs. A serum protein electrophoresis is indicated to evaluate for monoclonal versus polyclonal gammopathy (discussed later), and evaluation for tick - borne diseases should be pursued. The albumin concentration is moderately decreased, which may be from decreased production in response to the hyperglobulinemia. However, there is also evidence for urinary protein loss that may be contributing to the hypoalbuminemia.

The other signifi cant abnormality present is hypercalce-mia. Since this dog has palpable perianal masses and adeno-carcinoma of the anal sac is a common cause of paraneoplastic hypercalcemia in dogs, aspiration cytology or biopsy of those masses should be performed. Hypercalcemia may also accompany lymphoid neoplasia, which is another possible differential in this dog based on the hyperglobulinemia. Ionized calcium could be measured to confi rm the hypercal-cemia, but was not done in this case.

The slight increase in ALP suggests induction due to cho-lestasis or steroids. Decreased bicarbonate concentration with a normal anion gap is consistent with a secretional metabolic acidosis, and can be explained by the diarrhea (GI loss of bicarbonate).

Urinalysis The urine is in the isosthenuric range, which can be explained by the hypercalcemia. Hypercalcemia interferes with ADH

Page 26: VI Clinical Case Presentations - Wiley · Clinical Case Presentations ... Case 17: Dog with vitamin K antagonist (rodenticide) toxicity, 643 ... PT Prothrombin time

SECTION VI Clinical Case Presentations

640

CA

SES

Hematology Reference Interval

PCV (%) 23 37 – 55

Hgb (g/dL) 8.5 12 – 18

RBC ( × 10 6 / μ L) 3.27 5.5 – 8.5

MCV (fL) 71 60 – 72

MCHC(g/dL) 37 34 – 38

Retics ( × 10 3 / μ L) 130.8 < 60

NCC ( × 10 3 / μ L) 45.4 6 – 17

Segs ( × 10 3 / μ L) 41.8 3 – 11.5

Bands ( × 10 3 / μ L) 0.5 0 – 0.3

Monos ( × 10 3 / μ L) 3.2 0.1 – 1.3

Lymphs ( × 10 3 / μ L) 0.0 1 – 4.8

Eos ( × 10 3 / μ L) 0.0 0.1 – 1.2

Platelets

( × 10 3 / μ L)

25 200 – 500

TP (P) (g/dL) 4.6 6 – 8

Hemopathology: Increased polychromasia and giant platelets

Case 15 Signalment: 8 - year - old spayed female golden retriever History: Presented for lethargy, anorexia, a mass over the humeroradial joint, and prolonged bleeding at a biopsy site Physical examination: Fever, icterus, and an enlarged liver and spleen

Biochemical Profi le Reference Interval

Gluc (mg/dL) 70 65 – 122

BUN (mg/dL) 11 7 – 28

Ca (mg/dL) 8.2 9.0 – 11.2

Phos (mg/dL) 4.0 2.8 – 6.1

TP (g/dL) 5.0 5.4 – 7.4

Alb (g/dL) 2.2 2.7 – 4.5

Glob (g/dL) 2.8 1.9 – 3.4

T. Bili (mg/dL) 7.6 0 – 0.4

Chol (mg/dL) 329 130 – 370

ALT (IU/L) 58 10 – 120

ALP (IU/L) 775 35 – 280

Na (mEq/L) 144 145 – 158

K (mEq/L) 4.0 4.1 – 5.5

CL (mEq/L) 109 106 – 127

TCO 2 (mEq/L) 16.6 14 – 27

An. gap (mEq/L) 22 8 – 25

Urinalysis

Color Yellow Urine Sediment

Transparency Clear WBCs/hpf 0 – 2

Sp. Gr. 1.012 RBCs/hpf 0 – 1

Protein – Epith cells/hpf 0

Gluc – Casts/lpf 0

Bilirubin 4 + Crystals 0

Blood Trace Bacteria 0

pH 5.5 Other 1 + fat

Coagulation Data Reference Interval

Activated clotting time (seconds) > 300 72 – 86

PT (seconds) 14.5 6.4 – 7.4

aPTT (seconds) 32.3 9 – 11

FDPs ( μ g/mL) > 80 < 10

Page 27: VI Clinical Case Presentations - Wiley · Clinical Case Presentations ... Case 17: Dog with vitamin K antagonist (rodenticide) toxicity, 643 ... PT Prothrombin time

Case 15

641

CA

SES

Interpretative discussion

Hematology There is anemia that is regenerative as evidenced by the signifi cant reticulocytosis and polychromasia. Anemia in combination with low total protein suggests that the cause is blood loss. There is a mixed infl ammatory and stress (steroid) leukogram evidenced by the neutrophilia with band neutrophils, monocytosis, and absence of lymphocytes and eosinophils. The thrombocytopenia in conjunction with the large platelets suggests increased consumption and pro-duction of platelets.

Biochemical profi le Hypocalcemia corrects into the normal range, and thus is due to hypoalbuminemia. Hypoproteinemia is due to blood loss. The hyperbilirubinemia may result from increased erythrocyte destruction or cholestasis. Increased alkaline phosphatase activity supports cholestasis. Very mild hypo-natremia and hyperkalemia are probably insignifi cant in this case.

Urinalysis The urine specifi c gravity is isosthenuric, but the urea nitro-gen is normal, so the specifi c gravity may not be signifi cant.

Water deprivation and ensuing specifi c gravity would deter-mine renal function reserve. Hyperbilirubinuria is a conse-quence of hyperbilirubinemia.

Coagulation data Decreased platelets, prolonged ACT, PT, aPTT, and increased FDPs support disseminated intravascular coagulation. Eryth-rocytes may be destroyed during disseminated intravascular coagulation, thus contributing to elevated total bilirubin.

Summary

The mass was diagnosed as malignant histiocytosis, with nodules in the liver, spleen, mediastinum, and peripheral lymph nodes at necropsy. A likely scenario is that extensive tumor mass developed necrosis and/or infl ammation that triggered hypercoagulability leading to disseminated intra-vascular coagulation. Involvement of liver likely explains the hypoalbuminemia and other liver changes.

Page 28: VI Clinical Case Presentations - Wiley · Clinical Case Presentations ... Case 17: Dog with vitamin K antagonist (rodenticide) toxicity, 643 ... PT Prothrombin time

SECTION VI Clinical Case Presentations

642

CA

SES

Interpretative discussion

Hematology The anemia is regenerative as the reticulocyte count is increased and there is polychromasia and anisocytosis on the blood fi lm. The cause of anemia is not determined, but is likely due to hemolysis or blood loss since it is regenerative. The clinical fi ndings of hemorrhage suggest that blood loss is the cause. Thrombocytosis is common in iron defi ciency anemia. Microcytosis is evident in chronic iron defi ciency, and may contribute to anisocytosis. Size of erythrocytes is not known, since the MCV is not provided. Serum iron and iron binding capacity would be useful in determination of the cause of anemia. The neutrophilia, left shift, and mono-cytosis indicate an infl ammatory leukogram. Lymphopenia is indicative of a concurrent stress/steroid mediated response.

Coagulation data The coagulation profi le suggests a defi ciency of one or mul-tiple coagulation factors in the intrinsic pathway. Platelet concentration is increased in number, and no large forms are seen in peripheral blood. Bleeding time is normal, and in the face of adequate platelet concentration, indicates that platelet function is normal. The most common cause of a severe coagulopathy with normal platelet concentration, normal hepatic enzyme activity, and a prolongation of the aPTT with normal PT is factor 8 defi ciency. The occurrence is less common in females, and to have an affected female requires that the sire also be affected.

Summary

This dog was tested for factor 8 plasma activity and was found to have 21% of normal activity, which is diagnostic for factor 8 defi ciency or hemophilia A. This is compatible with the major abnormalities in the ACT and APTT and the clinical description of bleeding in a young dog.

Hematology Reference Interval

PCV (%) 19 37 – 55

Hgb (g/dL) 6.1 12 – 18

Retics ( × 10 3 / μ L) 188 < 60

NCC ( × 10 3 / μ L) 35.4 6 – 17

Segs ( × 10 3 / μ L) 29.7 3 – 11.5

Bands ( × 10 3 / μ L) 2.5 0 – 0.3

Monos ( × 10 3 / μ L) 3.2 0.1 – 1.3

Lymphs ( × 10 3 / μ L) 0.0 1 – 4.8

Eos ( × 10 3 / μ L) 0.0 0.1 – 1.2

Platelets ( × 10 3 / μ L) 915 200 – 500

TP (P) (g/dL) 6.5 6 – 8

Hemopathology: Moderate polychromasia and anisocytosis is present.

Case 16 Signalment: 5-month - old female dog History: The puppy bleeds excessively when it loses teeth Physical examination: The mucous membranes were pale. There is moderate bleeding evident at the site of a recent tooth loss.

Coagulation Data Reference Interval

Activated clotting time > 180 72 – 86

PT (seconds) 6.8 6.4 – 7.4

aPTT (seconds) > 120 9 – 11

Fibrinogen (mg/dL) 200 100 – 400

Bleeding Time (minutes) 3 1 – 5

Page 29: VI Clinical Case Presentations - Wiley · Clinical Case Presentations ... Case 17: Dog with vitamin K antagonist (rodenticide) toxicity, 643 ... PT Prothrombin time

Case 17

643

CA

SES

Case 17

Hematology Reference Interval

PCV (%) 25 37 – 55

Hgb (g/dL) 8.4 12 – 18

RBC ( × 10 6 / μ L) 4.03 5.5 – 8.5

MCV (fL) 62 60 – 72

MCHC (g/dL) 34 34 – 38

Retics ( × 10 3 / μ L) 44 < 60

NCC ( × 10 3 / μ L) 14.4 6 – 17

Segs ( × 10 3 / μ L) 12.2 3 – 11.5

Monos ( × 10 3 / μ L) 1.6 0.1 – 1.3

Lymphs ( × 10 3 / μ L) 0.6 1 – 4.8

Platelets ( × 10 3 / μ L) 315 200 – 500

TP (P) (g/dL) 4.6 6 – 8

Hemopathology: 1 + leptocytosis and anisocytosis.

Signalment: 7 - year - old female Walker hound History: The owner noticed a swelling on the right front leg on the day of admission Physical examination: The mucous membranes were pale. There was a subcutaneous swelling in the right ventral thoracic area, with some dried blood on all four legs.

Interpretive discussion

Hematology The anemia is nonregenerative as the erythrocyte indices are normal and the reticulocyte count is normal. The plasma and serum protein are low, with equal defi ciency of globulin and albumin, suggesting blood loss as the cause of anemia. The anemia is likely too acute for there to be a regenerative response. Aspiration of the subcutaneous mass confi rmed the presence of blood. Mild neutrophilia, monocytosis, and lymphopenia are indicative of a stress leukogram.

Biochemical profi le The protein changes discussed above indicate subacute blood loss, with fl uid shifting and dilution of plasma protein result-ing in anemia and hypoproteinemia. The mild decrease in sodium and potassium are insignifi cant.

Coagulation data The coagulation data indicates either a defi ciency of multiple coagulation factors, or a single factor defi ciency of the common pathway. Platelets are normal in number, and, no large forms are seen in peripheral blood. Bleeding time is normal and in the face of normal platelet concentration, indicates that platelet function is also normal. The most common cause of a severe coagulopathy with normal plate-lets and normal hepatic enzyme activities is vitamin K antagonism.

Summary

This dog was exposed to diphacinone, a rodentocide that is a vitamin K antagonist; coagulation times returned to normal following administration of vitamin K.

Biochemical Profi le Reference Interval

Gluc (mg/dL) 88 65 – 122

BUN (mg/dL) 17 7 – 28

Creat (mg/dL) 1.1 0.9 – 1.7

Ca (mg/dL) 10.2 9.0 – 11.2

Phos (mg/dL) 3.5 2.8 – 6.1

TP (g/dL) 4.1 5.4 – 7.4

Alb (g/dL) 2.3 2.7 – 4.5

Glob (g/dL) 1.8 1.9 – 3.4

T. Bili (mg/dL) 0.3 0 – 0.4

Chol (mg/dL) 188 130 – 370

ALT (IU/L) 35 10 – 120

ALP (IU/L) 40 35 – 280

Na (mEq/L) 144 145 – 158

K (mEq/L) 4.0 4.1 – 5.5

CL (mEq/L) 107 106 – 127

TCO 2 (mEq/L) 18 14 – 27

Coagulation Data Reference Interval

Activated clotting time (sec) > 180 72 – 86

PT (seconds) > 180 6.4 – 7.4

aPTT (seconds) > 180 9 – 11

Fibrinogen (mg/dL) 300 100 – 400

Bleeding Time (minutes) 4 1 – 5

Page 30: VI Clinical Case Presentations - Wiley · Clinical Case Presentations ... Case 17: Dog with vitamin K antagonist (rodenticide) toxicity, 643 ... PT Prothrombin time

SECTION VI Clinical Case Presentations

644

CA

SES

Case 18 Signalment: 2 - month - old female horse History: Off feed Physical examination: Depressed, evidence of diarrhea

Hematology Reference Interval

PCV (%) 14 32 – 52

Hgb (g/dL) 6.5 11 – 19

NCC ( × 10 3 / μ L) 6.5 5.5 – 12.5

Segs ( × 10 3 / μ L) 4.7 2.7 – 6.7

Monos ( × 10 3 / μ L) 0.1 0 – 0.8

Lymphs ( × 10 3 / μ L) 1.6 1.5 – 5.5

NRBCs ( × 10 3 / μ L) 0.1 0

Platelets ( × 10 3 / μ L) 14 100 – 600

TP (P) (g/dL) 6.3 6 – 8

Hemopathology: mod toxic neutrophils, few reactive lymphs, mod

Howell - Jolly bodies, few echinocytes, marked anisocytosis.

Biochemical Profi le Reference Interval

Gluc (mg/dL) 91 70 – 110

BUN (mg/dL) 40 14 – 27

Creat (mg/dL) 2.1 1.1 – 2.0

Ca (mg/dL) 9.7 11.0 – 13.7

Phos (mg/dL) 6.3 1.9 – 4.1

TP (g/dL) 4.6 5.8 – 7.6

Alb (g/dL) 2.2 2.7 – 3.7

Glob (g/dL) 2.4 2.6 – 4.6

T. Bili (mg/dL) 3.2 0.6 – 2.1

AST (IU/L) 280 185 – 300

GGT (IU/L) 28 7 – 17

SDH (IU/L) 27 0 – 9

CK (IU/L) 169 130 – 470

Na (mEq/L) 120 133 – 145

K (mEq/L) 3.8 2.2 – 4.6

CL (mEq/L) 84 100 – 111

TCO 2 (mEq/L) 11.0 24 – 34

An. gap 28.8 5 – 15

Calc. Osmolality (mOsm/kg) 250 280 – 310

Amylase (IU/L) 34 0 – 87

Lipase (IU/L) 534 ND *

Grossly lipemic serum

* ND — Not determined for foals

Blood Gas Data (arterial) Reference Interval

pH 7.282 7.38 – 7.46

pCO 2 (mmHg) 20.6 35 – 47

pO 2 (mmHg) 60.9 67 – 96

HCO 3 (mEq/L) 9.3 22 – 30

Coagulation Profi le Reference Interval

PT (seconds) 14.6 9.5 – 11.5

aPTT (seconds) 39.8 24 – 45

Fibrinogen (mg/dL) 500 100 – 400

FDPs ( μ g/mL) > 10 & < 40 ND *

* ND — Not determined for foals

Abdominal Fluid Analysis

Color Red

Clarity Opaque

NCC (/ μ L) 16,000

TP (g/dL) 5.7

PCV 13%

Comments: Erythrophagia and platelets noted in fi lm.

Interpretive discussion

Hematology There is a marked anemia. While it is not unusual for neo-natal animals to have a “ congenital anemia ” due to iron defi ciency, the PCV is much lower than is typically encoun-tered by this physiological change. The presence of anisocy-tosis leads one to suspect that there may be a regenerative response, for which evaluation of the MCV and RBC histo-gram would be useful. The presence of nucleated erythro-cytes in the peripheral blood is uncommon in horses, but occasionally seen in foals with profound regenerative responses, or with damage to the bone marrow endothe-lium, as might occur with sepsis. Combined decreases in PCV and serum proteins may indicate hemorrhage. There is a marked thrombocytopenia, which may be due to decreased production or increased consumption; thrombocytopenia is severe enough to be resulting in blood loss. Refer to the discussion of the coagulation profi le for more on this matter.

Page 31: VI Clinical Case Presentations - Wiley · Clinical Case Presentations ... Case 17: Dog with vitamin K antagonist (rodenticide) toxicity, 643 ... PT Prothrombin time

Case 18

645

CA

SES

ance associated with endotoxemia. One should consider other potential causes of hyperlipidemia such as pancreatitis. In this case, serum amylase activity is normal, but lipase activity may be increased. There is evidence of recent hem-orrhage into the abdominal cavity, which could be related to acute pancreatitis, but is more often due, in diseased foals, to a bleeding gastric ulcer. The low calculated osmolality would be expected, given the hyponatremia and hypochloremia.

Blood gas data There is an increased anion gap metabolic acidosis with respiratory compensation. This is consistent with a secretory diarrhea, complicated by hypovolemia and/or sepsis. If there were gastrointestinal stasis, one might expect an alkalosis. If there were a ruptured urinary bladder, one might expect a metabolic acidosis with hyperkalemia. The increased anion gap may result from sepsis, with hypovolemia and lactic acidosis due to reduced tissue perfusion and/or the meta-bolic effects of endotoxemia. The decreased oxygen tension may indicate respiratory compromise as well.

Coagulation data The prothrombin time is prolonged slightly, the activated partial thromboplastin time is normal, and the FDP concen-tration is in an intermediate range. These fi ndings may indi-cate disseminated intravascular coagulation, particularly in light of the severe thrombocytopenia, wherein Factor VII levels are becoming depleted, thereby prolonging the PT, but other coagulation factor concentrations are adequate to maintain a normal APTT. The concurrent observation of thrombocytopenia and fi ndings consistent with blood loss anemia support a diagnosis of DIC with pathologic hemorrhage.

Summary

Enterotoxigenic E. coli diarrhea, pancreatitis, hepatitis, and DIC were fi ndings confi rmed at necropsy. On necropsy, the pancreas was 5 to 6 times normal size and the liver was swollen. Histopathology showed necrosis and infl ammation of the pancreas, diffuse mesenteric steatitis, fat necrosis and fat saponifi cation, infl ammation of the liver with thrombi in central veins and associated focal ischemic necro-sis. Infl ammatory disease is often not refl ected in equine leukograms.

Biochemical profi le The BUN, serum creatinine, and phosphorus concentrations are increased, but the nature of this azotemia cannot be defi nitively differentiated without a urinalysis.

There is hypocalcemia and hyperphosphatemia. This com-bination of mineral abnormalities may be seen in nutritional secondary hyperparathyroidism due to excessive dietary phosphorus. However, higher serum phosphorus concentra-tions are commonly observed in growing animals, and hypo-calcemia may also be due to uptake by widespread damaged tissues, decreased intake with anorexia, or to an apparent decrease due to hypoalbuminemia.

Serum total protein concentration is decreased, including both hypoalbuminemia and hypoglobulinemia. Low serum albumin may be hepatocellular dysfunction or cachexia and decreased albumin synthesis. Alternatively, there may be pathologic albumin loss due to gastrointestinal or renal disease. Low serum globulin in a 2 - month - old foal is not due to failure of passive transfer, but may be due to decreased production, malnutrition, or pathologic loss. Loss of all proteins would be expected with hemorrhage, which could also account for the profound anemia. This is the most likely cause.

The serum total bilirubin concentration is increased, with only a mild increase in serum GGT activity. This may refl ect hyperbilirubinemia of fasting in an equine anorexic patient. However, SDH activity is increased, indicating hepatocellular damage.

Serum sodium and chloride concentrations are decreased. This is commonly observed in young animals with an enterotoxigenic or secretory diarrhea. This may also be due to gastrointestinal stasis, a third space accumulation in the abdominal cavity, as well as to decreased intake. One would typically expect a hyperkalemia to occur in secretory diar-rhea, owing to acidosis - induced intercompartmental exchange. Hypokalemia may be expected in third space syn-dromes, owing to potassium loss and renal decompensation. It is possible to observe normokalemia with concomitant potassium loss and metabolic acidosis, wherein redistribu-tion of potassium from the intracellular to the extracellular fl uid compartment obscures the whole body potassium defi cit. There is evidence in support of abdominal hemor-rhage and third spacing due either to acute pancreatitis or a gastric ulcer (see below). Marked lipemia is often seen in ponies with starvation and metabolic disease, but is unusual in horses. In other species, hyperlipidemia may occur due to impaired triglyceride clear-

Page 32: VI Clinical Case Presentations - Wiley · Clinical Case Presentations ... Case 17: Dog with vitamin K antagonist (rodenticide) toxicity, 643 ... PT Prothrombin time

SECTION VI Clinical Case Presentations

646

CA

SES

Case 19 Signalment: 3 - year - old spayed female cocker spaniel History: Left in owner ’ s car in shopping mall parking lot for approximately 3 hours on a hot summer afternoon Physical examination: Depressed and mildly dehydrated

Hematology Reference Interval

PCV (%) 58 37 – 55

NCC ( × 10 3 / μ L) 16.0 6 – 17

Segs ( × 10 3 / μ L) 13.4 3 – 11.5

Monos ( × 10 3 / μ L) 1.6 0.1 – 1.3

Lymphs ( × 10 3 / μ L) 1.0 1 – 4.8

Platelets ( × 10 3 / μ L) Adequate 200 – 500

Biochemical Profi le Reference Interval

Gluc (mg/dL) 142 65 – 122

BUN (mg/dL) 62 7 – 28

Creat (mg/dL) 3.0 0.9 – 1.7

Ca (mg/dL) 8.4 9.0 – 11.2

Phos (mg/dL) 4.9 2.8 – 6.1

TP (g/dL) 9.4 5.4 – 7.4

Alb (g/dL) 5.4 2.7 – 4.5

Glob (g/dL) 4.0 1.9 – 3.4

T. Bili (mg/dL) 0.4 0 – 0.4

Chol (mg/dL) 160 130 – 370

ALT (IU/L) 178 10 – 120

ALP (IU/L) 60 35 – 280

Na (mEq/L) 164 145 – 158

K (mEq/L) 5.4 4.1 – 5.5

CL (mEq/L) 124 106 – 127

TCO 2 (mEq/L) 14 14 – 27

An. gap (mEq/L) 31.4 8 – 25

Meas. Osmolality (mOsm/kg) 358 290 – 310

Calc. Osmolality (mOsm/kg) 344 290 – 310

Osmol gap (mOsm/kg) 14 0 – 10

Blood Gas Data (arterial) Reference Interval

pH 7.09 7.33 – 7.45

PCO 2 (mmHg) 46 24 – 39

HCO 3 (mEq/L) 13 14 – 24

Urinalysis

Color Dk yellow Urine Sediment

WBCs/hpf 2 – 3

Transpareney Cloudy RBCs/hpf 4 – 5

Sp. Gr. 1.011 Epith cells/hpf 2 – 3

Protein 1 + Casts/lpf 2 – 3 fi ne granular

Gluc Neg

Bilirubin Neg Crystals 2 + Ca oxalate

Blood Neg Bacteria 0

pH 5.5

Page 33: VI Clinical Case Presentations - Wiley · Clinical Case Presentations ... Case 17: Dog with vitamin K antagonist (rodenticide) toxicity, 643 ... PT Prothrombin time

Case 19

647

CA

SES

of unmeasured osmotically active solutes in the blood. The anion gap is likewise increased, and given the dehydration and probable tissue hypoperfusion, some degree of lactic acidosis is likely.

Blood gas data There is a combined metabolic (decreased bicarbonate) and respiratory (increased pCO 2 ) acidosis. The metabolic acidosis results from lactic acidosis due to tissue hypoperfusion. The respiratory acidosis suggests compromised pulmonary function.

Urinalysis The presence of 1 + proteinuria with a specifi c gravity of 1.011 indicates signifi cant urinary protein loss. The fi ne granular casts indicate tubular epithelial damage. The isos-thenuric specifi c gravity in the face of dehydration and azo-temia, yet in the absence of electrolyte depletion, suggests renal disease as well. This is likely a case of acute renal failure secondary to heat stress. The presence of calcium oxalate crystals may have no importance, or may represent one potential route of calcium loss due to renal tubular damage associated with the hypocalcemia.

Summary

Heat stress, hypertonic dehydration, and acute renal failure. If one did not have the history given, or doubted its veracity, laboratory fi ndings like these would strongly suggest anti-freeze intoxication. One could analyze serum for ethylene glycol concentration to defi nitively rule this possibility in or out.

Interpretive discussion

Hematology Hemoconcentration is indicated by the increased PCV and physical signs of dehydration. Mild neutrophilia, monocytosis, and borderline lymphopenia is interpreted as a stress leukogram.

Biochemical profi le There is a mild hyperglycemia, which may be due to a cat-echolaminergic or steroid stress response.

The BUN and serum creatinine concentrations are increased. See discussion of urinalysis below to explain whether the azotemia is likely prerenal, renal, or postrenal.

Hyperalbuminemia with hyperproteinemia indicates dehydration. In this case, hyperglobulememia is also likely caused by dehydration.

There is a mild hypocalcemia in the face of hyperalbumin-emia due to dehydration. Thus, serum calcium concentra-tion is truly decreased. This is often seen in heat - stressed animals, subsequent to widespread tissue damage and precipitation of calcium salts in ischemic areas.

The small increase in serum ALT activity may not be signifi cant, or may refl ect some hepatocellular damage.

The hypernatremia, in concert with other signs of dehy-dration, indicates a hypertonic dehydration. This is com-monly seen in heat - stressed dogs owing to increased insensible losses of water, in excess of solute, due to hyper-ventilatory evaporation.

The measured and calculated osmolality values are increased, consistent with hypertonic dehydration. However, the osmol gap is also increased, indicating the accumulation

Page 34: VI Clinical Case Presentations - Wiley · Clinical Case Presentations ... Case 17: Dog with vitamin K antagonist (rodenticide) toxicity, 643 ... PT Prothrombin time

SECTION VI Clinical Case Presentations

648

CA

SES

Case 20 Signalment: 4 - year - old intact male dog History: Experiencing intermittent periods of weakness and lameness Physical examination: Mild dehydration, foul smelling breath, teeth covered with tartar

Hematology Reference Interval

PCV (%) 11 37 – 55

Hgb (g/dL) 4.0 12 – 18

RBC ( × 10 6 / μ L) 1.64 5.5 – 8.5

MCV (fL) 67 60 – 72

MCHC (g/dL) 36 34 – 38

Retics ( × 10 3 / μ L) 13.1 < 60

NCC ( × 10 3 / μ L ) 8.7 6 – 17

Segs ( × 10 3 / μ L) 7.7 3 – 11.5

Bands ( × 10 3 / μ L) 0.1 0 – 0.3

Monos ( × 10 3 / μ L ) 0.3 0.1 – 1.3

Lymphs ( × 10 3 / μ L ) 0.4 1 – 4.8

Eos ( × 10 3 / μ L ) 0.2 0.1 – 1.2

Platelets ( × 10 3 / μ L) 370 200 – 500

TP (P) (g/dL) 6.8 6 – 8

Hemopathology: Slight anisocytosis and slight polychromasia.

Biochemical Profi le

Reference Interval

Gluc (mg/dL) 91 65 – 122

BUN (mg/dL) 183 (65.3) 7 – 28 (2.5 – 10.0 mmol/L)

Creat (mg/dL) 8.1 (716) 0.9 – 1.7 (79 – 150 μ moJ/L)

Ca (mg/dL) 8.2 (2.05) 9.0 – 11.2 (2.25 – 2.8 mmol/L)

Phos (mg/dL) 17.2 (5.5) 2.8 – 6.1 (0.9 – 2.9 mmol/L)

TP (g/dL) 5.8 5.4 – 7.4

Alb (g/dL) 3.2 2.7 – 4.5

Glob (g/dL) 2.6 1.9 – 3.4

T. Bili (mg/dL) 0.4 0 – 0.4

Chol (mg/dL) 180 130 – 370

ALT (IU/L) 19 10 – 120

AST (IU/L) 17 16 – 40

ALP (IU/L) 40 35 – 280

Na (mEq/L) 146 145 – 158

K (mEq/L) 5.0 4.1 – 5.5

Cl (mEq/L) 115 106 – 127

TCO 2 (mEq/L) 16 14 – 27

An. gap (mEq/L) 20 8 – 25

Urinalysis (catheterized)

Color Yellow Urine Sediment

Transparency Clear WBCs/hpf 3 – 5

Sp. Gr. 1.008 RBCs/hpf 2 – 3

Protein Trace Epith cells/hpf 0

Gluc Negative Casts/lpf 0

Bilirubin Negative Crystals 0

Blood Negative Bacteria 0

pH 5.0

Page 35: VI Clinical Case Presentations - Wiley · Clinical Case Presentations ... Case 17: Dog with vitamin K antagonist (rodenticide) toxicity, 643 ... PT Prothrombin time

Case 20

649

CA

SES

Interpretive discussion

Hematology The nonregenerative anemia is secondary to chronic renal disease. Decreased erythropoietin production by the kidneys is a major factor leading to anemia in animals with chronic renal disease. The severity of anemia is unusual for chronic renal disease. Such anemias are typically of mild to moder-ate severity. Other causes of nonregenerative anemia should also be considered in this case.

The cause of the lymphopenia is increased blood steroid concentration associated with stress. The leukocyte response in not a typical steroid - mediated response in that a mature neutrophilia typically accompanies lymphopenia. It is likely that the animal ’ s resting neutrophil concentration was low normal and it has approximately doubled due to the steroid infl uence.

Biochemical profi le The triad of BUN, creatinine, and phosphorus concentra-tions is markedly increased indicating decreased glomerular fi ltration. These products are passively fi ltered by the glom-erulus, and any cause of decreased glomerular fi ltration will result in retention of these analytes in the blood. In light of the urine specifi c gravity in the isosthenuric range, primary renal azotemia is interpreted.

At least two mechanisms have played a role in causing the hypocalcemia. The phosphorus concentration is mark-edly increased, and the Ca × P product is 141. When this product exceeds 70, calcium and phosphorus precipitate in soft tissues, decreasing the serum calcium concentration. In

addition, chronic renal disease may result in decreased acti-vation of vitamin D by the kidneys (i.e., conversion of 25 - hydroxycholecalciferol to 1,25 - dihydroxycholecalciferol). Decreased activated vitamin D results in decreased absorp-tion of calcium from the intestinal tract.

Urinalysis A urine specifi c gravity in the isosthenuric range in an azo-temic animal suggests an inability to concentrate urine. Animals with prerenal azotemia due to decreased renal per-fusion (e.g., dehydration, cardiac insuffi ciency, circulatory shock) should be conserving water and concentrating urine. The urine specifi c gravity is a key to properly interpreting the cause of the azotemia in this case.

Urine sediment — Small numbers of leukocytes and eryth-rocytes are normal in urine. These numbers must be inter-preted in light of the urine concentration and the technique used to concentrate the sediment. Leukocyte numbers may be slightly increased in this case, suggesting minimal infl am-mation in the urinary tract.

Summary

These data indicate chronic renal failure. Chronicity is sug-gested by the nonregenerative anemia, which would not be present with acute renal failure. Postmortem diagnosis in this case was chronic interstitial nephritis or end - stage renal disease. No lesions suggesting suppurative infl ammation in the urinary tract were found.

Page 36: VI Clinical Case Presentations - Wiley · Clinical Case Presentations ... Case 17: Dog with vitamin K antagonist (rodenticide) toxicity, 643 ... PT Prothrombin time

SECTION VI Clinical Case Presentations

650

CA

SES

Case 21 Signalment: 9 - year - old intact female dog History: Abscess on rear leg 2 months ago. Intermittent vomiting began 2 days ago. Physical examination: Popliteal and cervical lymph nodes are enlarged

Hematology Reference Interval

PCV(%) 35 37 – 55

Hgb (g/dL) 12.1 12 – 18

RBC ( × 10 6 / μ L) 5.6 5.5 – 8.5

MCV (fL) 62 60 – 72

MCHC (g/dL) 36 34 – 38

Retics ( × 10 3 / μ L) 22.4 < 60

NCC ( × 10 3 / μ L) 13 6 – 17

Segs ( × 10 3 / μ L) 9.4 3 – 11.5

Bands ( × 10 3 / μ L) 0.1 0 – 0.3

Monos ( × 10 3 / μ L) 0.8 0.1 – 1.3

Lymphs ( × 10 3 / μ L) 2.4 1 – 4.8

Eos ( × 10 3 / μ L) 0.3 0.1 – 1.2

Platelets ( × 10 3 / μ L) 250 200 – 500

TP (P) (g/dL) 6.2 6 – 8

Hemopathology: Normal.

Biochemical Profi le

Reference Interval

Gluc (mg/dL) 89 65 – 122

BUN (mg/dL) 114 (40.7) 7 – 28 (2.5 – 10.0 mmol/L)

Creat (mg/dL) 3.2 (283) 0.9 – 1.7 (79 – 150 μ mol/L)

Ca (mg/dL) 8.5 (2.12) 9.0 – 11.2

(2.25 – 2.8 mmo//L)

Phos (mg/dL) 8.8 (2.84) 2.8 – 6.1 (0.9 – 2.9 mmol/L)

TP (g/dL) 5.2 5.4 – 7.4

Alb (g/dL) 1.2 2.7 – 4.5

Glob (g/dL) 4.0 1.9 – 3.4

T. Bili (mg/dL) 0.3 0 – 0.4

Chol (mg/dL) 582 (15.1) 130 – 370 (3.4 – 9.6 mmol/L)

ALT (IU/L) 18 10 – 120

AST (IU/L) 20 16 – 40

ALP (IU/L) 22 35 – 280

Na (mEq/L) 142 145 – 158

K (mEq/L) 4.7 4.1 – 5.5

CL (mEq/L) 120 106 – 127

TCO 2 (mEq/L) 18 14 – 27

An. gap (mEq/L) 9 8 – 25

Amylase (IU/L) 1530 50 – 1250

Lipase (IU/L) 720 30 – 560

Urinalysis (catheterized)

Color Yellow Urine Sediment

Transparency Cloudy WBCs/hpf 0

Sp. Gr. 1.021 RBCs/hpf 0

Protein 4 + Epith cells/hpf 0

Gluc Negative Casts/lpf 2 – 3 granular

Bilirubin Negative Crystals 0

Blood Negative Bacteria 0

pH 6.0

protein/

creatinine ratio 5.4

Page 37: VI Clinical Case Presentations - Wiley · Clinical Case Presentations ... Case 17: Dog with vitamin K antagonist (rodenticide) toxicity, 643 ... PT Prothrombin time

Case 21

651

CA

SES

The cause of mild hyponatremia is not certain in this case. Renal Na loss is a possible cause. If edema were present, it is possible that dilution of extracellular Na in this fl uid (third - spacing) could result in decreased serum Na concen-tration. Edema was, however, not evident in this case, and, even in animals with edema, hyponatremia is not common.

Serum amylase and lipase activities are commonly increased in animals with decreased GFR. Although other causes of increased activities such as pancreatitis could be considered in this case, the clinical presentation and other laboratory data are more compatible with decreased GFR resulting in mildly increased amylase and lipase activities.

Urinalysis Urine concentrating ability is inadequate. If the azotemia in this dog were due to prerenal causes such as dehydration, cardiac insuffi ciency, or circulatory shock, urine specifi c gravity should be > 1.030. The specifi c gravity suggests inad-equate concentrating ability and primary renal azotemia. Post - renal azotemia is ruled out by the demonstration of a patent urethra via catheterization and by the absence of evidence of urine leakage into tissues or the abdomen. Lack of concentrating ability results from loss of nephrons and/or tubular damage. Both of these alterations are probably occurring in this dog. Although the disease is primarily glo-merular, severe damage to glomeruli results in secondary tubular damage and in loss of nephrons.

A 4 + protein in a moderately dilute urine and a urine protein/creatinine ratio (UPC) of 5.4 are evidence of severe proteinuria. In the absence of evidence of hemorrhage or infl ammation (i.e., increased erythrocyte or leukocyte numbers in the urine sediment), a UPC > 1.0 is abnormal in the dog, and a UPC > 5.0 is indicative of glomerular disease. A UPC of > 15 is diagnostic for glomerular disease.

Summary

Renal biopsy revealed amyloidosis. Chronic infection result-ing in chronic antigenic stimulation probably predisposed the dog to this disease. The lymph node enlargement was most likely due to hyperplasia secondary to chronic anti-genic stimulation. The combination of hypoalbuminemia, proteinuria, and hypercholesterolemia suggest imminent onset of nephrotic syndrome.

Interpretive discussion

Hematology A mild nonregenerative anemia (reticulocyte count is in the normal range) is present. This should prompt an evaluation for endocrine disease, renal disease, and chronic infl amma-tory disease as potential causes. In this case, chronic renal disease is probably the underlying cause. There is no evi-dence of infl ammatory disease in the leukogram.

Biochemical profi le The azotemia indicated by increased concentrations of BUN, creatinine, and phosphorus should be classifi ed as renal since urine concentration is not adequate (i.e., < 1.030), sug-gesting a loss of renal concentrating ability.

The hypocalcemia is probably due to two factors. The calcium × phosphorus product is 75. When this product exceeds 70, precipitation of calcium and phosphorus in soft tissues can occur, and decreased serum calcium concentra-tions may result. In addition, activation of vitamin D by the kidney is decreased in chronic renal disease, resulting in decreased absorption of calcium from the small intestine.

The hyperphosphatemia is due to decreased glomeru-lar fi ltration rate (GFR). In this case, glomerular disease has caused decreased GFR and subsequent hyperphosphatemia.

In light of the marked proteinuria, the most likely cause of the hypoproteinemia and hypoalbuminemia is renal protein loss, due to glomerular disease. The hyperglobulin-emia most likely resulted from chronic antigenic stimula-tion. History of a previous abscess and subsequent lymph node enlargement are compatible with such antigenic stim-ulation (i.e., the original infection may not have been com-pletely eliminated, resulting in chronic antigenic stimulation and hyperplasia in lymphoid tissue). Such chronic antigenic stimulation can predispose to some forms of glomerular disease.

Hypercholesterolemia is interpreted as a component of nephrotic syndrome. Nephrotic syndrome, a group of abnor-malities that may be associated with serious glomerular disease, includes hypoalbuminemia, proteinuria, hypercho-lesterolemia and edema. In this case, edema was not observed; however, presence of the other three components is still suggestive of this syndrome. Edema is not likely to occur until the albumin is below 1.0 g/dL. The mechanism causing hypercholesterolemia in this syndrome has not been identifi ed.

Page 38: VI Clinical Case Presentations - Wiley · Clinical Case Presentations ... Case 17: Dog with vitamin K antagonist (rodenticide) toxicity, 643 ... PT Prothrombin time

SECTION VI Clinical Case Presentations

652

CA

SES

Case 22 Signalment: 13 - year - old castrated male cat History: Rear leg paralysis, dyspnea, vomiting Physical examination: Lethargy and dyspnea. Systolic murmur detected.

Hematology Day 1 Reference Interval

PCV (%) 35 24 – 45

Hgb (g/dL) 11.3 8 – 15

RBC ( × 10 6 / μ L ) 8.05 5 – 11

MCV (fL) 44 39 – 50

MCHC (g/dL) 32 33 – 37

NCC ( × 10 3 / μ L) 18.1 5.5 – 19.5

Segs ( × 10 3 / μ L ) 16.3 2.5 – 12.5

Monos ( × 10 3 / μ L) 0.5 0 – 0.8

Lymphs ( × 10 3 / μ L) 0.9 1.5 – 7.0

Basophils ( × 10 3 / μ L) 0.2 Rare

NRBC ( × 10 3 / μ L) 0.2 0

Platelets ( × 10 3 / μ L) Adequate

TP (P) (g/dL) 7.2 6 – 8

Hemopathology: Normal

Biochemical Profi le Day 1 Day 3 Reference Interval

Gluc (mg/dL) 153 (8.4) 360 (19.8) 67 – 124 (3.7 – 6.8 mmol/L)

BUN (mg/dL) 46 (16.4) 137 (48.9) 17 – 32 (6.1 – 11.4 mmol/L)

Creat (mg/dL) 2.9 (256) 9.8 (866) 0.9 – 2.1 (80 – 186 mmol/L)

Ca (mg/dL) 8.4 (2.12) 4.9 (1.22) 8.5 – 11 (2. 12 – 2.75 mmol/L)

Phos (mg/dL) 8.0 (2.6) 16.1 (5.2) 3.3 – 7.8 (1.1 – 2.5 mmol/L)

TP (g/dL) 6.9 5.4 5.9 – 8.1

Alb (g/dL) 2.8 2.4 2.3 – 3.9

Glob (g/dL) 4.1 3.0 2.9 – 4.4

T. Bili (mg/dL) 0.2 0.3 0 – 0.3

Chol (mg/dL) 192 151 60 – 220

ALT (IU/L) 158 294 30 – 100

AST (IU/L) 461 643 14 – 38

ALP (IU/L) 54 25 6 – 106

GGT (IU/L) 0 1 0 – 1

CK (IU/L) 45,313 350,930 60 – 300

Na (mEq/L) 150 139 146 – 160

K (mEq/L) 4.9 6.6 3.7 – 5.4

CL (mEq/L) 119 99 112 – 129

TCO 2 (mEq/L) 19.2 15.9 14 – 23

An. gap (mEq/L) 17 31 10 – 27

Page 39: VI Clinical Case Presentations - Wiley · Clinical Case Presentations ... Case 17: Dog with vitamin K antagonist (rodenticide) toxicity, 643 ... PT Prothrombin time

Case 22

653

CA

SES

Urinalysis (cystocentesis)

Day 1 Day 3 Urine Sediment Day 1 Day 3

Color Dark yellow Light yellow WBCs/hpf 15 – 20 0 – 1

Transparency Hazy Clear RBCs/hpf 35 – 50 5 – 10

Sp. Gr. 1.050 1.010 Epith cells/hpf 0 0

Protein 2 + 1 + Casts/lpf Few granular 0

Gluc 4 + 4 + Crystals 0 0

Ketones 2 + Negative

Bilirubin Negative Negative Bacteria 0 0

Blood 4 + 4 +

pH 5.5 5.0

Fractional excretion Day 1 Reference Interval

Na (%) 7.2 < 1.0

K (%) 165.1 5 – 20

P (%) 68.6 < 7 – 21

Ca (%) 10.5 < 1.0

Coagulation Data Day 1 Day 3 Reference Interval

PT (seconds) 10.0 8.9 7 – 11.5

aPTT (seconds) 8.2 16.5 10 – 18

Endocrine Data Day 1 Day 3 Reference Interval

Total T4 ( μ g/dL) 1.34 1.2 – 4.8

Interpretive discussion

Hematology In light of normal values for other erythrocyte measure-ments, the slightly decreased MCHC is not signifi cant. Mature neutrophilia and lymphopenia are typical of a stress leukogram. The basophils are not signifi cant. Occasionally, nucleated RBCs may be found in the blood of normal animals. In the absence of anemia or other erythrocyte abnormalities, the few nucleated RBCs noted in this cat are not important.

Biochemical profi le The cat is hyperglycemic on Days 1 and 3. This abnormal-ity could be due to severe excitement or stress with result-ing increased epinephrine or corticosteroid levels, respectively. The leukogram is suggestive of stress. The presence of ketonuria on Day 1 suggests that diabetes mel-litus should also be considered. Although this cat ’ s blood glucose concentration on Day 1 is not above the renal threshold, detection of glucosuria on this day suggests that

the cat may have had periods with higher blood glucose concentrations or that this cat has an abnormally low renal threshold for glucose.

The cat has an azotemia which progresses from mild to severe. Since urine specifi c gravity is high on Day 1, the azotemia on that day appears to be prerenal. Urine specifi c gravity is in the isosthenuric range on Day 3 and may be of renal origin; however, the cat had received fl uid therapy, and this, rather than renal failure, likely caused the low urine specifi c gravity on this day.

Hypocalcemia progresses from mild on Day 1 to marked on Day 3. While ethylene glycol toxicosis may result in hypocalcemia and causes severe azotemia, rear leg paralysis and increased CK activity are not associared with ethylene glycol toxicosis. The Ca × P product on Day 1 is 67 on Day 1 and 79 by Day 3. Precipitation of Ca and P in the tissues may, therefore, be occurring on Day 3 and may, in part, explain the decreasing Ca concentration. Massive muscle tissue destruction, as evidenced by increased CK activity, may have resulted in calcium preciptitation in damaged tissues and subsequent hypocalcemia.

Page 40: VI Clinical Case Presentations - Wiley · Clinical Case Presentations ... Case 17: Dog with vitamin K antagonist (rodenticide) toxicity, 643 ... PT Prothrombin time

SECTION VI Clinical Case Presentations

654

CA

SES

concentration of the urine with more dilution of protein on Day 3. Both pyuria and hematuria probably contributed to the proteinuria. Other causes of proteinuria such as glo-merular or tubular disease cannot be eliminated. The dipstick test for blood was equally increased on Days 1 and 3, but the RBC concentration decreased markedly between these days. This suggests that the positive test is due to either hemoglobinuria or myoglobinuria. In light of the apparent muscle injury (increased CK), myoglobinuria is most likely.

Glucosuria is marked on both days. On Day 3, this refl ects a blood glucose concentration which exceeds the renal threshold. The glucosuria is more diffi cult to explain on Day 1, when the blood glucose is below the renal threshold. While it is possible that this cat has a lowered renal thresh-old, it is also possible blood glucose concentrations were fl uctuating on Day 1 with periods above the renal threshold occurring.

Presence of a few granular cast suggests tubular damage. The signifi cance of the positive urine ketone reaction is

considered in the discussion of hyperglycemia. Fractional excretions of Na, K, P, and Ca are increased.

This indicates abnormal reabsorption of these electrolytes and, in this case, is probably due to acute renal damage.

Coagulation data The activated partial thromboplastin time (APTT) is slightly decreased on Day 1 and probably refl ects this cat ’ s hyperco-agulable condition. The mechanism of this change is not known but may be related to this cat ’ s cardiac problem (see Summary). This cat was treated with streptokinase between Days 1 and 3, and this treatment increases APTT and PT, and return of APTT to within the reference interval on Day 3 may have resulted from this treatment; however, the absence of a longer PT on Day 1 as compared to Day 3 makes a sig-nifi cant effect of streptokinase treatment less certain.

Summary

Clinical diagnosis was restrictive cardiomyopathy with aortic thromboemboli (saddle and renal thrombosis and pulmo-nary thromboembolism). Restrictive cardiomyopathy pre-disposes to thrombosis. In this case, the thrombotic disease involved the kidneys and resulted in acute renal failure. In addition, hypoxia occurred in other tissues including the muscles of the rear legs. This resulted in increased serum activities of AST and CK. Necropsy examination was not performed.

Hyperphosphatemia resulted from decreased glomerular fi ltration rate. Maintenance of normal serum P concentra-tion depends on normal glomerular clearance of P.

Total serum protein concentration was normal on Day 1, but decreased by Day 3. Although both albumin and globu-lin concentrations remained within reference intervals, con-centrations of both of these proteins decreased due to fl uid therapy and subsequent expansion of blood volume. In light of normal serum albumin and globulin concentrations, the signifi cance of the hypoproteinemia is borderline.

Increased serum ALT activity suggests mild hepatocyte injury that progressed to moderate.

The combination of increased serum AST and CK activities indicates muscle injury. Since CK has a short half - life (less than four hours), the extremely high CK activity implies active muscle damage. AST is also present in hepatocytes, and hepatic injury is an alternate explanation for the increased serum AST activity, but, in light of the increased serum CK activity, muscle origin is most likely.

Hyponatremia on Day 3 may be due to renal loss (see fractional excretion results). Since hypochloremia is also evident, vomiting could also be a cause of Na loss. Hypo-chloremia on Day 3 may be due to both renal loss and vomiting.

Hyperkalemia on Day 3 may be due to several different causes. Since the cat is in renal failure, kidneys may not be excreting K normally. This cat also had a signifi cant degree of tissue necrosis that could have resulted in release of K from dead or dying cells.

The increased anion gap suggests increased concentrations of anions such as ketones, uremic acids, phosphate, sulfate, or lactate. Ketones are not present in the urine of this cat on Day 3, and a signifi cant ketosis is, therefore, not likely. Since the cat is severely azotemic, concentrations of uremic acids are probably increased. Serum phosphorus concentra-tion confi rms that increased phosphate is contributing to the anion gap. The fi nal diagnosis suggested that this cat had signifi cant tissue damage, and this probably increased serum sulfate concentrations. Hypoxia was also a component of this cat ’ s disease; therefore, lactic acidosis was also occurring.

Urinalysis The implications of the urine specifi c gravities were dis-cussed in the interpretation of this cat ’ s azotemia. The cat has proteinuria and hematuria on both days and pyuria on Day 1. These abnormalities suggest urinary tract infl amma-tion. Cystitis or pyelonephritis are possible causes of this infl ammation. The protein concentration decreased between Days 1 and 3, but this probably refl ects the change in the

Page 41: VI Clinical Case Presentations - Wiley · Clinical Case Presentations ... Case 17: Dog with vitamin K antagonist (rodenticide) toxicity, 643 ... PT Prothrombin time

Case 23

655

CA

SES

Case 23 Signalment: 11 - year - old FS canine History: Weight loss and polyuria Physical examination: Thin, slightly dehydrated

Hematology Reference Interval

PCV (%) 36.0 37 – 55

Hgb (g/dL) 12.5 12 – 18

RBC ( × 10 6 / μ L) 5.38 5.5 – 8.5

MCV (fL) 67.0 60 – 72

MCHC (g/dL) 35.0 34 – 38

NCC ( × 10 3 / μ L) 7.0 6 – 17

Segs ( × 10 3 / μ L) 6.1 3 – 11.5

Bands ( × 10 3 / μ L) 0.1 0 – 0.3

Monos ( × 10 3 / μ L) 0.2 0.1 – 1.3

Lymphs ( × 10 3 / μ L) 0.5 1 – 4.8

Eos ( × 10 3 / μ L) 0.1 0.1 – 1.2

Platelets ( × 10 3 / μ L) 400 200 – 500

TP (P) (g/dL) 8.1 6 – 8

Biochemical Profi le

Reference Interval

Gluc (mg/dL) 112 65 – 122

BUN (mg/dL) 216 (77.1) 7 – 28 ( 2.5 – 10.0 mmol/L )

Creat (mg/dL) 15.6 (1379) 0.9 – 1.7 (79 – 150 μ mol/L)

Ca (mg/dL) 12.1 (3.0) 9.0 – 11.2 (2.25 – 2.8 mmol/L)

Phos (mg/dL) 20.9 (6.75) 2.8 – 6.1 (0.9 – 2.9 mmol/L)

TP (g/dL) 6.9 5.4 – 7.4

Alb (g/dL) 4.0 2.7 – 4.5

Glob (g/dL) 2.9 1.9 – 3.4

T. Bili (mg/dL) 0.4 0 – 0.4

Chol (mg/dL) 335 130 – 370

ALT (IU/L) 73 10 – 120

AST (IU/L) 25 16 – 40

ALP (IU/L) 662 35 – 280

GGT (IU/L) 8 0 – 6

Na (mEq/L) 144 145 – 158

K (mEq/L) 6.2 4.1 – 5.5

CL (mEq/L) 98 106 – 127

TCO 2 (mEq/L) 13.1 14 – 27

An. gap (mEq/L) 39 8 – 25

Amylase (IU/L) 866 50 – 1250

Lipase (IU/L) 386 30 – 560

Urinalysis

Color Yellow Urine Sediment

Transparency Cloudy WBCs/hpf 1 – 2

Sp. Gr. 1.011 RBCs/hpf 1 – 2

Protein 3 + Epith cells/hpf 5 – 8

Gluc Negative Casts/lpf 0 – 1 coarse

granular and waxy

Bilirubin 1 + Crystals Negative

Blood Trace Bacteria Negative

pH 6.0

UPC 11.1

Page 42: VI Clinical Case Presentations - Wiley · Clinical Case Presentations ... Case 17: Dog with vitamin K antagonist (rodenticide) toxicity, 643 ... PT Prothrombin time

SECTION VI Clinical Case Presentations

656

CA

SES

inability of the damaged renal tubules to respond appropri-ately to mineralocorticoids or may be due to a simple loss of sodium and retention of potassium because of renal disease and oliguria. The serum total CO 2 is decreased, indicating a metabolic acidosis, while the anion gap is increased, indicat-ing the accumulation of organic anions. Acidosis may result in hyperkalemia as well.

The serum calculated osmolality is increased, predomi-nantly because of the profound azotemia. Likewise, the increased anion gap is due to retention of urinary metabolic products.

Serum amylase and lipase activities are normal, and while not defi nitive, lessen the probability for pancreatitis.

Urinalysis The urinary specifi c gravity is in the isosthenuric range, and there is 3 + proteinuria in the absence of signifi cant hema-turia or pyuria. The urinary protein : creatinine ratio is 11.1, which is signifi cantly increased. The mild bilirubinuria is likely signifi cant considering the low specifi c gravity. The coarse granular and waxy casts also defi nitively indicate renal tubular damage. Together with the marked azotemia, these fi ndings support a diagnosis of renal disease.

Summary

Malignant fi brous histiocytoma of both kidneys identifi ed at postmortem examination. This accounted for chronic renal failure.

Interpretive discussion

Hematology The PCV is marginally decreased, but without reticulocyte count it is diffi cult to classify the regenerative response. A marginal normocytic, normochromic anemia is observed in renal failure, for which there are other indications in the laboratory data.

The lymphopenia indicates a steroid response.

Biochemical profi le The serum glucose concentration is normal.

The BUN, serum creatinine, and serum phosphorus values are markedly increased. These fi ndings are consistent with decreased glomerular fi ltration rate. However, one cannot differentiate the nature of the azotemia (prerenal, renal, or postrenal) based on these fi ndings alone. Refer to the discus-sion of urinalysis results for further interpretation.

Serum total calcium is mildly increased, for which one should consider hypercalcemia of malignancy, hypoadreno-corticism, renal failure, vitamin D toxicosis, or primary hyperparathyroidism.

A signifi cant increase in serum ALP activity and mild increase in GGT activity is consistent with cholestasis. Because AST and ALT activities are normal, there is not likely any hepatocellular damage. ALP and GGT activities may also be increased by corticosteroids.

Serum Na and Cl are decreased In concentration, while serum K is increased. The Na : K ratio is 23.2, which may indicate hypoadrenocorticism. Alternatively, renal disease may result in a functional hypoadrenocorticism due to

Page 43: VI Clinical Case Presentations - Wiley · Clinical Case Presentations ... Case 17: Dog with vitamin K antagonist (rodenticide) toxicity, 643 ... PT Prothrombin time

Case 24

657

CA

SES

Hematology Reference Interval

Packed cell volume (%) 23 32 – 52 Hemoglobin (g/dL) 7.9 11 – 18 RBC (10 6 / μ L) 4.41 6.5 – 10.5 MCV (fL) 52 36 – 52 MCHC (g/dL) 34 34 – 39 Total nucleated cell count ( × 10 3 / μ L) 2.5 5.5 – 12.5 Segmented neutrophils ( × 10 3 / μ L) 0.10 2.7 – 6.7 Band neutrophils ( × 10 3 / μ L) 0.025 0 – 0.1 Monocytes ( × 10 3 / μ L) 0.150 0 – 0.8 Lymphocytes ( × 10 3 / μ L) 2.20 1.5 – 5.5 Eosinophils ( × 10 3 / μ L) 0.025 0 – 0.9 Platelets ( × 10 3 / μ L) 217 150 – 500 Plasma protein (g/dL) 4.6 6 – 8 Note: Toxic changes in neutrophils

Case 24 Signalment: 12 - year - old Quarter horse gelding History: Losing weight and recently has loose stools Physical examination: Thin to poor body condition, mild dependent edema all four limbs. Horse developed watery diarrhea during hospitalization.

Interpretive discussion

Hematology The PCV, hemoglobin, and total RBC count are decreased, indicating a moderate anemia is present. It cannot be deter-mined if the anemia is regenerative because reticulocytes are not released from marrow in the horse. Given the biochem-istry profi le the anemia is likely due to chronic renal failure (CRF). The mild to moderate degree of anemia fi ts with CRF as severe anemia is not seen with renal failure unless there is a second problem, e.g., blood loss. If this horse is dehy-drated the anemia is more severe as are the protein losses. The plasma protein is markedly decreased, likely due to diarrhea and GI loss. There is severe, life - threatening leuko-penia, neutropenia with a left shift and toxic changes in neutrophils. This is seen with acute diarrheal disease in horses due to endotoxemia and or overwhelming sepsis. The bone marrow cannot meet demands and a likely source of the problem is enteric salmonellosis with or without septicemia.

Biochemistry profi le This horse has GI and renal disease, both appear to be severe. Marked azotemia, isosthenuria and hypercalcemia with hypophosphatemia are diagnostic for renal failure in the horse. Suspect chronic renal failure due to poor body condition, history, anemia, hypoalbuminemia, electrolyte abnormalities and an inactive urine sediment. Additionally, chronic renal failure is more common than acute renal failure in horses. UN and Ct are about as high as possible in a living patient; clearly this is not due to a prerenal cause. Isosthenuria confi rms renal and postrenal is rare in an adult horse, especially one that is urinating, perhaps even with an increased volume of urine. Lesion in the kidneys is likely to be end - stage, small shrunken kidneys with fi brosis and little to no chance of regeneration or long term survival. Lesion may have started as glomerulonephritis or amyloidosis as hypoalbuminemia is moderate but it can also be seen in severe chronic renal failure from any cause that compro-mises glomerular function. Peripheral edema is attributed to hypoalbuminemia and decreased colloidal osmotic pressure. Horses tend to develop peripheral edema rather then ascites with hypoalbuminemia.

Hypercalcemia and hypophosphatemia are only seen with renal failure in equidae, all other species develop hyperhos-phatemia even if hypercalcemia is present. The only other differentials for hypercalcemia and hypophosphatemia are a malignancy and primary hyperparathyroidism. Hypercalce-mia of malignancy (HHM) is uncommon in horses but has been reported in horses with gastric carcinoma and lym-phoma. Primary hyperparathyroidism is very rare in horses

Biochemical Profi le Reference Interval

Glucose (mg/dL) 153 70 – 110 Blood Urea Nitrogen (mg/dL) 254 14 – 27 Creatinine (mg/dL) 23 1.1 – 2.0 Calcium (mg/dL) 16.7 11 – 13.7 Phosphorus (mg/dL) 1.2 1.9 – 4.1 Total Protein (g/dL) 4.1 5.8 – 7.6 Albumin (g/dL) 1.7 2.7 – 3.7 Globulin (g/dL) 2.4 2.6 – 4.6 Total Bilirubin (mg/dL) 5.2 0.6 – 2.1 Aspartate aminotransferase (IU/L) 229 185 – 300 Alkaline phosphatase (IU/L) 255 90 – 290 GGT (IU/L) 23 7 – 17 CK (IU/L) 2341 130 – 470 Sodium (mEq/L) 125 133 – 145 Potassium (mEq/L) 8.5 2.2 – 4.6 Chloride (mEq/L) 95 100 – 111 Total CO 2 (mEq/L) 17 24 – 34 Anion Gap (mEq/L) 22 5 – 15

Urinalysis voided

Color yellow Urine Sediment Transparency clear WBCs/hpf 0 – 3 Specifi c Gravity 1.009 RBCs/hpf 0 – 5 Protein 2 + Epithelial cells/hpf none Glucose neg Casts/lpf neg Ketones neg Crystals calcium carbonate Blood neg

Page 44: VI Clinical Case Presentations - Wiley · Clinical Case Presentations ... Case 17: Dog with vitamin K antagonist (rodenticide) toxicity, 643 ... PT Prothrombin time

SECTION VI Clinical Case Presentations

658

CA

SES

The proteinuria is more severe than 2 + since the urine is dilute. A protein to creatinine ratio could help assess the degree of severity but is not needed in this case, given all the data. The few white and red blood cells are inconsequen-tial, may be due to voided urine and the calcium carbonate crystals are normal for a horse.

Summary

Marked chronic renal failure and salmonellosis

Outcome Horse was euthanized and chronic renal failure and salmo-nellosis were confi rmed at autopsy. Initially the horse pre-sented primarily for weight loss, while it was being evaluated it developed profuse diarrhea in the hospital. The horse was sick and stressed and likely had a recrudescence of salmo-nella. The horse may have been a carrier of salmonella.

Both kidneys were small, knobby and looked like irregular bovine kidneys due to marked fi brosis. Microscopically there was severe glomerulonephritis, chronic interstitial nephritis and even oxalate crystals in many tubules. The oxalate crys-tals were attributed to either being a horse or endogenous formation of oxalates which happens in chronic renal failure in dogs and other species as well. It was not ethylene glycol toxicity. Glomerulonephritis was considered the primary lesion but it was diffi cult to determine if it preceded the interstitial nephritis. Pulmonary thrombi were present and one large thrombus was present in the pulmonary artery. These thrombi were likely due to hypoalbuminemia and decreased AT III (not measured) which lead to a state of increased coagulability. Pulmonary thrombosis associated with decreased AT III and glomerular lesions is seen most frequently, or documented most frequently in dogs.

Hypercalcemia and hypophosphatemia are present in some horses with renal failure, empirically, perhaps about one third. The pathogenesis is not known and multiple theories are postulated. These electrolyte changes will even occur with nephrectomy in the horse. Some horses may excrete excess dietary calcium (from alfalfa - rich diet) in the urine and retain phosphorus. Hypercalcemia is then attrib-uted to decreased excretion by the failing kidneys and hypo-phosphatemia to increased excretion. However, if the kidneys are removed it seems impossible to explain how they would develop hypophosphatemia. Certainly there cannot be increased renal loss as the kidneys have been removed so it must be increased GI loss, but this is unproven. Many horses with chronic renal failure will have hyperphos-phatemia and normo or hypocalcemia, similar to other species.

and would only be pursued if renal failure and HHM are ruled out fi rst. All three differentials are associated with dilute urine due to inhibition of ADH by hypercalcemia. If mild or moderate azotemia is present with dilute urine in a horse with hypercalcemia and hypophosphatemia it can be diffi cult to distinguish HHM and chronic renal failure. The easiest diagnostic tests are to fi rst search for cancer (enlarged lymph nodes and endoscopy to look for gastric SCC), rectal and or ultrasound examination of kidneys to determine if they are small and shrunken. If this does not clarify then consider protein creatinine ratio and or fractional excretion of sodium, if < 1% rule out renal if > 1% rule in renal. Always favor renal failure over HHM in horses.

Hyponatremia and hypochloremia can be attributed to chronic renal failure and or GI loss, the latter is more likely. The hyperkalemia is severe and life threatening. This is unusual as adult horses tend to develop hypokalemia with GI disease but young horses with diarrhea will have hyper-kalemia. The most likely explanation is metabolic acidosis even though it does not appear that severe in this horse. The pattern of hyponatremia and hyperkalemia can be seen with renal failure, urinary bladder rupture and hypoadrenocorti-cism. Decreased bicarbonate (TCO 2 ) is due to GI and or renal loss. Increased anion gap is due to retained uremic acids, shock and anaerobic glycolysis with accumulation of lactic acid. Horses with diarrheal diseases usually have a metabolic acidosis as does this horse. Hypoalbuminemia and concur-rent decreased serum globulin is due to GI disease, suspect salmonellosis. Hypoalbuminemia is moderate and more severe than the decrease in globulins which may be due to renal loss of albumin in addition to the loss in GI tract. Pro-teinuria without blood is present in the urine and supports renal loss of albumin. Increased bilirubin is due to anorexia, which is the most common cause of icterus in horses. Although hepatic disease and cholestasis are possible expla-nations they are too unlikely given all the clinical and bio-chemistry data (liver enzymes WRI). The mild increase in GGT may be spurious rather than a true indicator of liver issues. GGT seems to increase in horses easily and is not reliable indicator of hepatic problems if increased without any increases in other hepatic parameters.

Increase in CPK is mild for a horse and is due to recum-bency, it is too low to consider a primary muscle disease. The AST is WRI so it is not a muscle problem in which the CPK is decreasing while the AST is still increased. The urine is yellow, no evidence of brown color or blood, rule out myoglobin induced renal damage with these values. Urine is not concentrated; a second check to confi rm isosthenuria is usually recommended but in this case is not needed. Pro-teinuria with no evidence of hemorrhage or active sediment in an animal with hypoalbuminemia is due to renal loss.

Page 45: VI Clinical Case Presentations - Wiley · Clinical Case Presentations ... Case 17: Dog with vitamin K antagonist (rodenticide) toxicity, 643 ... PT Prothrombin time

Case 25

659

CA

SES

Case 25 Signalment: 6 - year - old MC feline DSH History: Approximately 1 month ’ s duration of intermittent weakness, exercise intolerance, poor hair coat Physical examination: Cat is near collapse and approximately 10% dehydrated. There is defi nite cervical ventrofl exion.

Hematology Reference Interval

PCV (%) 41.0 24 – 45

Segs ( × 10 3 / μ L) 18.0 2.5 – 12.5

Monos ( × 10 3 / μ L) 0.7 0 – 0.8

Lymphs ( × 10 3 / μ L) 0.5 1.5 – 7.0

Platelets

( × 10 3 / μ L)

Adequate 150 – 700

Biochemical Profi le

Reference Interval

Gluc (mg/dL) 98 67 – 124

BUN (mg/dL) 68 (24.3) 17 – 31 (6.1 – 11.4 mmol/L)

Creat (mg/dL) 2.8 (247) 0.9 – 2.1 (80 – 186 μ mol/L)

Ca (mg/dL) 10.9 8.5 – 11

Phos (mg/dL) 6.8 3.3 – 7.8

TP (g/dL) 9.3 5.9 – 8.1

Alb (g/dL) 5.3 2.3 – 3.9

Glob (g/dL) 4.0 2.9 – 4.4

T. Bili (mg/dL) 0.3 0 – 0.3

Chol (mg/dL) 180 60 – 220

ALT (IU/L) 52 30 – 100

ALP (IU/L) 48 6 – 106

CK (IU/L) 2419 60 – 300

Na (mEq/L) 157 146 – 160

K (mEq/L) 2.0 3.7 – 5.4

CL (mEq/L) 114 112 – 129

T CO 2 (mEq/L) 15 14 – 23

An. gap (mEq/L) 30 10 – 27

Blood Gas Data (arterial)

Reference Interval

pH 7.130 7.33 – 7.44

pCO 2 (mmHg) 44.0 35 – 42

HCO 3 (mEq/L) 14.0 16 – 22

Urinalysis

Color Yellow Urine Sediment

Transparency Cloudy WBCs/hpf 0 – 2

Sp. Gr. 1.014 RBCs/hpf 0 – 2

Protein Trace Epith cells/hpf 0 – 2

Gluc Negative Casts/lpf Negative

Bilirubin Negative Crystals Negative

Blood Negative Bacteria Negative

pH 5.5 Other

Fractional excretion

Reference Interval

Na (%) 0.55 < 1.0

K (%) 37.7 < 20.0

Page 46: VI Clinical Case Presentations - Wiley · Clinical Case Presentations ... Case 17: Dog with vitamin K antagonist (rodenticide) toxicity, 643 ... PT Prothrombin time

SECTION VI Clinical Case Presentations

660

CA

SES

Interpretive discussion

Hematology There is a mature neutrophilia and lymphopenia, indicating a stress leukogram. Other components of the hemogram are normal.

Biochemical profi le The BUN and serum creatinine concentrations are mildly increased. These fi ndings are consistent with decreased glo-merular fi ltration rate. However, one cannot differentiate the nature of the azotemia (prerenal, renal, or postrenal) based on these fi ndings alone. Refer to the discussion of urinalysis results for further interpretation. The normal serum phosphorus and total calcium concentrations do not contribute to the characterization of renal disease.

Serum total protein and albumin concentrations are increased; this documents marked dehydration or hemoconcentration.

Serum CK activity is increased signifi cantly and is indica-tive of muscle damage.

Serum Na and Cl concentrations are normal, but serum K concentration is markedly decreased. This is especially signifi cant in light of the acidosis, which results in a shift of potassium from within cells to extracellular fl uid and sug-gests a marked potassium defi cit.

Blood gas data There is prominent acidosis. This is due to a combined meta-bolic (decreased HCO 3 ) and respiratory (increased pCO 2 )

acidosis, with an increased anion gap. It would not be unusual for this degree of dehydration to lead to hypovolemia - induced lactic acidosis. It is also possible that this degree of hypokalemia may have caused suffi cient respiratory muscle dysfunction to impair normal ventilation.

Urinalysis The urinary specifi c gravity is in the isosthenuric range. Given the azotemia and normal serum Na and Cl concentra-tions, this indicates probable renal disease. However, hypo-kalemia can also impair ADH responsiveness by the kidneys, so that urine concentration should be evaluated following rehydration and K repletion.

The urinary FE Na is 0.55%, which speaks against a gener-alized renal tubular disease. However, the . FE K is 37.7%, which is markedly increased, especially for a cat with this degree of hypokalemia.

Summary

The combined observations of azotemia, hypokalemia, aci-dosis, and hyperkaluria in a cat with cervical ventrofl exion and evidence of widespread muscle damage support a diag-nosis of feline kaliopenic polymyopathy/nephropathy syn-drome. In this case, it was completely corrected by dietary change (non - acidifying, higher K diet). This syndrome is no longer seen, as dietary imbalances in commercial cat food were corrected.

Page 47: VI Clinical Case Presentations - Wiley · Clinical Case Presentations ... Case 17: Dog with vitamin K antagonist (rodenticide) toxicity, 643 ... PT Prothrombin time

Case 26

661

CA

SES

Case 26 Signalment: 2 - year - old male canine, West Highland white terrier History: Polyuria, polydipsia

Hematology Reference Interval

PCV (%) 33.0 37 – 55

Hgb (g/dLO) 11.3 12 – 18

RBC ( × 10 6 / μ L) 4.45 5.5 – 8.5

MCV (fL) 74.0 60 – 72

MCHC (g/dL) 35.0 33 – 38

NCC ( × 10 3 / μ L) 5.9 6 – 17

Segs ( × 10 3 / μ L) 3.9 3 – 11.5

Monos ( × 10 3 / μ L) 0.4 0.1 – 1.3

Lymphs ( × 10 3 / μ L) 1.2 1 – 4.8

NRBC ( × 10 3 / μ L) 0.4 0

Platelets ( × 10 3 / μ L) 425 200 – 500

TP (P) (g/dL) 6.7 6 – 8

Hemopathology: Few acanthocytes and schistocytes

Biochemical Profi le

Reference Interval

Gluc (mg/dL) 108 65 – 122

BUN (mg/dL) 65 (23.2) 7 – 28 (2.5 – 10.0 mmol/L)

Creat (mg/dL) 2.0 (176.8) 0.9 – 1.7 (79 – 150 μ mol/L)

Ca (mg/dL) 7.2 (1.8) 9.0 – 11.2 (2.25 – 2.8 mmol/L)

Phos (mg/dL) 6.1 2.8 – 6.1

TP (g/dL) 5.8 5.4 – 7.4

Alb (g/dL) 3.7 2.7 – 4.5

Glob (g/dL) 2.1 1.9 – 3.4

T. Bili (mg/dL) 0.3 0 – 0.4

Chol (mg/dL) 382 (9.9) 130 – 370 (3.4 – 9.6 mmol/L)

ALT (IU/L) 56 10 – 120

ALP (IU/L) 137 35 – 280

Na (mEq/L) 147 145 – 158

K (mEq/L) 3.0 4.1 – 5.5

CL (mEq/L) 115 106 – 127

TCO 2 (mEq/L) 22.3 14 – 27

An. gap (mEq/L) 12.7 8 – 25

Calc. osmolality

(mOsm/kg)

317 290 – 310

Blood Gas Data (arterial)

Reference Interval

pH 7.349 7.33 – 7.45

PO 2 (mmHg) 80.1 67 – 92

PCO 2 (mmHg) 39.1 24 – 39

HCO 3 (mEq/L) 21.0 14 – 24

ionized Ca + + (mEq/L) 3.44 4.5 – 5.6

Urinalysis

Color Yellow Urine Sediment

Transparency Clear WBCs/hpf 3 – 6

Sp. Gr. 1.028 RBCs/hpf 3 – 6

Protein 2 + Epith cells/hpf 0 – 2

Gluc 3 + Casts/lpf Rare fi ne gran

Bilirubin 1 + Crystals Negative

Blood Negative Bacteria Negative

pH 5.0

Ketones Trace

osmolality 358

(mOsm/L)

UPC 1.75

Fractional excretion

Reference Interval

Na (%) 1.62 < 1.0

Ca (%) 7.47 < 1.0

Page 48: VI Clinical Case Presentations - Wiley · Clinical Case Presentations ... Case 17: Dog with vitamin K antagonist (rodenticide) toxicity, 643 ... PT Prothrombin time

SECTION VI Clinical Case Presentations

662

CA

SES

Urinalysis Although the urinary specifi c gravity indicates some concen-trating ability, one would expect this to be greater if the azotemia were pre - renal in origin. It is also possible for the specifi c gravity to be increased by the presence of solutes which do not contribute to renal concentration capacity (glucose, protein, amino acids). Concomitant determination of urinary . osmolality (358 mOsm/L) confi rms that the urine is not being adequately concentrated relative to the calcu-lated osmolality of the serum. Inability to concentrate the urine may be due to central diabetes insipidus (a defect in hypothalamic/pituitary antidiuretic hormone release), or nephrogenic diabetes insipidus (ADH is released, but the kidney is unable to respond). The latter may be caused by anatomic pathology or functional impairment of renal tubular actions necessary to maintain a medullary concen-tration gradient and water reabsorption. This fi nding may indicate that the observed azotemia is renal in origin.

The presence of proteinuria on the dipstick was followed by a chemical determination of urinary protein concentra-tion. When indexed to the urinary creatinine value, the urinary protein : creatinine ratio is 1.75. While this value is probably abnormal, it is not suffi ciently high to indicate glomerular protein loss. Values in the range of 1.0 to 2.0 have been associated epidemiologically with tubular or infl ammatory causes of proteinuria. The absence of signifi -cant numbers of leukocytes suggests there is no infl amma-tory disease. The presence of fi ne granular casts is indicative of renal tubular damage, and may explain the proteinuria.

Glucosuria concomitant to euglycemia may be explained by three mechanisms. (1) There is a Fanconi ’ s - type syn-drome wherein tubular malfunction leads to loss of glucose, protein, and other solutes which would otherwise be reab-sorbed from the glomerular fi ltrate. This is supported by the fi ndings of modest proteinuria and increased urinary frac-tional excretion of electrolytes. Fanconi ’ s syndromes may be inherited (as reported in Basenjis and Whippets) or acquired (as reported following exposure to nephrotoxicants, includ-ing aminoglycoside antibiotics and heavy metals). (2) There was an earlier episode of hyperglycemia which exceeded the renal threshold for glucose reabsorption, during which time the urine analyzed was produced. Depending on the rate of urine formation, a single void may represent blood chemistry - related changes for many hours prior to specimen collection. (3) A laboratory error was made in the determi-nation of either the serum glucose (improper preservation of the blood sample or analytical error) or the urinary glucose (cross - contamination of dipstick reaction squares by excess urine or operator error in interpreting the color change).

The FE Na is 1.62%. This may be indicative of renal tubular disease or dysfunction due to mineralocorticoid defi ciency or transport malfunction. The FE Ca is 7.47%. This is particu-larly inappropriate given the hypocalcemia, and may well

Interpretive discussion

Hematology The packed cell volume, erythrocyte count, and hemoglobin concentration are decreased, indicating an anemia. Observed red blood cell morphologic abnormalities include acantho-cytes and schistocytes. These may be observed when there is erythrocytic membrane damage due to free radical or lipid metabolic abnormalities, or when there is microangiopathic pathology due to vascular disease or neoplasia. Although a reticulocyte count has not been provided, the increased erythrocyte MCV and nucleated erythrocytes are consistent with a regenerative responce. There are no other hemato-logic abnormalities.

Biochemical profi le Serum glucose concentration is normal, and its importance in the interpretation of the glucosuria is discussed below.

The BUN and serum creatinine concentrations are increased, while the serum phosphorus value is at the upper limit of the reference interval. These fi ndings are consistent with decreased glomerular fi ltration rate. However, one cannot differentiate the nature of the azotemia (prerenal, renal, or postrenal) based on these fi ndings alone. Refer to the discussion of urinalysis results for further interpretation.

The serum total calcium concentration is decreased. The ionized calcium concentration reported with the blood gas panel is likewise less than normal, indicating a true hypo-calcemia. In this case, excessive loss of calcium in the urine is the likely cause (see urinalysis discussion).

Serum total protein, albumin, and globulin concentrations are within the reference interval. This observation suggests that there is not hemoconcentration due to dehydration, although a concurrent protein - losing disorder might exist. Thus, the azotemia noted above is less likely due to dehydra-tion, and more likely renal in origin.

Serum cholesterol is increased, whereas other indices of hepatic function are normal. There are no other indicators of a primary metabolic disease like diabetes mellitus, but it is possible, nevertheless, that this dog has hypothyroidism or hyperadrenocorticism.

The serum sodium and chloride concentrations are normal, yet there is hypokalemia. Possible causes in this case might include hyperadrenocorticism, chronic renal disease, or urinary potassium wasting associated with diuresis. Cal-culated serum osmolality is mildly increased due to the azotemia.

Blood gas data Indices of acid - base metabolism (pH, pCO 2 , HCO 3 , and anion gap) are normal.

Page 49: VI Clinical Case Presentations - Wiley · Clinical Case Presentations ... Case 17: Dog with vitamin K antagonist (rodenticide) toxicity, 643 ... PT Prothrombin time

Case 26

663

CA

SES

be the cause of calcium loss from the body. This may be indicative of renal tubular disease or dysfunction due to parathyroid hormone defi ciency or transport malfunction. Increased urinary excretion of both of these electrolytes may be observed in renal failure (consider the azotemia and impaired urinary concentrating ability) or in Fanconi ’ s syn-drome, wherein proximal renal tubule reabsorptive function is impaired (consider the euglycemic glucosuria).

Summary

This is a case of congenital Fanconi ’ s syndrome which did not resolve following supportive treatment for renal failure. Other tests one should perform include those that evaluate the parathyroid gland.

Page 50: VI Clinical Case Presentations - Wiley · Clinical Case Presentations ... Case 17: Dog with vitamin K antagonist (rodenticide) toxicity, 643 ... PT Prothrombin time

SECTION VI Clinical Case Presentations

664

CA

SES

Signalment: 8 - year - old male canine History: Polydipsia Physical Examination: Slightly dehydrated

Case 27

Hematology Reference Interval

PCV (%) 38.0 37 – 55

Hgb (g/dL) 12.0 12 – 18

RBC ( × 10 6 / μ L) 5.51 5.5 – 8.5

MCV (fL) 69.0 60 – 72

NCC ( × 10 3 / μ L) 18.2 6 – 17

Segs ( × 10 3 / μ L) 2.0 3 – 11.5

Monos ( × 10 3 / μ L) 0.6 0.1 – 0.3

Lymphs ( × 10 3 / μ L) 13.8 1 – 4.8

Platelets ( × 10 3 / μ L) 298 200 – 500

TP (P) (g/dL) 8.8 6 – 8

Hemopathlogy: clumped platelets

Biochemical Profi le

Reference Interval

Gluc (mg/dL) 91 65 – 122

BUN (mg/dL) 33 (11.8) 7 – 28 (2.5 – 10.0 mmol/L)

Creat (mg/dL) 2.9 (256) 0.9 – 1.7 (80 – 150 μ mol/L)

Ca (mg/dL) 15.4 (3.85) 9.0 – 11.2 (2.25 – 2.80 mmol/L)

Phos (mg/dL) 7.1 (2.3) 2.8 – 6.1 (0.9 – 2.0 mmol/L)

TP (mg/dL) 7.9 5.4 – 7.4

Alb (g/dL) 4.0 2.7 – 4.5

Glob (g/dL) 3.9 1.9 – 3.4

T. Bili (mg/dL) 1.0 (17) 0 – 0.4 (0 – 6.8 μ mol/L)

Chol (mg/dL) 291 130 – 370

ALT (IU/L) 152 10 – 120

AST (IU/L) 64 16 – 40

ALP (IU/L) 361 35 – 280

GGT (IU/L) 14 0 – 6

Na (mEq/L) 154 145 – 158

K (mEq/L) 5.8 4.1 – 5.5

CL (mEq/L) 109 106 – 127

TCO 2 (mEq/L) 12.1 14 – 27

An. gap (mEq/L) 38.7 8 – 25

Urinalysis

Color Straw Urine Sediment

Transparency Clear WBCs/hpf 2 – 3

Sp. Gr. 1.011 RBCs/hpf 1 – 2

Protein 2 + Epith cells/hpf Negative

Gluc Negative Casts/lpf Negative

Bilirubin 2 + Crystals Negative

Blood Negative Bacteria Negative

pH 6.5

UPC 2.6

Fractional excretion

Reference Interval

Na (%) 1.73 < 1.0

Ca (%) 3.37 < 1.0

Page 51: VI Clinical Case Presentations - Wiley · Clinical Case Presentations ... Case 17: Dog with vitamin K antagonist (rodenticide) toxicity, 643 ... PT Prothrombin time

Case 27

665

CA

SES

of cholestasis. There are mild increases in the serum activi-ties of ALT and AST, so there may be some hepatocellular damage as well.

The increase in serum potassium is probably due to redis-tribution of intracellular potassium to the extracellular space secondary to acidosis. The serum total CO 2 concentration is mildly decreased, indicating a metabolic acidosis. A complete blood gas panel is required to completely evaluate acid - base status.

Urinalysis The urine specifi c gravity is in the isosthenuric range. The dog does not appear to be dehydrated, and it is possible for a normal dog to produce urine with a specifi c gravity in this range. However, this dog is azotemic. Dilute urine in the face of azotemia usually indicates renal disease, but hypercalce-mia interferes with concentrating ability by antagonizing the actions of ADH. Hypercalcemia may also cause damage to the kidney, especially when phosphorus is concurrently increased. There is signifi cant proteinuria of 2 + on the dip-stick, and a UPC of 2.6. In the absence of signifi cant sedi-ment changes, this is indicative of renal protein loss, probably glomerular in origin. The FE Na is 1.73%, indicating tubular dysfunction. Increased fractional excretion of Ca is expected given the hypercalcemia.

Summary

This is a case of lymphoma with hypercalcemia of malig-nancy and hypercalcemic nephropathy.

Interpretive discussion

Hematology The nucleated cell count is mildly increased, but there is a neutropenia and marked lymphocytosis. Other hematologic parameters, including cell morphology, are normal. However, the concurrent observation of marked lymphocytosis and neutropenia should alert one to the possibility of lympho-cytic leukemia, lymphoma with bone marrow involvement, or ehrlichiosis. The concurrent observation of marked lym-phocytosis and hypercalcemia should likewise lead to con-sideration of lymphoma and humoral hypercalcemia of malignancy.

Biochemical profi le The BUN, creatinine, and phosphorus concentrations are mildly increased. These fi ndings are consistent with decreased glomerular fi ltration rate. However, one cannot differentiate the nature of the azotemia (pre - renal, renal, or post - renal) based on these fi ndings alone. Refer to the dis-cussion of urinalysis results for further interpretation.

The serum total calcium concentration is markedly increased. In light of the lymphocytosis, humoral hyper-calcemia of malignancy is most likely. PTHrP could be measured to support this interpretation. The Ca × P pro-duct is increased at 109, indicating likely soft tissue mineralization.

The serum total protein and globulin concentrations are slightly increased. Increased globulin concentration may occur in dogs with lymphoproliferative disorders.

The total bilirubin concentration is increased, as are the serum ALP and GGT activities. These fi ndings are evidence

Page 52: VI Clinical Case Presentations - Wiley · Clinical Case Presentations ... Case 17: Dog with vitamin K antagonist (rodenticide) toxicity, 643 ... PT Prothrombin time

SECTION VI Clinical Case Presentations

666

CA

SES

Case 28 Signalment: 9 - year - old female dog History: Polydipsia, polyuria Physical Examination: Mass in pelvic inlet

Interpretive discussion

Biochemical profi le Hyperproteinemia is due to hyperalbuminemia, indicating dehydration. There is marked hypercalcemia and this mag-nitude of increase is suggestive of hypercalcemia of malig-nancy or primary hyperparathyroidism. Hypercalcemia of this magnitude may result in renal injury leading to azote-mia and loss of concentrating ability. A dehydrated animal should be maximally concentrating its urine, and this dog ’ s urine is nearly isosthenuric. This could be consistent with renal disease (early, prior to development of azotemia), but hypercalcemia alone is suffi cient to explain this abnormality (due to antagonism of ADH at the renal tubules).

A slight increase in alkaline phosphatase activity (ALP) suggests cholestasis or drug induction (corticosteroids, anti-convulsants). If cholestasis is present it is not of suffi cient magnitude to affect the serum bilirubin. A more likely expla-nation for the increased alkaline phosphatase is increased bone turnover secondary to increased serum concentration of PTH or PTHrp, which could be measured.

Summary

The mass in the pelvis was aspirated, and appeared neuro-endocrine, rather than lymphoid. The mass was surgically removed and confi rmed by histopathology to be an apocrine gland adenocarcinoma of the anal sac. Following surgery, the calcium normalized, but later metastasis to the lungs resulted in return of hypercalcemia. In contrast to the previ-ous case, the hypercalcemia has not resulted in suffi cient renal injury to cause azotemia.

Biochemical Profi le

Reference Interval

Gluc (mg/dL) 106 65 – 122

BUN (mg/dL) 8 7 – 28

Creat (mg/dL) 1.4 0.9 – 1.7

TP (g/dL) 7.7 5.4 – 7.4

Alb (g/dL) 5.2 2.7 – 4.5

Ca (mg/dL) 16.4 (4.5) 9.0 – 11.2 (2.25 – 2.80 mmol/L)

Phos (mg/dL) 3.5 2.8 – 6.1

T. Bili (mg/dL) 0.2 0 – 0.4

ALT (IU/L) 43 10 – 120

ALP (IU/L) 428 35 – 280

Na (mEq/L) 155 145 – 158

K (mEq/L) 3.9 4.1 – 5.5

CL (mEq/L) 119 106 – 127

TCO 2 (mEq/L) 21.6 14 – 27

Urinalysis

Specifi c gravity 1.014

Page 53: VI Clinical Case Presentations - Wiley · Clinical Case Presentations ... Case 17: Dog with vitamin K antagonist (rodenticide) toxicity, 643 ... PT Prothrombin time

Case 29

667

CA

SES

Interpretive discussion

Hematology There is a stress leukogram indicated by mature neutrophilia and lymphopenia.

Biochemical profi le Increased glucose is compatible with stress identifi ed in the leukogram. The BUN and creatinine are mildly increased, indi-cating azotemia. The urine specifi c gravity is less than what one would expect in a cat with prerenal azotemia, therefore renal azotemia should be considered. However, hypercalcemia alone can interfere with normal concentrating ability. With a calcium of 18 mg/dL, renal dysfunction is likely occurring due to soft tissue mineralization. The total protein and albumin are increased, particularly for a young cat, indicating dehydration.

The calcium is markedly increased. Primary causes of this degree of hypercalcemia are hypercalcemia of malignancy, primary hyperparathyroidism, and hypervitaminosis D. Vitamin D toxicosis should be very high on the differential list, due to the age of the cat and the acute onset. Because of acidosis, ionized calciun is likely very high. Phosphorus is mildly increased. This could be in part due to the young age of the cat or may be due to decreased GFR. It is also seen with hypervitaminosis D. The Ca × P product is markedly increased at 171, which will result in calcifi cation of renal tubules, lungs and other soft tissues.

Potassium is increased. This may be due to acidosis and an associated shift of K out of cells or the animal may be becoming oliguric. Increased anion gap indicates increase in unmeasured anions. Possible unmeasured anions are lactic acid or uremic acids. Increased phosphates are also adding to the anion gap.

Blood gas data The pH is extremely decreased indicating severe acidemia. The pCO 2 is the major abnormality in the balance between bicarbonate and CO 2 . Therefore, respiratory acidosis is the major component of the acidosis. The bicarbonate is also decreased indicating a component of metabolic acidosis is superimposed. Hypoxemia is also present. The combined hypoxemia and retention of CO 2 indicate a severe ventila-tion abnormality, probably due to calcifi cation of lungs. The metabolic acidosis is probably a result of renal failure.

Summary

The cat was diagnosed with renal disease with severe acide-mia, both metabolic and respiratory. The respiratory compo-nent may be due to calcifi cation of lungs. Cholecalciferol toxicosis was diagnosed, as the cat had an opportunity to ingest a rodenticide containing cholecalciferol.

Case 29 Signalment: 6 - month - old DSH female cat History: Vomiting, weakness, acute onset Physical examination: Tachypnea for 24 hours, weakness

Hematology Reference Interval

PCV (%) 40 24 – 45

WBC ( × 10 3 / μ L) 21.0 5.5 – 19.5

Segs ( × 10 3 / μ L) 20.2 2.5 – 12.5

Bands ( × 10 3 / μ L) 0 0 – 0.3

Lymphs ( × 10 3 / μ L) 0.2 1.5 – 7.0

Monos ( × 10 3 / μ L) 0.6 0 – 0.85

Biochemical Profi le

Reference Interval

Gluc (mg/dL) 150 (8.2) 67 – 124 (3.7 – 6.8 mmol/L)

BUN (mg/dL) 45 (16.1) 17 – 32 (6.1 – 11.4 mmol/L)

Creat (mg/dL) 2.2 (194) 0.9 – 2.1 (80 – 186 μ mol/L)

Ca (mg/dL) 18 (4.5) 8.5 – 11 (2.12 – 2.75 mmol/L)

Phos (mg/dL) 9.5 (3.1) 3.3 – 7.8 (1.1 – 2.5 mmol/L)

TP (g/dL) 8.0 5.9 – 8.1

Alb (g/dL) 4.2 2.3 – 3.9

Glob (g/dL) 3.8 2.9 – 4.4

T. Bili (mg/dL) 0.2 0 – 0.3

Chol (mg/dL) 120 60 – 270

ALT (IU/L) 100 30 – 100

ALP (IU/L) 25 11 – 210

Na (mEq/L) 159 146 – 160

K (mEq/L) 6.4 3.7 – 5.4

CL (mEq/L) 112 112 – 129

TCO 2 (mEq/L) 16.8 14 – 24

An. gap (mEq/L) 37 10 – 27

Blood Gas Data (arterial)

Reference Interval

pH 6.926 7.33 – 7.44

PCO 2 (mmHg) 72.1 35 – 42

PO 2 (mmHg) 65 80 – 95

HCO 3 (mEq/L) 14.9 16 – 22

Urinalysis

Sp. Gr. 1.020

Gran casts/hpf 2

Page 54: VI Clinical Case Presentations - Wiley · Clinical Case Presentations ... Case 17: Dog with vitamin K antagonist (rodenticide) toxicity, 643 ... PT Prothrombin time

SECTION VI Clinical Case Presentations

668

CA

SES

Interpretive discussion

Hematology PCV is slightly increased. This is likely due to dehydration, considering that the albumin is also increased. The mature neutrophilia and lymphopenia are suggestive of a stress or corticosteroid leukogram.

Biochemical profi le The serum glucose concentration is increased. Differentials should include stress or corticosteroids, excitement and dia-betes mellitus. Excitement is less likely than the others, since the cat does not have an excitement leukogram and there is glucosuria. (See summary for further discussion of hyperglycemia.)

The BUN and creatinine are increased, and considering that the cat is not concentrating its urine, this is most likely a renal azotemia. Since the cat is dehydrated, a prerenal component to the azotemia may be present as well. Because the cat is not anemic, is obese, and the history is acute, this is most likely acute renal failure. Phosphorus is increased due to decreased glomerular fi ltration rate.

The serum calcium is decreased. Considering that the cat likely has acute renal failure, the most likely cause of the hypocalcemia is formation of calcium oxalate crystals associ-ated with ethylene glycol toxicosis. Oxalate is one of the metabolites of ethylene glycol and combines with calcium to form calcium oxalate crystals.

Hyperproteinemia is due to hyperalbuminemia, indicating dehydration.

The sodium is increased, likely due to dehydration. Chlo-ride would be expected to increase with sodium, but is selectively decreased in this case, probably due to vomiting of gastric HCl. This causes a hypochloremic alkalosis. However, TCO 2 is decreased and the anion gap is increased, suggesting concurrent metabolic acidosis and a mixed acid - base disorder. A blood gas would more fully characterize the acid - base status.

The increased anion gap indicates increased concentra-tions of anions other than those used in the formula to calculate the anion gap (chloride and HCO 3

− ). In this case, uremic acids, phosphate, albumin, and most importantly, metabolites of ethylene glycol are probably contributing to the anion gap and a high - gap metabolic acidosis.

The calculated osmolality is increased, since the substances that are included in the formula to calculate osmolality are increased (glucose, urea, sodium, potassium). However, the measured plasma osmolality is much higher than the calcu-lated osmolality, since a substance is present in the blood that is not used in the formula to calculate osmolality. The most common cause of an increased osmole gap is the pres-

Case 30 Signalment: 3 - year - old male cat History: Acute lethargy, vomiting, and anorexia Physical examination: Obese, almost comatose

Hematology Reference Interval

PCV (%) 50 24 – 45

NCC ( × 10 3 / μ L) 24.0 5.5 – 19.5

Segs ( × 10 3 / μ L) 23.0 2.5 – 12.5

Monos ( × 10 3 / μ L) 0.7 0 – 0.88

Lymphs ( × 10 3 / μ L) 0.3 1.5 – 7.0

Platelets ( × 10 3 / μ L) Adequate 200 – 500

Biochemical Profi le Reference Interval

Gluc (mg/dL) 285 67 – 124

BUN (mg/dL) 110 17 – 32

Creat (mg/dL) 7.5 0.9 – 2.1

Ca (mg/dL) 6.5 8.5 – 11

Phos (mg/dL) 14 3.3 – 7.8

TP (g/dL) 9.0 5.9 – 8.1

Alb (g/dL) 4.9 2.3 – 3.9

Glob (g/dL) 4.1 2.9 – 4.4

T. Bili (mg/dL) 0.3 0 – 0.3

ALT (IU/L) 35 30 – 100

ALP (IU/L) 45 11 – 210

Na (mEq/L) 165 146 – 160

K (mEq/L) 6.8 3.7 – 5.4

CL (mEq/L) 107 112 – 129

TCO 2 (mEq/L) 10 14 – 23

An. gap (mEq/L) 55 10 – 27

Calc. Osmolality (mOsm/kg) 394 290 – 310

Meas. Osmolality (mOsm/kg) 440 290 – 310

Osmolal Gap (mOsm/kg) 46 > 10

Urinalysis (cystocentesis)

Color Yellow Urine Sediment

Transparency Cloudy WBCs/hpf 2 – 3

Sp. Gr. 1.016 RBCs/hpf 2 – 3

Protein 1 + Epith cells/hpf 1 – 3 transitional

Gluc 1 + Casts/lpf 0

Bilirubin Negative Crystals Calcium oxalate monohydrate

Blood 1 + Bacteria 0

pH 5.0

Page 55: VI Clinical Case Presentations - Wiley · Clinical Case Presentations ... Case 17: Dog with vitamin K antagonist (rodenticide) toxicity, 643 ... PT Prothrombin time

Case 30

669

CA

SES

ence of ethylene glycol, which contributes to plasma osmo-lality due to its low molecular weight.

Urinalysis The urine specifi c gravity of 1.016 in an azotemic dehy-drated cat indicates that the cat is not capable of concentrat-ing urine, and that renal dysfunction is present. The presence of calcium oxalate monohydrate crystals in a cat with acute renal failure is very suggestive of ethylene glycol toxicosis. The renal threshold for glucose has been exceeded, resulting in glucosuria. The 1 + proteinuria is probably signifi cant in light of the low urine specifi c gravity and probably resulted from tubular damage.

Summary

The cat died, and necropsy revealed renal tubular necrosis and the presence of calcium oxalate crystals in the tubules due to ethylene glycol toxicosis. The cat had access to anti-freeze shortly before it became ill. Approximately 50% of dogs and cats with ethylene glycol induced renal failure have hyperglycemia, probably due to a combination of stress and the formation of aldehyde, a metabolite of ethylene glycol that interferes with glucose metabolism. While diabe-tes mellitus could cause hyperglycemia and metabolic aci-dosis, the presence of acute renal failure and calcium oxalate crystalluria should prompt consideration of ethylene glycol toxicosis.

Page 56: VI Clinical Case Presentations - Wiley · Clinical Case Presentations ... Case 17: Dog with vitamin K antagonist (rodenticide) toxicity, 643 ... PT Prothrombin time

SECTION VI Clinical Case Presentations

670

CA

SES

Case 31 Signalment: 3 - month - old Saint Bernard History: Stumbling for 4 hours Physical examination: Cannot stand, in a stupor Hematology: No abnormalities

Biochemical Profi le Reference Interval

Gluc (mg/dL) 129 65 – 122

BUN (mg/dL) 20 7 – 28

Creat (mg/dL) 1.6 0.9 – 1.7

Ca (mg/dL) 11.2 9.0 – 11.2

Ionized calcium (mg/dL) 5.6 4.5 – 5.6

Phos (mg/dL) 10.2 2.8 – 6.1

TP (g/dL) 5.8 5.4 – 7.4

Alb (g/dL) 2.9 2.7 – 4.5

Glob (g/dL) 2.4 1.9 – 3.4

T. Bili (mg/dL) 0.2 0 – 0.4

Chol (mg/dL) 220 130 – 37

ALT (IU/L) 60 10 – 120

AST (IU/L) 30 16 – 40

ALP (IU/L) 300 35 – 280

GGT (IU/L) 2 0 – 6

Na (mEq/L) 148 145 – 158

K (mEq/L) 5.2 4.1 – 5.5

CL (mEq/L) 105 106 – 127

HCO 3 (mEq/L) 15.1 14 – 27

An. gap (mEq/L) 33 8 – 25

Meas. Osmolality (mOsm/kg) 442 290 – 310

Calc. Osmolality (mOsm/kg) 330 290 – 310

Osmole gap (mOsm/kg) 112 0 – 10

Serum ethylene glycol

concentration (mg/dL)

> 250 0

Blood Gas Data (arterial)

Reference Interval

Blood pH 7.305 7.33 – 7.44

HCO 3 (mEq/L) 13.7 16 – 22

PCO 2 (mm/Hg) 29 35 – 42

Urinalysis

Urine specifi c gravity 1.012

Urine pH 5

Interpretive discussion

Biochemical profi le The serum glucose concentration is slightly increased. This may be due to stress, although the leukogram is normal. Aldehydes, a metabolite of ethylene glycol (see later discus-sion), are reported to interfere with glucose metabolism.

The BUN and creatinine are normal in this dog that has a high serum ethylene glycol concentration. In dogs, azotemia begins between 24 and 36 hours following ingestion. The history suggests that this dog ingested antifreeze approxi-mately 5 hours prior to the time of these laboratory data.

Phosphorus is markedly increased. Hyperphosphatemia may be due to the young age of the dog, but is somewhat high for this. In this case the serum phosphorus increase was likely due to phosphate rust inhibitors present in most com-mercial antifreeze.

Serum alkaline phosphatase activity is mildly increased, likely due to the bone isoform that is increased in growing dogs.

The anion gap is increased, likely due to either phosphates or metabolites of ethylene glycol, which are anions. The calculated osmolality is slightly increased. However, the actual (measured) plasma osmolality is much higher than the calculated osmolality, resulting in a large osmole gap, since a substance is present in the blood that is not used in the formula to calculate osmolality. The most common cause of an increased osmole gap is the presence of ethylene glycol, which contributes to plasma osmolality due to its low molecular weight. This was confi rmed by measuring serum ethylene glycol concentration.

Blood gas data The blood pH is slightly low and HCO 3 is decreased, indicat-ing metabolic acidosis. Metabolites of ethylene glycol are acids. Decreased pCO 2 is consistent with a compensatory respiratory alkalosis. The blood gases were determined about one hour following the biochemical profi le, which probably accounts for the discrepancy between the HCO 3 determined on the biochemical panel, and that from the blood gas machine.

Urinalysis The urine specifi c gravity of 1.012 in this patient is likely due to ethylene glycol causing osmotic diuresis. It is also possible that concentrating ability has been impaired, but the animal is not yet azotemic.

Summary

The dog was treated with fomepizole, an alcohol dehydro-genase inhibitor, approximately 7 to 8 hours following anti-freeze ingestion, and did not become azotemic. In contrast to the previous case, the biochemical profi le is often not diagnostic in acute ethylene glycol poisoning, and other tests, such as serum ethylene glycol concentration or mea-sured osmolality must be used to confi rm the diagnosis. The acute onset of stumbling and stupor triggered suspicion of ethylene glycol toxicosis.

Page 57: VI Clinical Case Presentations - Wiley · Clinical Case Presentations ... Case 17: Dog with vitamin K antagonist (rodenticide) toxicity, 643 ... PT Prothrombin time

Case 32

671

CA

SES

Case 32

Hematology Reference Interval

PCV (%) 32 28 – 46

Hgb (g/dL) 11 11 – 16

TP (P) (g/dL) 5.6 6 – 8

Signalment: Five - day - old male foal History: Foal was fi ne at birth, now will not eat Physical examination: Acts colicky, seems constipated

Interpretive discussion

Hematology There is a mild decrease in plasma proteins; concerned about failure of passive transfer of colostrum but globulins on chemistry panel are adequate.

Biochemical profi le Azotemia is present but there is no urine specifi c gravity to help determine if this is prerenal or renal azotemia. Recom-mend catheterization to collect urine and determine if the foal can urinate on its own. UN is mildly increased but cre-atinine (Ct) is moderately increased, the ratio of UN/Ct is 6. This can happen with recent fl uid therapy and a more rapid decrease in UN than Ct, but no fl uids have been adminis-tered. It could happen with a hepatic shunt and decreased synthesis of UN and with noncreatinine chromogens in horses and cattle. The latter is more likely but confuses the interpretation of the severity of the azotemia. The increase in phosphorus is consistent with a decreased GFR; prerenal, renal, or postrenal causes can do this.

The key abnormalities are the hyponatremia, hyperkale-mia and hypochloremia. The Na : K ratio is low at 16 (see comments section). In a foal the most likely differentials are diarrhea, ruptured urinary bladder, renal failure and hypo-adrenocorticism due to a septicemia. There is no evidence of diarrhea at this time; renal failure is a possible explana-tion. To evaluate uroabdomen recommend abdominocente-sis and determination of Ct (and or UN) on the abdominal fl uid and in a concurrent serum sample to compare Ct in abdominal fl uid to serum; they should be equal if the bladder wall is intact. Ct (and or UN) will be higher in abdominal fl uid than serum if bladder has ruptured. Hypoadrenocorti-cism is unlikely; only pursue if other differentials are ruled out. Bicarbonate (TCO 2 ) is decreased, AG is high end of reference interval, and there likely is a metabolic acidosis, possibly due to dehydration and decreased tissue perfusion. ALP is mildly increased probably due to bone (growth) or liver source; does not seem important to pursue at this time. The bilirubin is increased which could be due to a liver problem but anorexia is the more likely cause (anorexia is number one reason for hyperbilirubinemia in horses).

Biochemical Profi le

Reference Interval

Gluc (mg/dL) 80 70 – 110

UN (mg/dL) 32 7 – 27

Creat (mg/dL) 4.8 1.1 – 2.0

Ca (mg/dL) 9.6 11 – 13.7

Phos (mg/dL) 10 1.9 – 3.6

TP (g/dL) 5.9 5.8 – 7.6

Alb (g/dL) 3.0 2.7 – 3.7

Glob (g/dL) 2.9 2.6 – 4.6

T. Bili (mg/dL) 3.8 0.6 – 2.5

AST (IU/L) 229 185 – 300

ALP (IU/L) 340 66 – 180

CK (IU/L) 237 130 – 470

Na (mEq/L) 118 133 – 145

K (mEq/L) 7.1 2.2 – 4.6

CL (mEq/L) 92 98 – 103

TCO 2 (mEq/L) 18 24 – 29

An. gap (mEq/L) 15 10 – 15

Abdominocentesis

Clear, slight yellow color

TNCC 8500/ µ L 50:50 neutrophils and mononuclear cells

Total protein (refractive index) 2.8 g/dL

Creatinine (mg/dL)

Abdominal fl uid 9.2

Serum 4.8

Page 58: VI Clinical Case Presentations - Wiley · Clinical Case Presentations ... Case 17: Dog with vitamin K antagonist (rodenticide) toxicity, 643 ... PT Prothrombin time

SECTION VI Clinical Case Presentations

672

CA

SES

mg/dL higher than the serum Ct. If the bladder wall is intact the serum and abdominal Ct will be similar or identical. If the patient is not azotemic then both samples will have Ct concentrations WRI, if the bladder wall is intact. If the patient has renal failure and is azotemic than the abdominal Ct concentration will be increased to a comparable value, if the bladder wall is intact.

The serum electrolytes and Na : K ratio of less than 20 are key to consider this differential diagnosis. In dogs the fol-lowing are the four most likely differentials and the tests to rule in or rule out each differential for this electrolyte pattern:

Summary and comment : Uroabdomen

There was a hole in the dorsal aspect of the urinary bladder. It was surgically repaired and the bladder wall appeared healthy at surgery. The foal recovered.

Rupture of the urinary bladder is usually caused by an obstruction (calculi) in males or excessive trauma (hit by car) in all species except horses. Typically the problem is seen in male foals that appeared fi ne at birth but gradually develop anorexia and other problems. The male urethral lumen is small and apparently does not allow expulsion of urine easily enough during birth, and the back pressure caused by the mare ’ s strong contractions during parturition causes the bladder to rupture. There are no calculi obstruct-ing outfl ow. Usually there is a history of dribbling urine or that the foal was seen to urinate. The hole is almost always located dorsally due to the musculature of the bladder wall and therefore patients may still urinate. If contrast dyes are used to determine bladder integrity the dye may still be retained for the same reason. Comparison of abdominal Ct concentration to serum Ct is the diagnostic test of choice. Abdominal Ct does not have to be twice as great as serum Ct to rule in a ruptured bladder, it just needs to be several

Diagnosis Test of choice

Hypoadrenocorticism Basal cortisol; ACTH stim

Renal failure Fluid therapy; complete UA;

fractional excretion of Na

Uroabdomen Compare abdominal and serum Ct

concentrations

GI – whipworms, Salmonella Fecal fl oatation; fecal culture

Page 59: VI Clinical Case Presentations - Wiley · Clinical Case Presentations ... Case 17: Dog with vitamin K antagonist (rodenticide) toxicity, 643 ... PT Prothrombin time

Case 33

673

CA

SES

Interpretive discussion

There is moderate hyperglycemia that exceeds the renal glucose threshold. There is resultant glucosuria. Ketonuria indicates impaired glucose utilization by tissues suggesting insulin defi ciency. This establishes a working diagnosis of diabetes mellitus.

There is marked hypokalemia in the face of acidosis (see blood gas discussion), which would be expected to increase potassium due to an intracellular to extracellular shift. It would be suspected that total body potassium is depleted and there should be caution in insulin administration that would drive potassium into cells, resulting in weakness due to even more severe hypokalemia. There is also hyponatre-mia and disproportionate hypochloremia. The hyponatre-mia may be due to a combination of urinary and gastrointestinal loss. The disproportionate hypochloremia is of a magnitude that suggests upper gastrointestinal chloride loss due to vomiting.

The acid - base data show a low pH and decreased bicar-bonate, consistent with a metabolic acidosis. The decreased pCO 2 is a compensatory response (respiratory alkalosis). There is also a likely hidden component of metabolic alkalosis (hypochloremic alkalosis), resulting in a mixed acid - base disorder. The loss of HCl in gastric fl uid will meta-bolically generate bicarbonate. The generation of ketoacids that require buffering by bicarbonate is judged to be more severe in the balance between bicarbonate utilization and production.

Summary

Diabetic ketoacidosis pattern, largely compensated.

Signalment 10 - year - old castrated male domestic short hair cat History Anorexia, lethargy Physical examination Dehydrated

Miscellaneous tests Reference Interval

Plasma glucose (mg/dL) 328 67 – 124

Urinalysis dipstick

abnormalities

glucosuria and ketonuria

Na (mEq/L) 130 146 – 160

K (mEq/L) 2.2 3.7 – 5.4

CL (mEq/L) 74 112 – 129

pH 7.28 7.33 – 7.44

HCO 3 (mEq/L) 9.2 16 – 20

pCO 2 (mmHg) 20 28 – 34

Case 33

Page 60: VI Clinical Case Presentations - Wiley · Clinical Case Presentations ... Case 17: Dog with vitamin K antagonist (rodenticide) toxicity, 643 ... PT Prothrombin time

SECTION VI Clinical Case Presentations

674

CA

SES

Case 34 Signalment: 5 - year - old spayed female Manx cat History: Decreased appetite of approximately 2 weeks ’ duration. Fluid draining from a fi stula over the terminal coccygeal vertebrae of 8 months ’ duration. Physical examination: Approximately 6% dehydrated. Coccygeal vertebrae were noted to terminate cranial to the anal sphincter. The fi stula was noninfl amed and draining a clear, colorless fl uid.

Hematology Reference Interval

PCV (%) 49 24 – 45

NCC ( × 10 3 / μ L) 11.6 5.5 – 19.5

Segs ( × 10 3 / μ L) 9.6 2.5 – 12.5

Monos ( × 10 3 / μ L) 0.6 0 – 0.8

Lymphs ( × 10 3 / μ L) 1.4 1.5 – 7.0

Platelets ( × 10 3 / μ L) Adequate 200 – 500

Biochemical Profi le

Reference Interval

Gluc (mg/dL) 91 67 – 124

BUN (mg/dL) 82 17 – 32

Creat (mg/dL) 2.2 0.9 – 2.1

Ca (mg/dL) 7.3 8.5 – 11

Phos (mg/dL) 5.2 3.3 – 7.8

TP (g/dL) 8.4 5.9 – 8.1

Alb (g/dL) 4.1 2.3 – 3.9

Glob (g/dL) 4.3 2.9 – 4.4

T. Bili (mg/dL) 0.1 0 – 0.3

Chol (mg/dL) 153 60 – 220

ALT (IU/L) 40 30 – 100

Na (mEq/L) 131 146 – 160

K (mEq/L) 4.6 3.7 – 5.4

CL (mEq/L) 101 112 – 129

TCO 2 (mEq/L) 16 14 – 23

An. gap (mEq/L) 18.6 10 – 27

Urinalysis

Color Straw

Transparency Clear

Sp. Gr. 1.015

Protein Negative

Gluc Negative

Bilirubin Negative

Blood Negative

pH 6.5

Fractional excretion

Reference Interval

Na (%) 0.03 < 1.0

CL (%) 0.08 < 1.0

Page 61: VI Clinical Case Presentations - Wiley · Clinical Case Presentations ... Case 17: Dog with vitamin K antagonist (rodenticide) toxicity, 643 ... PT Prothrombin time

Case 34

675

CA

SES

Interpretive discussion

Hematology The increased PCV is consistent with hemoconcentration due to dehydration. Other data are unremarkable.

Biochemical profi le The azotemia (increased BUN and serum creatinine concen-trations) may be prerenal and/or renal. Refer to the discus-sion of urinalysis below.

There is hypocalcemia, despite hyperproteinemia due to hemoconcentration, suggesting that serum calcium concen-tration may be truly decreased. An ionized calcium could be measured for confi rmation. Hypocalcemia could have occurred secondary to chloride depletion and loss of the electrochemical gradient needed to support calcium absorp-tion from the glomerular fi ltrate in the Loop of Henle of the renal tubules.

Serum sodium and chloride are decreased in concentra-tion. This usually refl ects increased loss, compounded by reduced intake in sick, anorexic patients. In this case, there is physical evidence of cerebrospinal fl uid loss from a drain-ing meningomyelocele. Cerebrospinal fl uid contains higher sodium and chloride concentrations than the blood, owing to the active chloride transport mechanism employed by the choroid plexus for secretion. Draining CSF from the body will create electrolyte depletion in excess of water, an oth-

erwise classic scenario for hypotonic dehydration. Although this cat had been losing CSF for some time, the development of anorexia probably precipitated an imbalance between these pathologic losses and replacement of the electrolytes, resulting in the clinical presentation.

Urinalysis The urinary fractional excretion values for sodium and chlo-ride were well within the normal reference interval, thereby ruling out renal loss as a cause for the electrolyte depletion. The only signifi cant abnormality is a urine specifi c gravity of 1.015. Dehydration should stimulate antidiuretic hormone release from the hypothalamus, and increased water recla-mation by the renal tubules. However, electrolyte loss in this type of hypotonic dehydration often leads to medullary solute depletion and a loss of the renal concentration gradi-ent. Another alternative is that there is renal disease, due to renal hypoperfusion, sepsis, etc., resulting in both azotemia and loss of concentrating ability.

Summary

Sodium chloride depletion in a manx cat with a fi stulated meningomyelocele (Hall JA, MJ Fettman, JT Ingram. Sodium chloride depletion in a cat with fi stulated meningomyelo-cele. J Am Vet Med Assoc 1988;192:1445 – 1448).

Page 62: VI Clinical Case Presentations - Wiley · Clinical Case Presentations ... Case 17: Dog with vitamin K antagonist (rodenticide) toxicity, 643 ... PT Prothrombin time

SECTION VI Clinical Case Presentations

676

CA

SES

rate. Creatinine is a more sensitive marker for decreased GFR in horses because of their ability to excrete urea into the GI tract. A urinalysis might help differentiate prerenal from renal azotemia, but the low chloride (discussed later) might affect urinary concentrating ability. There may be a prerenal component (dehydration) to the azotemia since the PCV is at the top of the reference interval. However, the normal albumin does not support dehydration, and there does not appear to be much protein loss into the abdominal effusion (which would lower the serum albumin).

There is a slight hypocalcemia, which may be due to deposition in injured tissue.

Increased CK activity indicates muscle cell damage. The increased SDH activity is specifi c for hepatocellular injury. The slight increase in AST activity could be from muscle or hepatocellular injury.

There is a selective hypochloremia, indicating a hypochlo-remic alkalosis. Selective chloride loss can result from upper GI loss or sequestration of CL and excessive sweating in horses. In addition, the decreased TCO 2 and increased anion gap indicates a high - gap metabolic acidosis; thus there is a mixed acid - base disorder. Uremic acids and lactic acid, sec-ondary to hypoperfusion, are likely contributors to the anion gap. A blood gas profi le is needed to fully assess acid-base status in this horse.

Abdominal fl uid analysis Although the quantitative indices are all within normal limits, the presence of degenerate infl ammatory cells, bac-teria, and debris are all consistent with an acute rupture of the intestinal tract.

Summary

This mare experienced intestinal colic, followed by acute rupture of the involved strangulated intestine. There had not been time for an infl ammatory leukogram to develop.

Case 35 Signalment: 10 - year - old female horse History: Abdominal pain Physical examination: Tense abdomen, slight fever

Hematology Reference Interval

PCV (%) 52.0 32 – 52 Hgb (g/dL) 18.1 11 – 19 RBC ( × 10 6 / μ L) 11.15 6.5 – 12.5 MCV (fL) 46.0 36 – 52 MCHC (g/dL) 34.0 34 – 39 NCC ( × 10 3 / μ L) 14.2 5.5 – 12.5 Segs ( × 10 3 / μ L) 11.8 2.7 – 6.7 Monos ( × 10 3 / μ L) 0.3 0 – 0.8 Lymphs ( × 10 3 / μ L) 2.1 1.5 – 5.5 Platelets ( × 10 3 / μ L) 162 100 – 600 TP (P) (g/dL) 7.0 6 – 8 Fibrinogen (mg/dL) 200 100 – 400

Biochemical Profi le

Reference Interval

Gluc (mg/dL) 166 70 – 110 BUN (mg/dL) 23 14 – 27 Creat (mg/dL) 4.2 1.1 – 2.0 Ca (mg/dL) 10.5 11.0 – 13.7 Phos (mg/dL) 4.5 1.9 – 4.1 TP (g/dL) 7.1 5.8 – 7.6 Alb (g/dL) 3.2 2.7 – 3.7 Glob (g/dL) 3.9 2.6 – 4.6 T. Bili (mg/dL) 1.4 0.6 – 2.1 AST (IU/L) 430 185 – 300 GGT (IU/L) 8 7 – 17 SDH (IU/L) 99 0 – 9 CK (IU/L) 8422 130 – 470 Na (mEq/L) 140 133 – 145 K (mEq/L) 3.5 2.2 – 4.6 CL (mEq/L) 86 100 – 111 TCO 2 (mEq/L) 22.6 24 – 34 An. gap 35 5 – 15

Abdominal Fluid Analysis

Fluid color Straw

Fluid clarity Hazy

Supernatant color Straw

Supernatant clarity Clear

TP (g/dL) 1.3

NCC (/ μ L) 300

Cytology: There are approximately equal numbers of neutrophils and

large mononuclear cells. Although the overall cellularity and protein

are low, some of the neutrophils are degenerate and bacteria are

seen extracellularly, predominantly rods. Some of the macrophages

and neutrophils contain cytoplasmic material suggestive of bacterial

remnants. There are moderate numbers of lymphocytes and rare mast

cells seen. There is debris present in the background.

Interpretive discussion

Hematology There is a neutrophilic leukocytosis with low normal lym-phocyte numbers, which most likely refl ects stress, rather than infl ammation. The fi brinogen is within normal limits. The PCV is at the top of the reference interval and serum proteins are normal, suggesting possible splenic contraction.

Biochemical profi le There is a mild hyperglycemia, which is consistent with stress. The increases in serum creatinine and serum phos-phorus are likely the result of decreased glomerular fi ltration

Page 63: VI Clinical Case Presentations - Wiley · Clinical Case Presentations ... Case 17: Dog with vitamin K antagonist (rodenticide) toxicity, 643 ... PT Prothrombin time

Case 36

677

CA

SES

Case 36 Signalment: 11 - year - old intact male miniature schnauzer History: Intermittent vomiting and diarrhea for last two weeks Physical examination: Tense, painful abdomen. Very fat.

Hematology Reference Interval

PCV (%) 38 37 – 55

Hgb (g/dL) 13.2 12 – 18

RBC ( × 10 6 / μ L) 5.7 5.5 – 8.5

MCV (fL) 67 60 – 72

MCHC (g/dL) 35 33 – 38

NCC ( × 10 3 / μ L) 17.9 6 – 17

Segs ( × 10 3 / μ L) 14.2 3 – 11.5

Bands ( × 10 3 / μ L) 0.5 0 – 0.3

Monos ( × 10 3 / μ L) 0.7 0.1 – 1.3

Lymphs ( × 10 3 / μ L) 2.5 1 – 4.8

Platelets ( × 10 3 / μ L) 250 200 – 500

TP (P) (g/dL) 9.0 6 – 8

Hemopathology: Moderate polychromasia

Biochemical Profi le (serum was lipemic)

Reference Interval

Gluc (mg/dL) 124 (6.8) 65 – 122 (3.5 – 6.7 mmol/L)

BUN (mg/dL) 42 (15) 7 – 28 (2.5 – 10.0 mmol/L)

Creat (mg/dL) 1.2 0.9 – 1.7

Ca (mg/dL) 9.8 9.0 – 11.2

Phos (mg/dL) 5.8 2.8 – 6.1

TP (g/dL) 7.7 5.4 – 7.4

Alb (g/dL) 3.7 2.7 – 4.5

Glob (g/dL) 4.0 1.9 – 3.4

T. Bili (mg/dL) 10.8 (184.7) 0 – 0.4 (0 – 6.8 μ mol/L)

Chol (mg/dL) 1230 (32) 130 – 370 (3.4 – 9.6 mmol/L)

ALT (IU/L) 600 10 – 120

AST (IU/L) 540 16 – 40

ALP (IU/L) 660 35 – 280

Na (mEq/L) 148 145 – 158

K (mEq/L) 4.3 4.1 – 5.5

CL (mEq/L) 110 106 – 127

TCO 2 (mEq/L) 24 14 – 27

An. gap (mEq/L) 18 8 – 25

Amylase (IU/L) 510 50 – 1250

Lipase (IU/L) 120 30 – 560

Urinalysis (voided)

Color Yellow Urine Sediment

Transparency Cloudy WBCs/hpf > 50

Sp. Gr. 1.022 RBCs/hpf 0 – 1

Protein 3 + Epith cells/hpf 0

Gluc Negative Casts/lpf 0

Bilirubin 2 + Crystals 0

Blood Negative Bacteria Many bacilli

pH 7.0

Page 64: VI Clinical Case Presentations - Wiley · Clinical Case Presentations ... Case 17: Dog with vitamin K antagonist (rodenticide) toxicity, 643 ... PT Prothrombin time

SECTION VI Clinical Case Presentations

678

CA

SES

Interpretive discussion

Hematology The PCV, hemoglobin concentration, and RBC count are at the lower end of the reference interval, and it is possible that an anemia has been masked by dehydration. With a history of vomiting and diarrhea and an increased plasma protein concentration, it is possible that this animal is dehydrated. However, we have no additional support for dehydration in these data. Because the serum is lipemic, a likely explana-tion for the high total plasma protein as measured by refrac-tometry is the presence of lipids, which interfere with the reading. The presence of moderate polychromasia suggests a regenerative response. Blood loss may have caused a regenerative anemia in this dog (see summary).

Neutrophilia with a left shift indicates an infl ammatory leukogram.

Biochemical profi le The BUN concentration is only mildly increased and the serum creatinine concentration is normal. Urine specifi c gravity indicates inadequate urine concentrating ability in the face of possible dehydration (if present) and azotemia, suggesting possible renal disease. Increased BUN with normal creatinine also prompts consideration of prerenal azotemia secondary to GI tract hemorrhage (which acts as a high protein meal).

Both plasma and serum protein concentrations are increased, but the plasma protein concentration is much higher than the serum protein concentration. Because fi brinogen is present in plasma but not in serum, one would expect the plasma protein concentration to be 0.2 to 0.4 g/dL higher than that of serum. However, the difference is often greater because plasma protein concentration is estimated using a refractometer, while serum protein is mea-sured spectrophotometrically. Increased plasma concentra-tion of lipids may falsely increase the protein estimate determined by a refractometer. The increased difference between these protein concentrations in this case is likely due to lipemia.

The hyperglobulinemia may be the result of chronic anti-genic stimulation with subsequent increase in antibody production.

The combination of hyperbilirubinemia and increased serum ALP activity is typical of cholestasis. The increased bilirubin concentration in the urine refl ects the hyperbiliru-binemia; conjugated bilirubin is cleared by the glomerulus and excreted in the urine. Although hypercholesterolemia is a nonspecifi c problem, cholestasis is a common cause of this abnormality and may be an explanation in this case. The magnitude of the hypercholesterolemia is unusual for cholestasis alone. Since this dog is a miniature schnauzer, and the serum is lipemic, suggesting hypertriglyceridemia and/or chylomicronemia, idiopathic hyperlipidemia is likely.

Increased serum AST and ALT activities indicate hepato-cyte injury. Both of these enzymes are leakage enzymes and are present in signifi cant concentrations in hepatocytes. AST is also present in high concentrations and ALT in low con-centrations in muscle, but muscle is an unlikely source of these enzymes in this case. In light of the evidence for cho-lestasis, hepatic origin is most likely for these enzymes in this dog.

Urinalysis Proteinuria, pyuria and bacteruria suggest infl ammation in the urinary tract. Since these are found in a voided urine sample, reproductive tract origin must also be considered. Bacteria in a voided urine sample may be contaminants but are more signifi cant when accompanied by pyuria. Culture of this urine sample is indicated.

Summary

This dog had a suppurative cholangiohepatitis, a duodenal ulcer, and pyelonephritis. The cholangiohepatitis resulted in the cholestasis and damage to hepatocytes. The chronic anti-genic stimulation caused by both cholangiohepatitis and pyelonephritis resulted in hyperglobulinemia. The mild azo-temia could have resulted from pyelonephritis or GI hemor-rhage secondary to the duodenal ulcer.

Page 65: VI Clinical Case Presentations - Wiley · Clinical Case Presentations ... Case 17: Dog with vitamin K antagonist (rodenticide) toxicity, 643 ... PT Prothrombin time

Case 37

679

CA

SES

Interpretive discussion

Hematology There is a moderate leukocytosis characterized by a mature neutrophilia, lymphopenia, and monocytosis. This is most consistent with a steroid/stress leukogram, which is expected given the history of prednisone administration. The magni-tude of the mature neutrophilia is slightly greater than that usually seen with a stress response alone, so chronic infl am-mation should also be considered.

Although there is no anemia present there are signifi cant erythrocyte membrane changes observed. Acanthocytes may be seen with liver disease and hemangiosarcoma. Kera-tocytes and schistocytes suggest there is some red cell frag-mentation occurring.

Biochemical profi le The most signifi cant abnormalities relate to the liver. Increased activities of the leakage enzymes ALT and AST indicate there has been hepatocellular injury. Increased activities of the inducible enzymes ALP and GGT may be due to cholestasis and/or drug induction, given the history of steroid administration. The increased total bilirubin concen-tration supports cholestasis, since there is no indication of hemolysis, and indicates that hepatic function has been compromised.

Albumin is slightly decreased, most likely due to the ongoing infl ammatory bowel disease. Decreased production due to liver failure is less likely since glucose, BUN, and cholesterol are normal. The mild decrease in calcium is likely due to a decrease in protein - bound calcium secondary to hypoalbuminemia. An ionizd calcium could be measured to confi rm this.

Summary

There is laboratory evidence for hepatocellular injury and cholestasis severe enough to affect hepatic function. Given the history of prednisone administration, a steroid hepa-topathy is most likely. Red cell membrane changes may be due to hepatic disease or potentially hemangiosarcoma. A liver aspirate was performed and revealed marked indistinct hepatic vacuolization consistent with glycogen, and numer-ous bile casts indicating cholestasis. These fi ndings were confi rmed on a liver biopsy; there was no evidence of neo-plasia or infl ammation. Biopsies of the skin lesions revealed pyogranulomatous dermatitis with intralesional pigmented fungal hyphae. Presumably immune suppression due to the prolonged steroid administration predisposed to the fungal infection.

Hematology Reference Interval

PCV (%) 38 36 – 60

Hgb (g/dL) 13.1 12 – 18

RBC ( × 10 6 / μ L) 4.9 4.8 – 9.3

MCV (fL) 79 58 – 79

MCHC (g/dL) 34 33 – 38

NCC ( × 10 3 / μ L) 27.8 4 – 15.5

Segs ( × 10 3 / μ L) 25.5 2 – 10.5

Bands ( × 10 3 / μ L) 0 0 – 0.3

Monos ( × 10 3 / μ L) 2.0 0 – 0.9

Lymphs ( × 10 3 / μ L) 0.3 1 – 4.5

Eos ( × 10 3 / μ L) 0 0.1 – 1.2

Platelets ( × 10 3 / μ L) 374 200 – 500

Hemopathology: Moderate acanthocytes, few echinocytes,

keratocytes, schistocytes.

Case 37 Signalment: Six - year - old CM German shepherd History: Receiving prednisone for infl ammatory bowel disease. Losing weight for 6 months. Physical examination: Thin, with multiple hairless scaly skin lesions. Hepatomegaly.

Biochemical Profi le Reference Interval

Gluc (mg/dL) 103 70 – 138

BUN (mg/dL) 11 6 – 25

Creat (mg/dL) 0.5 0.5 – 1.6

Ca (mg/dL) 8.4 8.9 – 11.4

Phos (mg/dL) 4.4 2.5 – 6.0

TP (g/dL) 5.3 5.0 – 7.4

Alb (g/dL) 2.3 2.7 – 4.4

Glob (g/dL) 3.0 1.6 – 3.6

T. Bili (mg/dL) 1.6 0.1 – 0.3

Chol (mg/dL) 121 92 – 324

Trig (mg/dL) 102 29 – 291

ALT (IU/L) 1041 12 – 128

AST (IU/L) 101 15 – 66

ALP (IU/L) 640 5 – 131

GGT (IU/L) 237 1 – 12

CK (IU/L) 174 59 – 895

Na (mEq/L) 149 139 – 154

K (mEq/L) 4.9 3.6 – 5.5

CL (mEq/L) 108 102 – 120

TCO 2 (mEq/L) 23 15 – 25

Coagulation Profi le Reference Interval

PT (seconds) 7.0 6 – 12

aPTT (seconds) 15.9 10 – 25

Page 66: VI Clinical Case Presentations - Wiley · Clinical Case Presentations ... Case 17: Dog with vitamin K antagonist (rodenticide) toxicity, 643 ... PT Prothrombin time

SECTION VI Clinical Case Presentations

680

CA

SES

Case 38 Signalment: 5 - year - old FS canine History: On phenobarbital to control seizures for 2.5 years. Vomiting daily and lethargic for about 1 month. Physical examination: Lethargic, icteric, pendulous abdomen; arthritic and appears older than stated age

Hematology Reference Interval

PCV (%) 40.0 37 – 55

Hgb (g/dL) 13.6 12 – 18

RBC ( × 10 6 / μ L) 5.53 5.5 – 8.5

MCV (fL) 72.0 60 – 72

MCHC (g/dL) 34.0 33 – 38

NCC ( × 10 3 / μ L) 47.2 6 – 17

Segs ( × 10 3 / μ L) 40.1 3 – 11.5

Bands ( × 10 3 / μ L) 0.9 0 – 0.3

Monos ( × 10 3 / μ L) 4.7 0.1 – 1.3

Lymphs ( × 10 3 / μ L) 0.9 1 – 4.8

Eos ( × 10 3 / μ L) 0.5 0.1 – 1.2

Platelets ( × 10 3 / μ L) 299 200 – 500

TP (P) (g/dL) 5.5 6 – 8

Hemopathology: slt toxic neutrophils, many echinocytes

Biochemical Profi le

Reference Interval

Gluc (mg/dL) 69 65 – 122

BUN (mg/dL) 5 (1.78) 7 – 28 (2.5 – 10.0 mmo1/L)

Creat (mg/d1) 0.6 0.9 – 1.7

Ca (mg/dL) 8.1 (2.02) 9.0 – 11.2 (2.25 – 2.80 mmol/L)

Phos (mg/dL) 5.1 2.8 – 6.1

TP (g/dL) 4.8 5.4 – 7.4

Alb (g/dL) 2.0 2.7 – 4.5

Glob (g/dL) 2.8 1.9 – 3.4

T. Bili (mg/dL) 4.5 (77) 0 – 0.4 (0 – 6.8 μ mol/L)

Chol (mg/dL) 126 (3.28) 130 – 370 (3.4 – 9.6 mmol/L)

ALT (IU/L) 348 10 – 120

AST (IU/L) 176 16 – 40

ALP (IU/L) 4503 35 – 280

GGT (IU/L) 426 0 – 6

Na (mEq/L) 142 145 – 158

K (mEq/L) 3.3 4.1 – 5.5

CL (mEq/L) 114 106 – 127

TCO 2 (mEq/L) 14.8 14 – 27

An. gap (mEq/L) 16.5 8 – 25

Lipase (IU/L) 575 30 – 560

Urinalysis

Color Orange Urine Sediment

Transparency Cloudy WBCs/hpf 8 – 10

Sp. Gr. 1.015 RBCs/hpf 0 – 2

Protein 2 + Epith cells/hpf 80 – 100

Gluc Negative Casts/lpf Negative

Bilirubin 4 + Crystals Negative

Blood 3 + Bacteria 4 + rods

pH 6.0

Ketones 3 +

Coagulation Data

Reference Interval

PT (seconds) 9.8 7.5 – 10.5

aPTT (seconds) 14.0 10.5 – 16.5

Interpretive discussion

Hematology There is a moderate neutrophilia with a mild left shift, monocytosis, and slightly toxic neutrophils were observed in the blood fi lm. This is an infl ammatory leukogram, but the lymphopenia indicates a concurrent steroid - induced component. Monocytosis is consistent with the combined leukocyte response.

Biochemical profi le The serum glucose concentration is at the low end of the reference interval and the BUN is decreased. These fi ndings may indicate hepatic functional impairment, particularly in light of the observation of a potential stress leukogram (stress would be expected to increase the glucose concentra-tion). See discussion of serum protein below. Decreased cre-atinine likely refl ects decreased muscle mass.

The serum total protein and albumin concentrations are decreased. Considerations in this case include renal loss (see urinalysis discussion) and, more likely, decreased production secondary to hepatic disease (discussed more later).

There is a mild hypocalcemia that is likely secondary to hypoalbuminemia, and therefore not clinically signifi cant. An ionized calcium could be measured to confi rm this.

Serum cholesterol is decreased. While one should not overinterpret decreases in some analytes, this is commonly observed in end - stage liver disease, owing to impaired hepatic lipid synthesis. This is particularly notable given the degree of hyperbilirubinemia and increases in enzyme activ-

Page 67: VI Clinical Case Presentations - Wiley · Clinical Case Presentations ... Case 17: Dog with vitamin K antagonist (rodenticide) toxicity, 643 ... PT Prothrombin time

Case 38

681

CA

SES

by the urinary tract infl ammation/infection as indicated by the signifi cant pyuria, bacteriuria, and presence of marked occult blood. The presence of bilirubin is not surprising given the degree of hyperbilirubinemia. The presence of ketonuria in the absence of glucosuria is unusual. Ketosis is a possible sequela to impaired oxidative lipid metabolism by the dis-eased liver, especially when triglyceride absorption from the GI tract or mobilization from peripheral stores is greater than hepatic functional capacity for processing.

Summary

There is biochemical evidence of chronic liver failure, cho-lestasis, and hepatocellular injury. Phenobarbital - induced hepatopathy was considered. Ultrasound of the liver showed an enlarged liver with numerous well - defi ned hypoechoic foci throughout. Masses throughout the cranial mid - abdomen had similar echogenicity as masses within liver. Cytology of a liver aspirate showed vacuolated hepatocytes, bile stasis, and a population of nonhepatic cells with a high nucleus : cytoplasm ratio, most of which were broken. Numerous cells in mitosis were observed, and neoplasia was diagnosed. Biopsy of liver revealed adenocarcinoma which effaced and replaced hepatic parenchyma, and glucocorti-coid hepatopathy with severe bile stasis. The neoplasm had a neuroendocrine (potentially adrenal) pattern, and was possibly causing the steroid hepatopathy. Endocrine panel was not performed. Dog was euthanized; necropsy was not allowed.

ities indicative of cholestasis (ALP and GGT). The magnitude of increase in serum ALP activity is large enough to warrant consideration of corticosteroid induction. Likewise, the degree of increase in GGT activity may be related to steroid induction rather than cholestasis alone. The serum ALT and AST activities are moderately increased, indicating hepato-cellular damage. Phenobarbital may induce increased pro-duction of several liver enzymes.

The serum sodium and potassium are decreased, and one should consider typical causes for electrolyte depletion, including pathologic losses from the gastrointestinal and urinary systems, as well as third space syndromes. Hypoka-lemia is a frequent observation in hepatic disease, often due to anorexia and vomiting.

Coagulation data The APTT and PT are normal. If hepatic disease or end - stage liver failure has progressed suffi ciently, as suggested by even lower values for glucose, BUN, albumin, and cholesterol, one might expect these indices of coagulation factor synthe-sis to become abnormal as well.

Urinalysis The urinary specifi c gravity indicates the urine is poorly concentrated and may refl ect impaired concentrating ability. This may be due to the decreased BUN, since urea also plays a role in urine concentration. The concentrations of protein, ketones, bilirubin, and blood are particularly notable given this weak urine concentration. The proteinuria is explained

Page 68: VI Clinical Case Presentations - Wiley · Clinical Case Presentations ... Case 17: Dog with vitamin K antagonist (rodenticide) toxicity, 643 ... PT Prothrombin time

SECTION VI Clinical Case Presentations

682

CA

SES

Case 39 Signalment: 6 - year - old spayed female dog History: Struck by car 1 month ago. Not taken to veterinarian. Dyspnea since accident. Anorexia. Physical examination: Emaciated and lethargic. Intes tinal sounds auscultated in thorax.

Hematology Reference Interval

PCV (%) 37 37 – 55

Hgb (g/dL) 12.3 12 – 18

RBC ( × 10 6 / μ L) 6.1 5.5 – 8.5

MCV (fL) 61 60 – 72

MCHC (g/dL) 33 33 – 38

NCC ( × 10 3 / μ L) 16.1 6 – 17

Segs ( × 10 3 / μ L) 13.5 3 – 11.5

Bands ( × 10 3 / μ L) 0.2 0 – 0.3

Monos ( × 10 3 / μ L) 1.0 0.1 – 1.3

Lymphs ( × 10 3 / μ L) 0.6 1 – 4.8

Eos ( × 10 3 / μ L) 0.8 0.1 – 1.2

Platelets ( × 10 3 / μ L) 330 200 – 500

TP (P) (g/dL) 3.3 6 – 8

Hemopathology: Normal

Biochemical Profi le Reference Interval

Gluc (mg/dL) 77 65 – 122

BUN (mg/dL) 3 (1.07) 7 – 28 (2.5 – 10.0 mmol/L)

Creat (mg/dL) 1.5 0.9 – 1.7

Ca (mg/dL) 6.3 9.0 – 11.2

Phos (mg/dL) 4.4 2.8 – 6.1

TP (g/dL) 2.9 5.4 – 7.4

Alb (g/dL) 0.6 2.7 – 4.5

Glob (g/dL) 2.3 1.9 – 3.4

T. Bili (mg/dL) 3.0 (51.3) 0 – 0.4 (0 – 6.8 μ mol/L)

Chol (mg/dL) 102 (2.65) 130 – 370 (3.4 – 9.6 mmol/L)

ALT (IU/L) 170 10 – 120

AST (IU/L) 72 16 – 40

ALP (IU/L) 540 35 – 280

Na (mEq/L) 146 145 – 158

K (mEq/L) 6.0 4.1 – 5.5

CL (mEq/L) 118 106 – 127

TCO 2 (mEq/L) 11 14 – 27

An. gap (mEq/L) 23 8 – 25

Plasma ammonia

(mg/dL)

150 0 – 90

Urinalysis (catheterized)

Color Yellow Urine Sediment

Transparency Clear WBCs/hpf 0

Sp. Gr. 1.035 RBCs/hpf 0

Protein Negative Epith cells/hpf 0

Gluc Negative Casts/lpf 0

Bilirubin 1 + Crystals Bilirubin

Blood Negative Bacteria 0

pH 5.5

Interpretive discussion

Hematology This dog ’ s erythrocyte measurements are near the lower end of their reference intervals, and there is no evidence of a regenerative response. This may be normal for this dog, or it is possible that this dog is developing a nonregenerative anemia secondary to chronic disease.

Leukocyte abnormalities are a mature neutrophilia and lymphopenia, typical of a corticosteroid - mediated leuko-gram.

Biochemical profi le Decreased BUN concentration can be caused by hepatic failure, diuresis, decreased protein intake, or treatment with anabolic steroids. BUN concentration below the reference interval can also occur in normal animals. In light of other laboratory fi ndings in this case, the decreased BUN concen-tration is probably due to hepatic failure and resulting failure of hepatocytes to synthesize urea. Anorexia resulting in decreased protein intake may have also contributed to this abnormality.

This dog has hypocalcemia; however, it also has severe hypoalbuminemia. Hypoalbuminemia, and resultant decreased protein - bound calcium, may be the cause for the hypocalcemia, in which case it is not clinically signifi cant. Ionized calcium could be measured to confi rm this. While not always necessary, determining the ionized calcium would be recommended in this case because of the magni-tude of the hypocalcemia and the critical condition of the patient.

Both plasma and serum protein concentrations are decreased. These decreases are a result of hypoalbuminemia. When interpreted in combination with other laboratory data, this abnormality is probably due to decreased albumin synthesis by the liver. Decreased protein intake can result in hypoalbuminemia and may also have played a role in this case. The albumin concentration is low enough to lead to ascites; however, ascites was not noted in this animal.

Page 69: VI Clinical Case Presentations - Wiley · Clinical Case Presentations ... Case 17: Dog with vitamin K antagonist (rodenticide) toxicity, 643 ... PT Prothrombin time

Case 39

683

CA

SES

markedly low albumin may be masking an increase in unmeasured anions, thus there could be a hidden high-gap acidosis. Abnormal renal regulation of acid - base balance is another possible cause of decreased total CO 2 , but there is no evidence of renal dysfunction in this case. A blood gas analysis would be helpful to further evaluate this dog ’ s acid - base status.

The hyperammonemia is a result of hepatic failure. Ammonia is normally absorbed from the digestive tract and transported to the liver by the portal circulation. The liver is responsible for removing and metabolizing this ammonia. Alterations in blood fl ow to the liver and/or markedly decreased numbers of functional hepatocytes can result in increased blood ammonia concentrations.

Urinalysis Bilirubinuria and the presence of bilirubin crystals are the only abnormalities in the urine. These changes refl ect the increased serum bilirubin concentration. Conjugated biliru-bin readily passes through glomeruli and is then excreted in the urine. The very mild increase in urine bilirubin suggests that most of the serum bilirubin is unconjugated. Interest-ingly, this dog is concentrating urine in the face of a very low bun.

Summary

Exploratory surgery revealed a diaphragmatic hernia through which the liver and a portion of the GI tract had passed. The liver was decreased in size and fi rm. Many fi brous adhesions were present. On the surgeon ’ s recom-mendation, the dog was euthanized.

This dog had hepatic failure due to chronically decreased blood supply to the liver. Decreased BUN, albumin, and cholesterol concentrations suggested decreased synthetic function by the liver. Increased bilirubin and ammonia con-centrations resulted from decreased delivery of these sub-stances to the liver and, therefore, decreased removal from the blood as well as due to decreased functional hepatic mass. Cholestasis resulting from partial occlusion of the bile duct also contributed to hyperbilirubinemia. Since this was an end - stage liver disease, leakage of ALT and AST from hepatocytes was minimal due to the small number of hepa-tocytes remaining, and serum activities of ALT and AST were, therefore, only slightly increased.

In dogs, hyperbilirubinemia can result from hemolysis, failure of hepatocyte uptake and metabolism of bilirubin, or failure to excrete bilirubin due to cholestasis or other disrup-tion of bile fl ow. In this case, failure of hepatic uptake and metabolism of bilirubin is probably the major abnormality leading to hyperbilirubinemia. It is also probable the bile duct is partially blocked and cholestasis is playing a role in producing this abnormality. The increased serum ALP activ-ity suggests cholestasis is present in this dog.

Hypocholesterolemia is probably another result of hepatic failure. The liver is a major site of cholesterol synthesis and excretion. Abnormalities of these two processes have oppo-site effects on serum cholesterol concentrations. In this case, synthetic failure is apparently more severe than failure to excrete cholesterol.

Both serum ALT and AST activities are mildly increased. These enzymes leak from injured hepatocytes, and liver injury is the appropriate interpretation in this case. AST is also present in muscle cells, and muscle injury cannot be ruled out, but the mild increase of AST activity in conjunc-tion with the increased ALT activity suggests the AST has leaked from the liver in this case.

Increased ALP activity most often results from either cho-lestasis or increased blood corticosteroid concentrations. In combination with other laboratory data suggesting hepatic disease, cholestasis is the most important cause of the increased ALP in this case. This dog probably had an increased blood corticosteroid concentration as suggested by the leukogram, and this may have also played a role in increasing the serum ALP activity.

Hyperkalemia may be a result of metabolic acidosis - induced shifting of potassium from within cells to extracel-lular fl uid. In animals with metabolic acidosis, hydrogen ions enter cells in an attempt to equalize their concentra-tions in the intracellular and extracellular compartments. In order to maintain electrical neutrality, potassium ions must leave the cells. The net result is increased extracellular and, therefore, serum potassium concentrations.

The cause of the decreased total CO 2 is not certain. Since this animal has a compromised respiratory system, it is rea-sonable to assume that it has a respiratory acidosis. However, the total CO 2 concentration would be expected to increase in compensation for the respiratory acidosis. Since this concentration decreased rather than increased, it is reason-able to assume the dog has another abnormality causing metabolic acidosis. Although the anion gap is normal, the

Page 70: VI Clinical Case Presentations - Wiley · Clinical Case Presentations ... Case 17: Dog with vitamin K antagonist (rodenticide) toxicity, 643 ... PT Prothrombin time

SECTION VI Clinical Case Presentations

684

CA

SES

Interpretive discussion

Hematology The hematocrit is low normal, but some degree of anemia is probable given physical dehydration; the baseline hema-tocrit is not known.

There is moderate hypoproteinemia, to be considered in chemistry interpretation.

The leukogram indicates moderate lymphocytosis. The presence of mostly abnormal lymphocyte forms indicates lymphproliferative disease that is leukemic. The left shift with mild neutrophilia indicates concurrent infl ammatory stimulus. Minimal monocytosis accompanies the infl amma-tory pattern.

There is thrombocytopenia with some giant platelets, indi-cating active thrombopoiesis. A consumptive process should be considered.

Chemistry Renal analytes The following processes are identifi ed: • Moderate azotemia indicated by BUN and prominent hyperphosphatemia. The creatinine has likely increased in this individual, within the reference interval. Increased mag-nesium is due to decreased GFR. • Likely a prerenal component given dehydration and other evidence of poor perfusion such as lactic acidosis (later). A urinalysis, especially specifi c gravity, would be helpful. • Possible renal component requires urinalysis for further characterization. • The Ca × P is 124 indicating calcifi cation will be occurring.

Liver The following processes are identifi ed: • There is hepatocellular injury indicated by increased activities of ALT/AST. • There is evidence of liver function failure in the form of probable decreased cholesterol synthesis and marked hyper-bilirubinemia in the face of near normal HCT.

Hematology Reference Interval

TP (P) (g/dL) 4.7 6.0 – 8.0

PCV (%) 41 40 – 55

Hgb (g/dL) 14.6 13.0 – 20.0

RBC ( × 10 6 / μ L) 6.07 5.5 – 8.5

MCV (fL) 67 62 – 73

MCHC (g/dL) 36 33 – 36

NCC ( × 10 3 / μ L) 26.6 4.5 – 15.0

Bands ( × 10 3 / μ L) 1.3 0 – 0.2

Segs ( × 10 3 / μ L) 12.2 2.6 – 11.0

Lymphs ( × 10 3 / μ L) 11.9 1.0 – 4.8

Monos ( × 10 3 / μ L) 1.1 0.2 – 1.0

Eos ( × 10 3 / μ L) 0 0.1 – 1.2

Platelets ( × 10 3 / μ L) 90 200 – 500

MPV (fL) 13.9 7.5 – 14.6

Hemopathology noted on blood fi lm:

• Most lymphoid cells are large with fi ne granular chromatin;

interpreted as mostly prolymphocytes with some blasts.

• No platelet clumps found, occasional giant platelets.

Case 40 Signalment: Ten - year - old spayed female Doberman dog History: Lethargy, weight loss, diarrhea, and poor appetite Physical examination: Depressed, dehydrated, hypo tensive, and icteric

Biochemical Profi le Reference Interval

Gluc (mg/dL) 65 75 – 130

BUN (mg/dL) 69 7 – 32

Creat (mg/dL) 1.5 0.4 – 1.5

Phos (mg/dL) 13.2 2.1 – 6.0

Ca (mg/dL) 9.4 9.2 – 11.7

Mg (mg/dL) 3.4 1.8 – 2.5

TP (g/dL) 3.9 5.3 – 7.2

Alb (g/dL) 2.4 2.5 – 4.0

Glob (g/dL) 1.5 2.0 – 3.8

Chol (mg/dL) 102 130 – 300

T. Bili (mg/dL) 12.6 0 – 0.3

ALP (IU/L) 1717 20 – 142

ALT (IU/L) 590 10 – 110

AST (IU/L) 401 16 – 50

GGT (IU/L) 5 0 – 8

Na (mEq/L) 138 142 – 152

K (mEq/L) 4.3 3.5 – 5.2

CL (mEq/L) 100 108 – 120

Bicarbonate (mEq/L) 10.5 16 – 25

An. gap (mEq/L) 32 13 – 22

Blood Gas Analysis – venous

Reference Interval

pH 6.92 7.33 – 7.45

pCO2 (mmHg) 57.3 24 – 39

pO2 (mmHg) 75.9 67 – 92

HCO 3 (mEq/L) 11.3 15 – 24

Lactate (mmol/L) 8.4 0.2 – 1.4

Page 71: VI Clinical Case Presentations - Wiley · Clinical Case Presentations ... Case 17: Dog with vitamin K antagonist (rodenticide) toxicity, 643 ... PT Prothrombin time

Case 40

685

CA

SES

• Cholestasis and/or steroid induced ALP are considerations given the ALP magnitude; cholestasis is a likely contributing factor to the hyperbilirubinemia. • Mild hypoglycemia and hypoalbuminemia may also be related in part to function failure.

Protein There is marked hypoproteinemia due to panhypoprotein-emia. All potential causes of loss should be evaluated, par-ticularly GI given the observation of diarrhea.

Acid - base/electrolyte The following processes are identifi ed: • There is severe, life - threatening acidemia. This is a mixed acidosis of two processes. • There is metabolic acidosis indicated by decreased bicar-bonate. This is most likely related to poor tissue perfusion and development of lactate acidemia, as well as uremic acids. • There is also a component of respiratory acidosis indicated by the prominent increase in pCO 2 . The respiratory compo-nent is likely related to terminal respiratory failure; this would prompt evaluation of cardiopulmonary function. • Both the pCO 2 and bicarbonate are counter to any detect-able compensation. • The increased anion gap is attributable to retention of renal - excreted anions such as phosphate and sulfate, and lactate is a contributing factor.

Summary

• Biochemical evidence of liver function failure, with hepa-tocellular injury and cholestasis. Hepatic infi ltrate with lymphoma should be considered given that there is blood evidence of lymphoproliferative disease. • Azotemia likely prerenal; kidneys should be investigated further. • Severe mixed acidemia compatible with multiple causes of metabolic acid and respiratory acid formation. • Lymphoproliferative disorder. • Marked panhypoproteinemia.

Recommendations for further characterization would include: • Urinalysis. • Evaluation of liver and kidney size with sampling for pos-sible infi ltrative disease (lymphoma). • Bone marrow for possible infi ltrate. • Cytometric analysis of blood leukocytes if treatment is contemplated.

As follow - up, the dog was euthanized. Necropsy fi ndings included splenomegaly, hepatomegaly, lymphadenopathy with marked involvement with lymphoma. The severity of this change in liver would explain the biochemical evidence of function failure.

Page 72: VI Clinical Case Presentations - Wiley · Clinical Case Presentations ... Case 17: Dog with vitamin K antagonist (rodenticide) toxicity, 643 ... PT Prothrombin time

SECTION VI Clinical Case Presentations

686

CA

SES

Case 41 Signalment: 9 - month - old intact female dog History: Struck by car 3 weeks ago. Treated for shock and released. Listless since then. Physical examination: Abdomen distended and fl uid - fi lled

Hematology Reference Interval

PCV (%) 30 37 – 55

Hgb (g/dL) 10.3 12 – 18

RBC ( × 10 6 / μ L) 5.45 5.5 – 8.5

MCV (fL) 55 60 – 72

MCHC (g/dL) 34 33 – 38

Retics (/ μ L) 42 < 60

NCC ( × 10 3 / μ L) 16 6 – 17

Segs ( × 10 3 / μ L) 12.8 3 – 11.5

Bands ( × 10 3 / μ L) 0.5 0 – 0.3

Lymphs ( × 10 3 / μ L) 2.7 1 – 4.8

Platelets ( × 10 3 / μ L) 270 200 – 500

TP (P) (g/dL) 6.5 6 – 8

Hemopathology: Slight hypochromasia, moderate number of

keratocytes

Biochernical Profi le

Reference Interval

Gluc (mg/dL) 65 65 – 122

BUN (mg/dL) 25 7 – 28

Creat (mg/dL) 1.2 0.9 – 1.7

Ca (mg/dL) 8.4 (2.1) 9.0 – 11.2 (2.25 – 2.80 mmol/L)

Phos (mg/dL) 6.0 2.8 – 6.1

TP (g/dL) 5.8 5.4 – 7.4

Alb (g/dL) 2.5 2.7 – 4.5

Glob (g/dL) 3.3 1.9 – 3.4

T. Bili 0.5 (8.5) 0 – 0.4 (0 – 6.8 μ mol/L)

Chol (mg/dL) 170 130 – 370

ALT (IU/L) 23 10 – 120

AST (IU/L) 28 16 – 40

ALP (IU/L) 51 35 – 280

Na (mEq/L) 139 145 – 158

K (mEq/L) 5.2 4.1 – 5.5

CL (mEq/L) 105 106 – 127

TCO 2 (mEq/L) 15 14 – 27

An. gap (mEq/L) 24 8 – 25

Urinalysis (voided)

Color Dark yellow Urine Sediment

Transparency Clear WBCs/hpf 0

Sp. Gr. 1.030 RBCs/hpf 0

Protein Negative Epith cells/hpf 0

Gluc Negative Casts/lpf 0

Bilirubin 3 + Crystals Bilirubin

Blood Negative Bacteria 0

pH 6.0

Body Fluid Analysis

Color Red - brown Differential

Transparency Hazy Neutrophils 74%

TP (g/dL) 3.8 Lymphs 5%

NCC (/ μ L) 8800 Macrophages 21%

Other observations: Neutrophils are nondegenerate. Lymphocytes are

uniformly small. Large mononuclear cells are a mixture of reactive

mesothelial cells and macrophages. Macrophages contain large

amounts of blue - green pigment, suggestive of bile. No micro -

organisms are evident. Moderate numbers of erythrocytes are present.

Page 73: VI Clinical Case Presentations - Wiley · Clinical Case Presentations ... Case 17: Dog with vitamin K antagonist (rodenticide) toxicity, 643 ... PT Prothrombin time

Case 41

687

CA

SES

marked increase in urine bilirubin concentration. Dogs have a low renal threshold for bilirubin, and, in dogs with abnor-malities of hepatic conjugated bilirubin excretion, urine bili-rubin concentration may increase prior to increases in serum biliruhin concentration, or urine bilirubin concentration may be markedly increased while serum bilirubin concen-tration is only mildly increased.

Hyponatremia and hypochloremia were probably caused by dilution of these electrolytes in an increased volume of extracellular fl uid. This increased fl uid volume is a result of accumulation of fl uid in the peritoneal cavity. This “ third - spacing ” phenomenon commonly results in hyponatremia and hypochloremia.

Urinalysis Marked bilirubinuria and presence of bilirubin crystals are the only abnormalities in the urinalysis. These changes prob-ably resulted from increased passage of conjugated bilirubin into the blood with subsequent renal excretion. Either cho-lestasis or rupture of the bile duct or gall bladder could be an underlying cause.

Abdominal fl uid analysis Based on the total nucleated cell count and on the predomi-nance of neutrophils, the peritoneal fl uid should be classi-fi ed as an exudate. Since neutrophils are nondegenerate and bacteria are absent, this is probably a nonseptic exudate. The pigment noted in macrophages is suggestive of bile and, therefore, gall bladder or bile duct rupture are likely.

Summary

Exploratory surgery revealed a ruptured gall bladder. Due to severe adhesions throughout the peritoneal cavity, the owner was offered a guarded prognosis. The owner opted for euthanasia.

Bilirubin leaking with bile into this dog ’ s peritoneal cavity was reabsorbed through the peritoneal wall. The bilirubin entered the blood and was effi ciently excreted by the kidneys. As a result, serum bilirubin concentration increased only slightly while urine bilirubin concentration increased markedly. Serum activities of the hepatic leakage enzymes, ALT and AST, did not increase since there was no direct liver injury. Serum activity of ALP did not increase since there was no cholestasis.

Interpretive discussion

Hematology This dog has a nonregenerative anemia. The indices reveal that this anemia is microcytic and borderline hypochromic. These abnormalities, in combination with the presence of hypochromasia and keratocytes observed on the blood fi lm, indicate iron defi ciency. Serum iron concentration should be measured in this dog. Although the most common cause of iron defi ciency is chronic blood loss, there is no history of such blood loss in this case. In most such cases, examination of feces will reveal the presence of blood. GI parasites such as hookworms, should also be considered.

Borderline neutrophilia with a slight left shift suggests a mild tissue demand for neutrophils, and, therefore, a mild infl ammatory process. It is likely that the anemia is non - regenerative due to the presence of infl ammation (anemia of infl ammatory disease).

Biochemical profi le The blood glucose concentration is at the bottom of the refer-ence interval. Decreased carbohydrate intake or decreased hepatic production are possible causes. Since there is little evidence of hepatic failure in this case, decreased intake appears to be the most likely explanation. Alternatively, it may be normal for this dog.

The hypocalcemia may be a result of hypoalbuminemia, in which case it is not clinically signifi cant. An ionized calcium could be measured to confi rm this.

The mild hypoalbuminemia is probably due to decreased protein intake or decreased amino acid absorption from the GI tract. Evidence of hepatic failure is not present, and urine protein concentration is normal; therefore, decreased albumin production by the liver and increased albumin loss through the kidneys are unlikely. In light of the anemia and the evidence of iron defi ciency, chronic blood loss should be considered as a cause of hypoalbuminemia in this case; however, globulin concentration usually decreases propor-tionally with albumin concentration during blood loss. The globulin concentration may, however, have been increased in this dog due to chronic antigenic stimulation, and this would explain a normal globulin concentration despite blood loss severe enough to result in hypoalbuminemia.

The combination of increased serum and urine bilirubin concentrations suggests disruption in the hepatic excretion of conjugated bilirubin. The serum bilirubin concentration, while increased, appears inappropriately low in light of the

Page 74: VI Clinical Case Presentations - Wiley · Clinical Case Presentations ... Case 17: Dog with vitamin K antagonist (rodenticide) toxicity, 643 ... PT Prothrombin time

SECTION VI Clinical Case Presentations

688

CA

SES

Case 42 Signalment: 2 - year - old male mixed breed dog History: Weight loss, lethargy Physical examination: Thin dog, pendulous abdomen Hematology: Unremarkable

Biochemical Profi le

Reference Interval

Gluc (mg/dL) 64 (3.5) 65 – 122 (3.5 – 6.7 mmol/L)

BUN (mg/dL) 6 (2.l) 7 – 28 (2.5 – 10.0 mmol/L)

Creat (mg/dL) 1.0 0.9 – 1.7

Ca (mg/dL) 7.4 (1.85) 9.0 – 11.2 (2.25 – 2.80 mmol/L)

Phos (mg/dL) 2.8 2.8 – 6.1

TP (g/dL) 4.2 5.4 – 7.4

Alb (g/dL) 1.2 2.7 – 4.5

Glob (g/dL) 3.0 1.9 – 3.4

T. Bili (mg/dL) 0.4 0 – 0.4

Chol (mg/dL) 65 (1.7) 130 – 370 (3.4 – 9.6 mmol/L)

ALT (IU/L) 30 10 – 120

ALP (IU/L) 260 35 – 280

Bile Acids ( μ mol/L) 30 3.0 – 9.0

Na (mEq/L) 146 145 – 158

K (mEq/L) 4.1 4.1 – 5.5

CL (mEq/L) 115 106 – 127

Abdominal Fluid Analysis

TP (g/dL) 1.0

NCC (/ μ L) 1500

Segs (%) 60

Lymphs (%) 22

Macrophages (%) 18

Morphology: neutrophils nondegenerate

Interpretive discussion

Biochemical profi le A number of factors in the profi le suggest liver failure. These include a borderline low glucose, low BUN, hypoprotein-emia characterized by severe hypoalbuminemia, and a markedly low cholesterol concentration. Hepatic enzymes are often normal with end - stage liver disease. An alterna-tive, but less likely, possibility for this pattern is severe star-vation. The increased bile acids indicate decreased liver function and help confi rm end - stage liver disease.

Hypocalcemia may be due to hypoalbuminemia, in which case it is clinically insignifi cant. An ionized calcium could be measured to confi rm this.

Body fl uid analysis The abdominal fl uid has the typical features of a transudate. With end - stage liver disease this is due to a combination of hypoalbuminemia and increased portal blood pressure resulting in transudation of fl uid into the cavity.

Summary

Hepatic cirrhosis; end - stage liver disease.

Page 75: VI Clinical Case Presentations - Wiley · Clinical Case Presentations ... Case 17: Dog with vitamin K antagonist (rodenticide) toxicity, 643 ... PT Prothrombin time

Case 43

689

CA

SES

Case 43 Signalment: 3 - month - old intact female dog History: Anorexia, depression, and diarrhea of one week duration. Poor growth rate. Physical examination: Severe, diffuse dermatitis with multifocal ulcerative lesions

Hematology Reference Interval

PCV (%) 13 37 – 55

Hgb (g/dL) 4.5 12 – 18

RBC ( × 10 6 / μ L) 2.5 5.5 – 8.5

MCV (fL) 52 60 – 72

MCHC (g/dL) 35 33 – 38

Retic ( × 10 3 / μ L) 2.5 < 60

NCC ( × 10 3 / μ L) 1.6 6 – 17

Segs ( × 10 3 / μ L) 0.5 3 – 11.5

Bands ( × 10 3 / μ L) 0.1 0 – 0.3

Monos ( × 10 3 / μ L) 0.1 0.1 – 1.3

Lymphs ( × 10 3 / μ L) 0.9 1 – 4.8

Platelets ( × 10 3 / μ L) 340 200 – 500

TP (P) (g/dL) 3.4 6 – 8

Hemopathology: Markedly toxic neutrophils, few RBC fragments.

Biochemical Profi le

Reference Interval

Gluc (mg/dL) 40 ( 2.2 ) 65 – 122 ( 3.5 – 6.7 mmol/L )

BUN (mg/dL) 4 ( 1.43 ) 7 – 28 ( 2.5 – 10.0 mmol/L )

Creat (mg/dL) 0.3 0.9 – 1.7

Ca (mg/dL) 7.8 ( 1.95 ) 9.0 – 11.2 ( 2.25 – 2.80 mmol/L )

Phos (mg/dL) 2.0 ( 0.65 ) 2.8 – 6.1 ( 0.9 – 2.0 mmol/L )

TP (g/dL) 2.9 5.4 – 7.4

Alb (g/dL) 1.7 2.7 – 4.5

Glob (g/dL) 1.2 1.9 – 3.4

T. Bili (mg/dL) 0.2 0 – 0.4

Chol (mg/dL) 142 130 – 370

ALT (IU/L) 15 10 – 120

AST (IU/L) 22 16 – 40

ALP (IU/L) 63 35 – 280

GGT (IU/L) 6 0 – 6

Na (mEq/L) 141 145 – 158

K (mEq/L) 3.7 4.1 – 5.5

CL (mEq/L) 114 106 – 127

TCO 2 (mEq/L) 17 14 – 27

An. gap (mEq/L) 14 8 – 25

Fasting Bile acids

( μ mol/L)

88 < 10

Iron ( μ g/dL) 50 ( 8.95 ) 60 – 110 ( 10.7 – 19.7 μ mol/L )

Urinalysis (cystocentesis)

Color Yellow Urine Sediment

Transparency Clear WBCs/hpf 0 – 2

Sp. Gr. 1.029 RBCs/hpf 0

Protein Negative Epith cells/hpf 0

Gluc 2 + Casts/lpf 0

Bilirubin Negative Crystals 0

Blood Negative Bacteria 0

pH 5.0

Page 76: VI Clinical Case Presentations - Wiley · Clinical Case Presentations ... Case 17: Dog with vitamin K antagonist (rodenticide) toxicity, 643 ... PT Prothrombin time

SECTION VI Clinical Case Presentations

690

CA

SES

Hypophosphatemia occurs most commonly in hypercalce-mic disorders such as primary hyperparathyroidism and pseudohyperparathyroidism, but these are unlikely in a 3 - month - old dog. Other causes include dietary phosphate or vitamin D defi ciency, malabsorption syndrome, diabetes mellitus, and canine Fanconi - like syndrome. This dog appears to have a renal tubular defect (see discussion of glucosuria), and it is possible that this defect is part of a canine Fanconi - like syndrome. In such a syndrome, inade-quate tubular reabsorption of phosphate results in excessive loss of phosphate in the urine.

The hypoproteinemia is the result of both hypoalbumin-emia and hypoglobulinemia. These abnormalities, in combi-nation with anemia, indicate that blood loss should be considered. In this case, it is likely that chronic liver disease is also contributing to hypoalbuminemia.

This dog ’ s serum bilirubin concentration, serum alkaline phosphatase (ALP) activity, and gamma glutamyltransferase (GGT) activity are normal, suggesting that cholestasis is not occurring. While most forms of liver failure result in some degree of cholestasis, liver failure resulting from a portosys-temic shunt usually does not. These normal values, in com-bination with the history and other laboratory abnormalities, suggest that a portosystemic shunt is likely. Since this is a young dog, a slight increase in ALP of bone origin would not have been unusual. Despite evidence of hepatic disease, serum ALT and AST activities are normal. Serum activities of hepatic leakage enzymes such as ALT and AST may be normal to increased in dogs with portosystemic shunts.

The hyponatremia and hypokalemia may have resulted from diuresis induced by glucosuria, or losses associated with diarrhea. It is also possible this dog ’ s tubular function defect includes abnormal reabsorption of Na and K. Decreased intake likely contributed to the hypokalemia.

Markedly increased fasting bile acid concentration can result from decreased hepatic blood fl ow, hepatic failure, or cholestasis. In this case, decreased hepatic blood fl ow and subsequent hepatic failure are the most likely explanations.

Blood loss is the most common cause of decreased serum iron concentration in animals, although nursing animals have low serum iron due to inadequate dietary intake. In this case, however, the decreased serum iron concentration is probably due to the presence of a portosystemic shunt. Decreased serum iron concentration does not always occur in dogs with portosystemic shunts and the cause is not known, but it appears to he related to iron sequestration in tissues such as liver and/or defects in the transport of iron. Some cases also have intermittent gastrointestinal bleeding associated with pica.

Urinalysis Moderate glucosuria in an animal with a low or normal blood glucose suggests a lowered renal threshold for glucose

Interpretive discussion

Hematology This dog has a severe nonregenerative anemia. The anemia in this dog is microcytic, and the serum iron concentration is decreased, suggesting iron defi ciency secondary to chronic blood loss. Alternately, microcytic anemia is also seen in dogs with portosystemic shunt, in which case serum iron may or may not be decreased, and anemia may be secondary to other abnormalities in iron metabolism. Red blood cell fragmentation is a typical fi nding in iron defi ciency anemia. While iron defi ciency anemia may be regenerative, this dog ’ s bone marrow is not adequately responding, perhaps due to viral damage or concurrent anemia of chronic disease.

Severe leukopenia has resulted from a combination of neutropenia and lymphopenia. In a young dog with diar-rhea as well as neutropenia and lymphopenia, parvovirus infection with virus - induced bone marrow damage should be a strong consideration. Acute bacterial enteritis resulting in endotoxemia may result in a similar leukogram. The pres-ence of toxic neutrophils suggests that the bone marrow is rapidly producing neutrophils, and this may signal early recovery of previously suppressed neutrophil production, or may be a response to loss of neutrophils due to rapid, destruction or emigration into tissues as would occur with endotoxemia or overwhelming tissue demand for neutro-phils, respectively.

Platelets are adequate, indicating chronic marrow failure is not present.

Biochemical profi le Hypoglycemia probably resulted from decreased hepatic glucose production. Numerous diseases can result in hypo-glycemia, but, in light of other laboratory data, hepatic failure is the most likely cause of hypoglycemia in this dog. The decreased blood supply to the liver which occurs with portosystemic shunts can result in liver atrophy. Such a liver cannot play its normal role in maintenance of blood glucose concentrations. A second possibility, in light of the decreased neutrophil concentration, is that the dog has bacteremia or endotoxemia which may result in hypoglycemia. A third possibility is that glucose is being lost through the urinary tract (see discussion of glucosuria).

Both the BUN and serum creatinine concentrations are decreased. Since there is evidence of hepatic failure, it is likely that the decreased BUN concentration resulted from decreased liver production of urea. Decreased creatinine refl ects decreased muscle mass.

This dog has both hypocalcemia and hypoalbuminemia. The hypocalcemia may be caused by decreased protein-bound calcium secondary to the hypoalbuminemia, in which case it is clinically insignifi cant. An ionized calcium could be measured to confi rm this.

Page 77: VI Clinical Case Presentations - Wiley · Clinical Case Presentations ... Case 17: Dog with vitamin K antagonist (rodenticide) toxicity, 643 ... PT Prothrombin time

Case 43

691

CA

SES

and, therefore, a renal tubular absorption defect. Such a defect may be confi ned to glucose absorption only, or may include defective absorption of several substances. As previ-ously noted, this dog may also have defective absorption of phosphate, sodium, and potassium. If this is the case, this is probably a form of canine Fanconi - like syndrome. Measure-ment of the fractional excretion of phosphate, sodium, and potassium would have helped in assessing this possibility.

Summary

This dog had a portosystemic shunt. Hypoglycemia, decreased BUN concentration, hypoalbuminemia, and increased serum

bile acid concentrations resulted from decreased hepatic blood fl ow and subsequent hepatic failure. Microcytosis has been reported as common in dogs with portosystemic shunts. This dog also has a renal tubular defect. Glucosuria in the presence of hypoglycemia, hypophosphatemia, hyponatre-mia, and hypokalemia are probably a result of defective tubular reabsorption of these substances. This defect is probably a canine Fanconi - like syndrome. Neutropenia does not typically occur in either portosystemic shunts or canine Fanconi - like syndrome, and this dog may have a concurrent enteric infection, most likely caused by parvovi-rus or endotoxin - producing bacteria, resulting in this abnormality.

Page 78: VI Clinical Case Presentations - Wiley · Clinical Case Presentations ... Case 17: Dog with vitamin K antagonist (rodenticide) toxicity, 643 ... PT Prothrombin time

SECTION VI Clinical Case Presentations

692

CA

SES

Case 44 Signalment: 6 - month - old intact male dog History: Struck by a car on Day 1

Physical examination: Pale mucous membranes. Day 1 blood sample obtained 12 hours after accident.

Hematology Day 1 Day 6 Reference Interval

PCV (%) 29 35 37 – 55

Hgb (g/dL) 9.6 11.5 12 – 18

RBC ( × 10 6 / μ L) 4.7 5.1 55 – 8.5

MCV (fL) 62 69 60 – 72

MCHC (g/dL) 33 33 33 – 38

Retics ( × 10 3 / μ L) 47 304 < 60

NCC ( × 10 3 / μ L) 22.7 20.0 6 – 17

Segs ( × 10 3 / μ L) 22.0 12.0 3 – 11.5

Bands ( × 10 3 / μ L) 0 2.0 0 – 0.3

Monos ( × 10 3 / μ L) 0 1.0 0.1 – 1.3

Lymphs ( × 10 3 / μ L) 0.7 5.0 1 – 4.8

Platelets ( × 10 3 / μ L) 340 460 200 – 500

TP (P) (g/dL) 5.4 6.5 6 – 8

Hemopathology: No abnormalities on Day 1. Moderate anisocytosis and polychromasia on Day 6.

Biochemical Profi le Day 1 Day 6 Reference Interval

Gluc (mg/dL) 125 ( 6.9 ) 105 65 – 122 ( 3.5 – 6.7 mmol/L )

BUN (mg/dL) 9 13 7 – 28

Creat (mg/dL) 1.1 1.3 0.9 – 1.7

Ca (mg/dL) 8.9 ( 2.22 ) 9.3 9.0 – 11.2 ( 2.25 – 2.80 mmol/L )

Phos (mg/dL) 5.5 5.6 2.8 – 6.1

TP (g/dL) 5.0 6.0 5.4 – 7.4

Alb (g/dL) 3.4 4.0 2.7 – 4.5

Glob (g/dL) 1.6 2.0 1.9 – 3.4

T. Bili (mg/dL) 0.3 0.4 0 – 0.4

Chol (mg/dL) 210 180 130 – 370

ALT (IU/L) 1098 150 10 – 120

AST (IU/L) 948 80 16 – 40

ALP (IU/L) 302 295 35 – 280

Na (mEq/L) 150 147 145 – 158

K (mEq/L) 4.8 4.7 4.1 – 5.5

CL (mEq/L) 120 121 106 – 127

TCO 2 (mEq/L) 12 21 14 – 27

An. gap (mEq/L) 23 10 8 – 25

Urinalysis (catheterized) — obtained on Day 1

Color Yellow Urine Sediment

Transparency Clear WBCs/hpf 1 – 2

Sp. Gr. 1.019 RBCs/hpf 3 – 5

Protein Trace Epith cells/hpf 0

Glucose Negative Casts/lpf 0

Bilirubin Negative Crystals 0

Blood Negative Bacteria 0

pH 6.5

Page 79: VI Clinical Case Presentations - Wiley · Clinical Case Presentations ... Case 17: Dog with vitamin K antagonist (rodenticide) toxicity, 643 ... PT Prothrombin time

Case 44

693

CA

SES

Both serum ALT and AST activities are markedly increased on Day 1, but decrease to nearly normal by Day 6. These increases suggest liver and/or muscle injury. High concen-trations of ALT are present in the liver and lower concentra-tions are present in muscle. The marked increase in ALT in this case, therefore, probably resulted from liver injury, but muscle injury may have also contributed. Aspartate amino-transferase (AST) is present in high concentrations in both liver and muscle, and both tissues may be sources of AST in this case. Liver and muscle trauma may explain these increased enzyme activities on Day 1. Shock with subse-quent hypoxia and tissue injury can also result in leakage and increased serum activities of both enzymes. Regardless of the underlying cause of their leakage, the decreasing activities of both enzymes by Day 6 imply the damage was acute, and it is no longer active.

Increased alkaline phosphatase (ALP) activities are likely normal for this dog. Young, growing animals commonly have slightly to moderately increased serum ALP activity since, due to active bone growth, increased amounts of ALP are released from osteoblasts.

There is a slight decrease in the total CO 2 on Day 1, sug-gesting metabolic acidosis. Hypovolemic shock leading to tissue hypoxia may have resulted in production of acid metabolites, and decreased renal blood fl ow may have inter-fered with renal acid - base regulation. The anion gap, while still within the reference interval, is higher on Day 1 as compared to Day 6, and this may have resulted from increased blood concentrations of anions such as lactate.

Urinalysis In light of the relatively dilute urine (specifi c gravity = 1.019), the urine concentration of erythrocytes may be slightly increased. Mild hematuria may have resulted from trauma.

Summary

This dog had a dislocated hip and broken femur. Surgery was performed between Days 1 and 6. The dog ’ s recovery was uneventful. This case demonstrates a normal response to acute blood loss. It also demonstrates the importance of serial measurement of serum enzyme activities in animals with increases of these activities. Steady or increasing activi-ties of these enzymes indicates active and continuing damage to the tissue(s) of origin. Decreasing activities usually indicate that the injury has ceased and/or is resolving.

Interpretive discussion

Hematology This dog is anemic on both days. The anemia is more severe on Day 1 and is nonregenerative. Since the Day 1 blood sample was obtained 12 hours after the accident, it is likely that the anemia is due to acute blood loss. The concurrent hypoproteinemia (see discussion below) also supports blood loss as the cause of this anemia. Increased polychromasia and reticulocyte count are not evident in blood until 2 to 4 days following acute blood loss. While the anemia appears nonregenerative on the initial sample, by Day 6, the eryth-rocyte values have increased, and there is evidence of increased erythrocyte production (increased polychromasia and reticulocyte count). This dog is, therefore, responding appropriately to the blood loss.

Although normal on both days, the MCV increased between Day 1 and Day 6, probably due to increased eryth-rocyte production resulting in increased number of large, immature erythrocytes.

The dog has a mature neutrophilia and lymphopenia on Day 1. This is compatible with a corticosteroid - mediated leukogram, resulting from stress associated with pain or trauma.

Neutrophilia and a left shift on Day 6 are typical of an infl ammatory leukogram. Tissue injury associated with the accident probably incited an infl ammatory response. An infectious etiology cannot be excluded, however.

Biochemical profi le Mild hyperglycemia on Day 1 resulted from stress. The pres-ence of a stress leukogram supports this explanation.

Slight hypocalcemia may be normal in this dog since young animals commonly have slightly lower serum Ca concentrations than adults. However, the serum Ca concen-tration returned to within the reference interval on Day 6, suggesting that this is the more normal value for this dog. It is possible that the hypocalcemia on day 1 resulted from loss of albumin and albumin - bound Ca during hemorrhage.

Hypoproteinemia and hypoglobulinemia on Day 1 prob-ably resulted from loss of protein during hemorrhage. Although the serum albumin concentration is in the refer-ence interval, this might actually be low for this animal. The serum albumin concentration increased by Day 6, implying that this is the more normal concentration for this dog. All protein concentrations returned to within the reference intervals by Day 6, indicating that compensatory mecha-nisms had replaced the protein lost through hemorrhage.

Page 80: VI Clinical Case Presentations - Wiley · Clinical Case Presentations ... Case 17: Dog with vitamin K antagonist (rodenticide) toxicity, 643 ... PT Prothrombin time

SECTION VI Clinical Case Presentations

694

CA

SES

Case 45 Signalment: 5 - year - old cocker spaniel History: Presented for anorexia and dark orange urine and feces. Dog had ITP 2 years previously, and has been given phenobarbital (100 mg bid) for epilepsy for several years.

Hematology Reference Interval

PCV (%) 13 37 – 55

RBC ( × 10 6 / μ f) 1.95 5.5 – 8.5

Hgb (g/dL) 4.6 12 – 18

MCV (fL) 67 60 – 72

MCHC (g/dL) 35 33 – 38

Retics (/ μ L) 0 0 – 60,000

NCC ( × 10 3 / μ L) 54.9 6.0 – 17.0

Metas ( × 10 3 / μ L) 1.1 0

Bands ( × 10 3 / μ L) 6.0 0 – 0.3

Segs ( × 10 3 / μ L) 43.4 3.0 – 11.5

Lymphs ( × 10 3 / μ L) 1.1 1.0 – 4.8

Monos ( × 10 3 / μ L) 2.2 0.2 – 1.4

Eos ( × 10 3 / μ L) 0.5 0.1 – 1.2

NRBCs ( × 10 3 / μ L) 0.5 0

Platelets ( × 10 3 / μ L) 260 200 – 500

TP (P) (g/dL) 6.3 6.0 – 8.0

Hemopathology: occasional imperfect spheres, slight agglutination.

Coombs test: positive

Bone marrow aspirate

Megakaryocytes present. Myeloid and erythroid hyperplasia, with

normal maturation up to metarubricyte stage. M : E ratio decreased

slightly. Rare erythrophagocytosis.

Biochemical Profi le

Reference Interval

Gluc (mg/dL) 56 ( 3.1 ) 65 – 122 ( 3.5 – 6.7 mmol/L )

BUN (mg/dL) 56 ( 19.9 ) 7 – 28 ( 2.5 – 10 mmol/L )

Creat (mg/dL) 0.6 0.6 – 1.5

Ca (mg/dL) 8.5 ( 2.1 ) 9.0 – 1.12 ( 2.25 – 2.80 mmol/L )

Phos (mg/dL) 6.4 ( 2.1 ) 2.8 – 6.1 ( 0.9 – 2.0 mmol/L )

TP (g/dL) 3.8 5.4 – 7.4

Alb (g/dL) 1.5 2.7 – 4.5

Glob (g/dL) 2.3 1.9 – 3.4

T. Bili (mg/dL) 35.8 ( 612.2 ) 0 – 0.4 ( 0 – 6.84 μ mol/L )

Chol (mg/dL) 64 ( 1.6 ) 130 – 370 ( 3.4 – 9.6 mmol/L )

ALT (IU/L) 70 16 – 40

ALP (IU/L) 566 18 – 141

GGT (IU/L) 15 0 – 6

Na (mEq/L) 160 145 – 158

K (mEq/L) 3.2 4.1 – 5.5

CL (mEq/L) 135 106 – 127

TCO 2 (mEq/L) 9.5 14 – 27

An. gap (mEq/L) 16 8 – 26

Urinalysis

Color brown

Transparency cloudy

Sp. Gr. 1.022

Bilirubin + + + +

No other abnormal fi ndings.

Page 81: VI Clinical Case Presentations - Wiley · Clinical Case Presentations ... Case 17: Dog with vitamin K antagonist (rodenticide) toxicity, 643 ... PT Prothrombin time

Case 45

695

CA

SES

fusing. The mild increase in serum phosphorus suggests decreased glomerular fi ltration rate.

The serum calcium is decreased, likely due to hypoalbu-minemia. An ionized calcium could be measured to confi rm this.

The serum total protein concentration is decreased due to hypoalbuminemia. Since the globulin is within the reference interval, liver failure would be the best differential, since the animal is not proteinuric. Another consideration would be that blood loss is causing the anemia and hypoproteinemia, but there is no clinical evidence of blood loss.

The serum bilirubin concentration is markedly increased and may be due to hemolysis, liver failure, or cholestasis or some combination of the three. The ALT is only slightly increased, indicating mild hepatocellular damage. The ALP activity is increased, as is GGT activity, indicating cholestasis. Alternatively, enzymes may be induced by phenobarbital.

Serum total CO 2 is decreased, suggesting metabolic acido-sis. This may be secondary to lactic acidosis associated with marked anemia. Increased sodium and chloride suggest hypertonic dehydration or administration of hypertonic fl uid. The hypokalemia in conjunction with metabolic aci-dosis (which should have caused an increased potassium) suggests whole body potassium depletion.

Urinalysis The marked bilirubinuria refl ects conjugated hyperbilirubi-nemia. Specifi c gravity is discussed above.

Summary

Immune - mediated hemolytic anemia, liver failure, and renal dysfunction. The dog died, and necropsy showed severe chronic micronodular cirrhosis and cholestasis, pos-sibly related to phenobarbital. Bone marrow showed myeloid and erythroid hyperplasia. Examination of the kidneys revealed severe hemoglobinemic nephrosis with mild chronic interstitial nephritis.

Interpretive discussion

Hematology The dog is markedly anemic. Reticulocytes are not increased, indicating that the anemia is not regenerative. The presence of the imperfect spherocytes and agglutination is suggestive of immune - mediated hemolytic anemia, possibly very acute, or with destruction of precursors. An unexplained nonre-generative anemia, when platelets and neutrophils are normal and increased, respectively, triggered a bone marrow aspirate. The bone marrow aspirate fi ndings further substan-tiated immune - mediated hemolytic anemia with destruction of polychromatophilic cells.

Neutrophilia, increased immature neutrophils, and mono-cytosis are indicative of infl ammation.

If the animal has not received a previous transfusion, a positive Coombs ’ test is suggestive of immune - mediated hemolytic anemia.

Bone marrow In light of marked erythroid response in marrow, anemia is either very acute, and will respond, or precursors are being destroyed. Because imperfect spherocytes are present on blood fi lm, the latter is more likely.

Biochemical profi le The serum glucose concentration is decreased. Differentials should include insulinoma and, in this patient, end - stage liver disease, since the animal is also hypoalbuminemic and hypocholesterolemic.

The BUN is increased, and although the creatinine is within the reference interval, one would expect the animal to be concentrating greater than 1.022 if the azotemia is prerenal. One should consider if the animal is bleeding into the GI tract, increasing the BUN, or since IMHA is suspected based on the hematology, if the animal has hemolysis with subsequent hemoglobinuric nephrosis. If the animal does have end - stage liver disease, one would expect the BUN to be decreased as well, so the increase in BUN is slightly con-

Page 82: VI Clinical Case Presentations - Wiley · Clinical Case Presentations ... Case 17: Dog with vitamin K antagonist (rodenticide) toxicity, 643 ... PT Prothrombin time

SECTION VI Clinical Case Presentations

696

CA

SES

Interpretive discussion

Hematology The dog has mild anemia that is not characterized by a reticulocyte count. The anemia is microcytic suggesting iron defi ciency. Microcytosis should also prompt consideration of a portosystemic shunt in a young dog. Although some dogs with portosystemic shunt will have low serum iron concen-trations, marked hypoferremia suggests iron defi ciency is the cause of the microcytosis. There is a marked leukocytosis characterized by neutrophilia with a left shift; this is inter-preted as an infl ammatory leukogram.

Biochemical profi le The decreased BUN suggests reduced biosynthesis of urea by the liver. The same may be interpreted for cholesterol and albumin. There is a slight increase in ALT suggesting a mild degree of hepatocellular injury. The markedly increased bile acid concentration confi rms a defect in hepatic function. The bilirubin and ALP do not indicate cholestasis. The slight increase in GGT is of questionable signifi cance.

Hypocalcemia may be due to hypoalbuminemia, in which case it is clinically insignifi cant. An ionized calcium could be measured to confi rm this.

Decreased creatinine refl ects decreased muscle mass.

Summary

The fi ndings of reduced hepatic biosynthesis with retention of bile acids in a young dog are highly suggestive of porto-systemic shunt.

Case 46 Signalment: 8 - month - old German shepherd History: Lethargic, “ poor doer, ” weight loss

Hematology Reference Interval

PCV (%) 34 37 – 55

MCV (fL) 52 60 – 72

NCC ( × 10 3 / μ L) 44.6 6 – 17

Segs ( × 10 3 / μ L) 38.0 3 – 11.5

Bands ( × 10 3 / μ L) 2.2 0 – 0.3

Lymphs ( × 10 3 / μ L) 3.1 1.0 – 4.8

Monos ( × 10 3 / μ L) 0.9 0.2 – 1.4

Eos ( × 10 3 / μ L) 0.4 0.1 – 1.2

Platelets ( × 10 3 / μ L) Adequate 200 – 500

Biochemical Profi le

Reference Interval

Gluc (mg/dL) 87 65 – 122

BUN (mg/dL) 6 ( 2.1 ) 7 – 28 ( 2.5 – 10.0 mmol/L )

Creat (mg/dL) 0.5 0.9 – 1.7

Ca (mg/dL) 8.6 ( 2.15 ) 9.0 – 11.2 ( 2.25 – 2.80 mmol/L )

Phos (mg/dL) 5.6 2.8 – 6.1

TP (g/dL) 4.3 5.4 – 7.4

Alb (g/dL) 2.4 2.7 – 4.5

Glob (g/dL) 1.9 1.9 – 3.4

T. Bili (mg/dL) 0.4 0 – 0.4

Chol (mg/dL) 75 (1.95) 130 – 370 ( 3.4 – 9.6 mmol/L )

ALT (IU) 250 10 – 120

ALP (IU) 129 35 – 280

GGT (IU) 7 0 – 6

Na (mEq/L) 154 145 – 158

K (mEq/L) 4.1 4.1 – 5.5

CL (mEq/L) 126 106 – 127

TCO 2 (mEq/L) 22.3 14 – 27

An. gap (mEq/L) 10 8 – 26

Bile acids, fasting

( μ mol/L)

88.5 < 10

Serum iron ( μ g/dL) 22 60 – 100

Page 83: VI Clinical Case Presentations - Wiley · Clinical Case Presentations ... Case 17: Dog with vitamin K antagonist (rodenticide) toxicity, 643 ... PT Prothrombin time

Case 47

697

CA

SES

Interpretive discussion

Hematology There is moderate regenerative anemia. Considering the hypo-proteinemia, hemorrhage is the most likely cause. The nucleated RBC are interpreted as part of the regenerative response. There is a marked leukocytosis characterized by prominent neutro-philia with toxic change and a left shift to myelocytes indicating infl ammation. The mild thrombocytopenia may be caused by hemorrhage, and giant platelets indicate active thrombopoiesis.

Biochemical profi le and urinalysis There is marked hyperglycemia. This is associated with an expected glucosuria. The magnitude of hyperglycemia should prompt consideration of diabetes mellitus. The lack of urine ketones makes the diagnosis more diffi cult.

Moderate azotemia is indicated by increased concentra-tions of BUN and creatinine. The specifi c gravity indicates minimal concentrating ability in the face of azotemia. This suggests an element of primary renal disease. However, elec-trolyte depletion (see later) may be contributing to the decreased concentrating ability. The increased phosphorus is compatible with decreased glomerular fi ltration.

The hypoproteinemia along with regenerative anemia is compatible with blood loss.

There is a marked increase in ALT activity indicating hepa-tocellular injury. Diabetes is associated with fat mobilization to the liver; this may result in modest ALT activity increases. The magnitude of this ALT suggests more severe injury. There is also an element of cholestasis indicated by the marked increase in ALP and a minimal increase in bilirubin.

The hyponatremia is likely due to urinary sodium loss sec-ondary to glucosuria (osmotic diuresis). Losses associated with diarrhea may have contributed. Additionally, cellular water may move from the intracellular compartment into the extra-cellular fl uid compartment, diluting serum sodium (expect 1.6 mEq/L decrease in sodium for every 100 mg/dL increase in glucose). The hyperkalemia is probably due to a shift of potas-sium ions out of cells in exchange for hydrogen ions, which enter cells during metabolic acidosis. Another possibility is that the animal is becoming oliguric and retaining potassium.

Increased anion gap is due to the presence of “ unmeasured ” anions. In this dog, these likely include phosphates, as well as lactate, since the dog is markedly anemic. In addition, because this dog is diabetic, ketones may contribute to unmeasured anions. Since beta hydroxybutyrate is not detected by routine urine dipstick methods, ketonuria may actually be present.

Summary

Further evaluation led to the fi ndings of diabetes mellitus and hepatic lipidosis. The enlarged, fragile liver had led to a fractured liver. This latter injury likely contributed to the magnitude of the ALT increase.

Signalment: 8 - year - old male Samoyed History: Diarrhea Physical examination: Recumbent, arrested prior to treatment

Case 47

Hematology Reference Interval

PCV (%) 18 37 – 55

Retics ( × 10 3 / μ L) 197,830 ( 7.3% ) < 60,000

MCV (fL) 66 60 – 72

NCC ( × 10 3 / μ L) 78.0 6 – 17

Segs ( × 10 3 / μ L) 44.5 3 – 11.5

Bands ( × 10 3 / μ L) 14.8 0 – 0.3

Metas ( × 10 3 / μ L) 3.9 0

Myelocytes ( × 10 3 / μ L) 0.8 0

Monos ( × 10 3 / μ L) 0.8 0.1 – 1.3

Lymphs ( × 10 3 / μ L) 3.1 1 – 4.8

NRBC ( × 10 3 / μ L) 9.4 0

Platelets ( × 10 3 / μ L) 158 200 – 500

Hemopathology: Increased polychromasia, target cells, giant platelets,

toxic neutrophils.

Biochemical Profi le

Reference Interval

Gluc (mg/dL) 580 ( 31.9 ) 65 – 122 ( 3.5 – 6.7 mmol/L )

BUN (mg/dL) 98 ( 35 ) 7 – 28 ( 2.5 – 10.0 mmol/L)

Creat (mg/dL) 3.1 ( 274 ) 0.9 – 1.7 ( 80 – 150 μ mol/L )

Ca (mg/dL) 9.6 9.0 – 11.2

Phos (mg/dL) 13.1 ( 4.2 ) 2.8 – 6.1 ( 0.9 – 2.0 mmol/L )

TP (g/dL) 4.7 5.4 – 7.4

Alb (g/dL) 2.4 2.7 – 4.5

T. Bili (mg/dL) 0.6 ( 10.3 ) 0 – 0.4 ( 0 – 6.8 μ mol/L )

Chol (mg/dL) 246 130 – 370

ALT (IU/L) 1031 10 – 120

ALP (IU/L) 2500 35 – 280

Na (mEq/L) 130 145 – 158

K (mEq/L) 6.5 4.1 – 5.5

CL (mEg/L) 87 106 – 127

TCO 2 (mEq/L) 10.6 14 – 27

An. gap (mEq/L) 39 8 – 26

Urinalysis

Sp. Gr. 1.017

Gluc 2 +

Protein 0

Ketones 0

No other abnormalities present.

Page 84: VI Clinical Case Presentations - Wiley · Clinical Case Presentations ... Case 17: Dog with vitamin K antagonist (rodenticide) toxicity, 643 ... PT Prothrombin time

SECTION VI Clinical Case Presentations

698

CA

SES

Interpretive discussion

Hematology The leukogram shows a lymphopenia with a high normal concentration of mature neutrophils; this is interpreted as a stress or steroid leukogram. There are acanthocyte - like or spiculated cells present. These are commonly observed in cats with liver disease or hepatic lipidosis.

Biochemical profi le The mildly decreased BUN may be insignifi cant or may be due to decreased hepatic urea production or decreased protein intake. The combination of hyperbilirubinemia and increased ALT and ALP activities is characteristic of hepatic lipidosis in cats. The combination of the hepatocellular injury (indicated by increased ALT) and cholestasis (indi-cated by increased ALP) lead to failure of bilirubin clearance and hyperbilirubinemia. This degree of increase in ALP activity is unusual in cats, other than in association with hepatic lipidosis. Lipidosis is thought to occur as a result of massive fat mobilization from adipocytes in association with anorexia of several days duration or acute diabetes mellitus.

Summary

The biochemical fi ndings are characteristic of hepatic lipido-sis, which was confi rmed by liver aspiration cytology.

Signalment: 4 - year - old DSH cat History: Anorexia, weight loss, depression Physical examination: Thin, icteric mucous membranes

Hematology Reference Interval

PCV (%) 29 24 – 45

NCC ( × 10 3 / μ L) 13.7 5.5 – 19.5

NRBC ( × 10 3 / μ L) 0.1 0

Segs ( × 10 3 / μ L) 11.6 2.5 – 12.5

Bands ( × 10 3 / μ L) 0.1 0 – 0.3

Monos ( × 10 3 / μ L) 0.4 0 – 0.8

Lymphs ( × 10 3 / μ L) 0.7 1.5 – 7

Eos ( × 10 3 / μ L) 0.8 0 – 1.5

Platelets ( × 10 3 / μ L) 304 200 – 500

Morphology: Many acanthocyte - like RBCs, occasional fragmented

RBC.

Case 48

Biochemical Profi le

Reference Interval

Gluc (mg/dL) 67 67 – 124

BUN (mg/dL) 14 17 – 32

Creat (mg/dL) 1.2 0.9 – 2.1

Ca (mg/dL) 9.0 8.5 – 11

Phos (mg/dL) 5. I 3.3 – 7.8

TP (g/dL) 6.2 5.9 – 8.1

Alb (g/dL) 3.0 2.3 – 3.9

T. Bili (mg/dL) 6.3 ( 108 ) 0 – 0.3 ( 0 – 5.1 μ mol/L )

ALT (IU/L) 332 30 – 100

ALP (IU/L) 2185 11 – 210

Na (mEq/L) 149 146 – 160

K (mEq/L) 5.2 3.7 – 5.4

CL (mEq/L) 109 112 – 129

TCO 2 (mEq/L) 19 14 – 23

Page 85: VI Clinical Case Presentations - Wiley · Clinical Case Presentations ... Case 17: Dog with vitamin K antagonist (rodenticide) toxicity, 643 ... PT Prothrombin time

Case 49

699

CA

SES

Hematology Reference Interval

PCV (%) 28 24 – 35

Hgb (g/dL) 12.7 11 – 19

RBC ( × 10 6 / μ L) 12.1 8.8 – 15.4

MCV (fL) 23 21 – 30

MCHC (g/dL) 45.9 39.2 – 46.1

NCC ( × 10 3 / μ L) 8.4 5.2 – 15.7

Segs ( × 10 3 / μ L) 6.0 2.1 – 9.5

Monos ( × 10 3 / μ L) 0.6 0 – 0.6

Lymphs ( × 10 3 / μ L) 1.3 0.9 – 4.4

Eos ( × 10 3 / μ L) 0.4 0 – 3.3

Platelets ( × 10 3 / μ L) 2,141 206 – 3,600

TP (P) (g/dL) 9.0 5.4 – 7.2

Hemopathology: few reactive lymphocytes, mild anisocytosis. Grossly

lipemic plasma.

Case 49 Signalment Five - year - old female alpaca in the late stage of gestation History Lethargy, anorexia Physical Thin, depressed

Biochemical Profi le Reference Interval

Gluc (mg/dL) 129 100 – 132

BUN (mg/dL) 14 12 – 33

Creat (mg/dL) 1.7 1.3 – 2.7

Ca (mg/dL) 8.9 8.0 – 10.4

Phos (mg/dL) 3.9 2.5 – 8.6

TP (g/dL) 6.3 5.3 – 7.6

Alb (g/dL) 3.6 2.6 – 4.7

Glob (g/dL) 2.7 2.7 – 2.9

T. Bili (mg/dL) 0.1 0 – 0.2

Chol (mg/dL) 364 12 – 58

Trig (mg/dL) 4,330 5 – 30

β - hydroxybutyrate (mmol/L) 26.0 0.2 – 1.1

AST (IU/L) 474 110 – 250

SDH (IU/L) 17.6 3 – 10

GGT (IU/L) 76 10 – 42

ALP (IU/L) 105 20 – 150

CK (IU/L) 45 40 – 500

Na (mEq/L) 146 142 – 156

K (mEq/L) 3.8 3.6 – 6.5

CL (mEq/L) 112 108 – 122

TCO 2 (mEq/L) 13 19 – 29

An. Gap (mEq/L) 25 12 – 25

Grossly lipemic serum

Urinalysis (catheter)

Color Light yellow Urine Sediment

Transparency Clear WBCs/hpf Rare

Specifi c Gravity 1.006 RBCs/hpf None seen

Protein Negative Epithelial cells/hpf 0 – 1

Glucose Negative Casts/lpf 0

Bilirubin Negative Crystals 0

Blood Negative Bacteria 0

pH 9.0

Ketones 1 +

Page 86: VI Clinical Case Presentations - Wiley · Clinical Case Presentations ... Case 17: Dog with vitamin K antagonist (rodenticide) toxicity, 643 ... PT Prothrombin time

SECTION VI Clinical Case Presentations

700

CA

SES

Urinalysis The hyposthenuria is of unknown signifi cance at this time. Camelids with hepatic lipidosis are at risk for developing renal failure secondary to accumulation of lipid in the renal parenchyma. Although her BUN and creatinine are normal, her renal function should be carefully monitored. The alka-line urine pH is normal for herbivores. However, this is somewhat surprising in the face of metabolic acidosis and ketonuria.

Summary

Increased activities of AST, SDH and GGT along with hyper-triglyceridemia and hypercholesterolemia are common fi nd-ings in camelids with hepatic lipidosis. Ketosis and metabolic acidosis also can develop in these patients. Although not measured in this case, nonesterifi ed fatty acids (NEFA) are expected to be increased as a result of mobilization of fat. Hepatic lipidosis can be precipitated by severe negative energy balance associated with late term pregnancy, stress, or anorexia. Fat is mobilized to supply fatty acids for energy production. However, the supply of fatty acids exceeds the ability to utilize them in the tricarboxylic acid (TCA) cycle. Fatty acids in the liver are incorporated into triglycerides and released as very low density lipoproteins (VLDL), resulting in the hypertriglyceridemia. However, hepatic production of triglycerides exceeds the ability to export them as VLDL so triglycerides accumulate in the cells. Fatty acids also will be shunted into ketogenesis, resulting in the observed increase in β - hydroxybutyrate and ketonuria.

Interpretive discussion

Hematology The plasma protein determined by refractometer is signifi -cantly higher than the total protein determined on the bio-chemistry panel. This is due to the marked lipemia of the sample. Lipemia occurs when triglyceride concentrations are increased. The refractometer estimates plasma proteins by the bending of light in relation to the concentration of solutes in the sample. Hyperlipidemia will cause an artifac-tual increase in the plasma protein measurement. Although hypercholesterolemia does not cause visible lipemia, if markedly increased it can artifactually increase the refracto-metric plasma protein measurement.

Biochemical profi le There is a marked increase in triglycerides and cholesterol, compatible with hyperlipidemia that may develop in sick camelids. In this case, negative energy balance accompany-ing late term pregnancy likely precipitated this condition. The glucose in this case is still normal.

Increased activity of AST in combination with a normal creatine kinase is compatible with hepatocellular injury rather than muscle origin of the AST. Increased activity of SDH also suggests hepatocellular damage while GGT is an indicator of cholestasis. Increased AST, SDH, GGT, triglycer-ides and cholesterol are common fi ndings in camelids with hepatic lipidosis. Neither ALP nor bilirubin are increased and have been shown to be less reliable indicators of hepatic lipidosis in camelids.

The alpaca is ketotic as evidenced by the increase in serum β - hydroxybutyrate and the presence of ketones in the urine. The low TCO2 indicates a decrease in bicarbonate and a metabolic acidosis. A blood gas profi le is needed to com-pletely assess acid/base status.

Page 87: VI Clinical Case Presentations - Wiley · Clinical Case Presentations ... Case 17: Dog with vitamin K antagonist (rodenticide) toxicity, 643 ... PT Prothrombin time

Case 50

701

CA

SES

Case 50 Signalment: 7 - year - old female border collie History: Depression, anorexia Physical examination: Ascites, dermatitis of face and genital area

Hematology Reference Interval

PCV (%) 15 37 – 55

MCV (fL) 57 60 – 72

Retics ( × 10 3 / μ L) 118 < 60

NCC ( × 10 3 / μ L) 9.5 6 – 17

Segs ( × 10 3 / μ L) 4.3 3 – 11.5

Bands ( × 10 3 / μ L) 2.2 0 – 0.3

Metas ( × 10 3 / μ L) 0.6 0

Monos ( × 10 3 / μ L) 0.8 0.1 – 1.3

Lymphs ( × 10 3 / μ L) 0.7 1 – 4.8

NRBC ( × 10 3 / μ L) 0.9 0

Platelets ( × 10 3 / μ L) 20 200 – 500

Hemopathology: target cells, acanthocytes, schistocytes, toxic

neutrophils, giant platelets.

Biochemical Profi le

Reference Interval

Gluc (mg/dL) 45 65 – 122

BUN (mg/dL) 16 7 – 28

Creat (mg/dL) 1.0 0.9 – 1.7

Ca (mg/dL) 9.2 9.0 – 11.2

Phos (mg/dL) 3.8 2.8 – 6.1

TP (g/dL) 4.5 5.4 – 7.4

Alb (g/dL) 1.7 2.7 – 4.5

Glob (g/dL) 2.8 1.9 – 3.4

T. Bili (mg/dL) 3.3 0 – 0.4

Chol (mg/dL) 86 130 – 370

ALP (IU/L) 1391 35 – 280

ALT (IU/L) 239 10 – 120

Na (mEq/L) 147 145 – 158

K (mEq/L) 2.6 4.1 – 5.5

CL (mEq/L) 122 106 – 127

TCO 2 (mEq/L) 8.5 14 – 27

Fluid Analysis (abdominal)

Color Straw

Transparency Clear

NCC (/ μ L) 1300

TP (g/dL) 1.5

Coagulation Data

PT (sec) 20 6.5 – 9.0

aPTT (sec) 36 12 – 16

Page 88: VI Clinical Case Presentations - Wiley · Clinical Case Presentations ... Case 17: Dog with vitamin K antagonist (rodenticide) toxicity, 643 ... PT Prothrombin time

SECTION VI Clinical Case Presentations

702

CA

SES

Interpretive discussion

Hematology PCV is decreased, indicating anemia. Reticulocytes are increased, indicating that the anemia is somewhat regenera-tive. MCV is decreased, particularly in light of increased reticulocytes, suggesting iron defi ciency anemia secondary to chronic blood loss.

Infl ammatory leukogram is present, as evidenced by the marked left shift and toxic change in neutrophils. In light of low normal number of segmented neutrophils, sepsis or endotoxemia may be present. Lymphopenia suggests a con-current stress response.

The combination of thrombocytopenia, schistocytes, and prolonged PT and APTT suggests disseminated intravascular coagulopathy (DIC). Alternatively, this degree of thrombo-cytopenia may be seen with immune - mediated destruction or ehrlichiosis.

Biochemical profi le Hypoglycemia may be due to sepsis (leukogram is suggestive of sepsis or endotoxemia), end - stage liver disease, insulino-nia, or other type of neoplasia, such as a large hepatoma.

Hypoalbuminemia, in conjunction with low cholesterol, is indicative of GI disease (malabsorption, maldigestion, protein losing enteropathy) or end - stage liver disease. Another possible cause of low total protein is blood loss, since MCV indicates iron defi ciency anemia. However, albumin is relatively lower than globulin.

Total bilirubin is increased. While the animal is anemic, and blood destruction is a possible cause, the MCV suggests blood loss. Therefore the bilirubin is probably increased

due to cholestasis or hepatocellular dysfunction. Increased alkaline phosphatase activity suggests cholestasis.

Cholesterol is decreased, likely due to end - stage liver disease (see hypoalbuminemia discussion).

Hypokalemia may be due to decreased intake. In face of acidosis, it indicates total body depletion of potassium.

Decreased total CO 2 indicates metabolic acidosis. The decrease is likely due to lactic acidosis in this patient, since the dog is not uremic and there is no evidence of diabetic ketoacidosis.

Abdominal fl uid analysis Transudate, likely due to liver disease and hypo-albuminemia.

Coagulation data While prolonged PT and APPT may be due to lack of syn-thesis of coagulation factors by the liver, another explana-tion is DIC, in light of the decreased platelets.

Summary

End - stage liver disease; cholestasis DIC Infl ammation, possibly sepsis Iron defi ciency anemia

Dermatitis was determined to be necrolytic migratory ery-thema (superfi cial necrolytic dermatitis), which is associated with hyperglucagonemia, often seen with severe hepatic disease (hepatocutaneous syndrome).

Page 89: VI Clinical Case Presentations - Wiley · Clinical Case Presentations ... Case 17: Dog with vitamin K antagonist (rodenticide) toxicity, 643 ... PT Prothrombin time

Case 51

703

CA

SES

Case 51 Signalment: 10 - year - old spayed female miniature schnauzer History: Polydipsia, polyuria, weight loss, abdominal “ cramping ” for 1 month Physical Examination: Tense abdomen, thin with mild truncal alopecia and comedones on dorsal midline

Hematology Reference Interval

PCV (%) 48 37 – 55

NCC ( × 10 3 / μ L) 34.4 6 – 17

Segs ( × 10 3 / μ L) 29.0 3 – 11.5

Bands ( × 10 3 / μ L) 2.0 0 – 0.3

Monos ( × 10 3 / μ L) 3.4 0.1 – 1.3

Lymphs ( × 10 3 / μ L) 0 1 – 4.8

Platelets ( × 10 3 / μ L) Adequate 200 – 500

TP * (g/dL)) 9.0 * 6 – 8

* although dog fasted, plasma is markedly lipemic, so refractometric

measurement of total protein may be falsely increased

Biochemical Profi le

Reference Interval

Gluc (mg/dL) 353 (19.4) 65 – 122 (3.7 – 6.8 mmol/L)

BUN (mg/dL) 35 (12.5) 7 – 28 (6.1 – 11.4 mmol/L)

Creat (mg/dL) 1.2 0.9 – 1.7

Ca (mg/dL) 11.0 9.0 – 11.2

Phos (mg/dL) 6.0 2.8 – 6.1

TP (g/dL) 6.0 5.4 – 7.4

Alb (g/dL) 2.7 2.7 – 4.5

Glob (g/dL) 3.3 1.9 – 3.4

T. Bili (mg/dL) 1.2 (26.5) 0 – 0.4 (0 – 6.8 μ mol/L)

Chol (mg/dL) 900 (23.4) 130 – 370 (3.4 – 9.6 mmol/L)

ALT (IU/L) 987 10 – 120

ALP (IU/L) 1200 35 – 280

Na (mEq/L) 139 145 – 158

K (mEq/L) 3.1 4.1 – 5.5

CL (mEq/L) 100 106 – 127

TCO 2 (mEq/L) 12.2 14 – 27

An. gap (mEq/L) 30 8 – 25

Lipase (IU/L) 3500 30 – 560

Urinalysis

Color Yellow

Transparency Clear

Sp. Gr. 1.035

Protein Neg

Gluc 2 +

Ketones Neg

Bilirubin +

Blood Neg

pH 6.0

Endocrine Data Reference Interval

ACTH stimulation:

serum cortisol

( μ g/dL) - (pre)

4.5 (124) 1 – 4 (25 – 110 nmol/L)

serum cortisol

( μ g/dL) - (post)

14.6 < 20

Low dose dexamethasone suppression test:

serum cortisol

( μ g/dL) - (pre)

3.5 1 – 4

serum cortisol

( μ g/dL) - (8 - hour

post)

1.5 < 1.5

Page 90: VI Clinical Case Presentations - Wiley · Clinical Case Presentations ... Case 17: Dog with vitamin K antagonist (rodenticide) toxicity, 643 ... PT Prothrombin time

SECTION VI Clinical Case Presentations

704

CA

SES

ketone tests that use the nitroprusside reaction do not detect β - hydroxybutyric acid; therefore, the presence of this ketone cannot be ruled out. Other possibilities include lactic acidosis.

Serum lipase activity is increased. In this patient, this increase could partially be due to decreased GFR, as indi-cated by azotemia. However, the infl ammatory leukogram, increased bilirubin, increased alkaline phosphatase activity, hyperglycemia, and lipernia are also suggestive of pancreati-tis. This magnitude of lipase increase is highly supportive of pancreatitis. Prerenal azoternia due to hemoconcentra-tion and poor renal perfusion is a common complication of pancreatitis. Likewise, so is hepatocellular injury and cholestasis.

Urinalysis Urine specifi c gravity of 1.035 indicates the dog is capable of concentrating, thus the increase in BUN is prerenal (perhaps dehydration). Glucosuria and bilirubinuria are to he expected in light of the serum concentrations.

Endocrine data ACTH stimulation test: Baseline cortisol is slightly above normal. Normal animals stimulate to around 10 to 16 μ g/dL. Low dose dexamethasone suppression test: Baseline cortisol is normal. Dog suppressed marginally at 8 hours. The endo-crine data are not supportive of hyper - adrenocorticism.

Summary

This dog has primary hyperlipidemia, which has been shown to be familial in miniature schnauzers (Rogers WA, EF Donovan, GJ Kociba. Idiopathic hyperlipoproteinenia in dogs. J Am Vet Med Assoc 1975;166:1087 – 1091), and pan-creatitis with secondary diabetes mellitus. Dogs with hyper-lipidemia are predisposed to development of pancreatitis. While diabetes mellitus may be transitory, treatment is indi-cated. Some abnormalities (hyperglycemia, stress leuko-gram, increased alkaline phosphatase activity, lipemia, history, and physical appearance) were suggestive of hyper-adrenocorticism. This possibility was ruled out by the ACTH stimulation and LDDS test. Imaging revealed evidence of swelling in the area of the pancreas.

Interpretive discussion

Hematology Lymphopenia is indicative of increased endogenous (stress or hyperadrenocorticism) or exogenous corticosteroids. Increased immature neutrophil concentration is indicative of infl ammation. Neutrophilia may be due to infl ammation or stress. In summary, an infl ammatory and stress (steroid) leukogram is present.

Biochemical profi le Hyperglycemia is of the magnitude that diabetes mellitus should be suspected. Hyperglycemia may also be secondary to hyperadrenocorticism; therefore, adrenocorticotropic hormone (ACTH) stimulation and low dose dexamethasone suppression tests (LDDS) are indicated.

BUN is increased, but creatinine is within the reference interval. Urine specifi c gravity indicates kidneys are capable of concentrating, thus the azotemia is prerenal, perhaps clue to dehydration. However, albumin is within reference inter-val. The PCV is normal, suggesting that GI bleeding is not the cause of the increased BUN.

Total bilirubin is increased suggesting cholestasis, because anemia is not present. Alkaline phosphatase activity is increased, which is also suggestive of cholestasis. Another consideration is hyperadrenocorticism, with an increase in the corticosteroid - induced alkaline phosphatase isoenzyme. Increased cholesterol of this magnitude is probably due to lipidemia, although some component of the increase could also be due to cholestasis. ALT activity is increased, which is indicative of hepatocellular damage.

Sodium and chloride concentrations are decreased. Sodium may be lost through the kidney, although this animal is capable of concentrating. Although it is not men-tioned in the history, abdominal pain may have been associ-ated with vomiting, which would result in electrolyte loss. Hyperglycemia results in increased serum osmolality with a shift of intracellular fl uid to extracellular fl uid in an attempt to decrease extracellular fl uid solute concentration. Sodium can be expected to decrease by 1.6 mEq/L for every 100 mg/dL increase in glucose.

Total CO 2 is decreased, indicating metabolic acidosis. The anion gap is increased, indicating increased unmeasured anions are present. In this case, unmeasured anions might be ketones, although they are not present in the urine. Urine

Page 91: VI Clinical Case Presentations - Wiley · Clinical Case Presentations ... Case 17: Dog with vitamin K antagonist (rodenticide) toxicity, 643 ... PT Prothrombin time

Case 52

705

CA

SES

Case 52 Signalment: 9 - year - old SF canine, miniature schnauzer History: Not eating, vomited a few times Physical examination: Tense abdomen

Hematology Reference Interval

PCV (%) 32.0 37 – 55

MCV (fL) 68.0 60 – 72

NCC ( × 10 3 / μ L) 5.2 6 – 17

Segs ( × 10 3 / μ L) 2.7 3 – 11.5

Bands ( × 10 3 / μ L) 1.4 0 – 0.3

Monos ( × 10 3 / μ L) 0.2 0.1 – 1.3

Lymphs ( × 10 3 / μ L) 0.6 1 – 4.8

Basophils ( × 10 3 / μ L) 0.1 rare

Platelets ( × 10 3 / μ L) 111 200 – 500

Hemopathology: marked toxic neutrophils, glant platelets, hemolyzed

and lipemic

Biochemical Profi le

Reference Interval

Gluc (mg/dL) 226 (12.4) 65 – 122 (3.5 – 6.7 mmol/L)

BUN (mg/dL) 20 7 – 28

Creat (mg/dL) 1.2 0.9 – 1.7

Ca (mg/dL) 8.2 (2.0) 9.0 – 11.2 (2.2 – 2.8 mmol/L)

Phos (mg/dL) 5.1 2.8 – 6.1

TP (g/dL) 5.0 5.4 – 7.4

Alb (g/dL) 1.8 2.7 – 4.5

Glob (g/dL) 3.2 1.9 – 3.4

T. Bili (mg/dL) 1.4 (23.9) 0 – 0.4 (0.6 – 8.4 μ mol/L)

Chol (mg/dL) 666 (17.3) 130 – 370 (3.4 – 9.6 mmol/L)

ALT (IU/L) 33 10 – 120

AST (IU/L) 51 16 – 40

ALP (IU/L) 1282 35 – 280

GGT (IU/L) 5 0 – 6

Na (mEq/L) 152 145 – 158

K (mEq/L) 3.7 4.1 – 5.5

CL (mEq/L) 116 106 – 127

TCO 2 (mEq/L) 14 14 – 27

An. gap (mEq/L) 25 8 – 25

Amylase (IU/L) 2421 50 – 1250

Lipase (IU/L) 2256 30 – 560

Triglycerides (mg/dL) 2884 ND *

* Not Determined

Urinalysis

Color Golden Urine Sediment

Transparency Cloudy WBCs/hpf 2 – 3

Sp. Gr. 1.034 RBCs/hpf 3 – 5

Protein 2 + Epith cells/hpf Negative

Gluc 4 + Casts/lpf 2

Bilirubin 3 + Crystals Negative

Blood 2 + Bacteria Negative

pH 8.0

ketones Negative

Coagulation Data

Reference Interval

PT (seconds) 9.3 7.5 – 10.5

aPTT (seconds) 19.5 10.5 – 16.5

Abdominal Fluid Analysis

Color Red

Supernatant Light yellow

Refractometric protein (g/dL) 7.2

NCC ( × 10 3 / μ L) 2.0

Triglyceride (mg/dL) 257

Chol (mg/dL) 728

Page 92: VI Clinical Case Presentations - Wiley · Clinical Case Presentations ... Case 17: Dog with vitamin K antagonist (rodenticide) toxicity, 643 ... PT Prothrombin time

SECTION VI Clinical Case Presentations

706

CA

SES

longed hypertipidemia. The concurrent fi ndings of hyperlipidemia and pancreatitis in a miniature schnauzer should alert one to the potential diagnosis of a primary dyslipidemia.

Coagulation data The coagulation profi le includes a normal PT, but prolonged APTT. While it is more common for the PT to become pro-longed fi rst when there is impaired coagulation factor syn-thesis by the liver, incipient DIC (note the thrombocytopenia) or heparinization of the patient may result in changes in the APTT alone.

Abdominal fl uid analysis Abdominal fl uid chemical analysis similarly indicates accu-mulation of excess lipids in the peritoneal cavity. It is likely that the increased total protein by refractometry is spuri-ously elevated by this lipid. The cell concentration suggests a modifi ed transudate.

Urinalysis The urine specifi c gravity indicates that the kidneys are capable of concentrating, and the number of leukocytes and erythrocytes are not signifi cant. However, there is 2 + pro-teinuria, some occult blood, and some hyaline and fi ne granular casts. Thus, there may be mild tubular and/or glo-merular disease. In addition, there is signifi cant glucosuria, which is explained by the hyperglycemia, It would be useful to evaluate the UPC in order to determine the magnitude of the proteinuria. Given the hypoalbuminemia and hypercho-lesterolemia, one should consider, the possibility of nephrotic syndrome; there may be a protein - losing glomerulopathy without azotemia.

Summary

Minature schnauzer hyperlipidemia and acute pancreatitis.

Interpretive discussion

Hematology The PCV is mildly decreased, no polychromasia was noted in the blood fi lm, and the MCV is normal, indicating a mild nonregenerative anemia. Marked lipemia and hemolysis may have resulted in in vitro hemolysis, but this typically does not result in an important decrease in the PCV. There is a neutropenia with increased bands and marked numbers of toxic neutrophils. This suggests consumption as a result of severe infl ammatory disease. Lymphopenia indicates a stress component. The thrombocytopenia is discussed with the coagulation data.

Biochemical profi le The serum glucose concentration is moderately increased. In this range, it is possible that this is a stress hyperglycemia, but is more likely due to some metabolic or endocrine abnormality.

The BUN and serum creatinine concentrations are normal. The serum phosphorus is normal, but there is a mild decrease in serum total calcium concentration. Given the degree of hypoalbuminemia, it is wise to attempt to correct the total calcium for the hypoproteinemia. In this case, the corrected value is 9.9 mg/dL (8.2 – 1.8 + 3.5), which is normal.

The serum cholesterol concentration is markedly increased. While this may be associated with cholestasis, given the degree of increase in cholesterol one should also consider other metabolic abnormalities including hepatic disease, dis-orders of lipoprotein metabolism, or endocrinopathies. The serum triglyceride concentration is markedly increased, and further supports a diagnosis of a metabolic and/or endocri-nologic disorder. Cholestasis is indicated by the increased total bilirubin and ALP activity. The serum ALT, AST, and GGT activities are normal or near normal, reducing the likelyhood of hepatocellular injury,

The serum amylase and lipase activities are signifi cantly increased, and in the absence of azotemia suggest acute pancreatitis. This is a frequent complication of severe pro-

Page 93: VI Clinical Case Presentations - Wiley · Clinical Case Presentations ... Case 17: Dog with vitamin K antagonist (rodenticide) toxicity, 643 ... PT Prothrombin time

Case 53

707

CA

SES

Case 53 Signalment: 11 - year - old castrated male cat History: Polyuria and polydipsia for 2 months, anorexia and lethargy more recently Physical examination: Presented in lateral recumbency, 10% dehydrated

Hematology Reference Interval

PCV (%) 40 24 – 45

Hgb (g/dL) 12.8 8 – 15

RBC ( × 10 6 / μ 1) 8.64 5 – 11

MCV (fL) 46 39 – 50

MCHC (g/dL) 34 33 – 37

NCC ( × 10 3 / μ 1) 18.7 5.5 – 19.5

Segs ( × 10 3 / μ 1) 15.0 2.5 – 12.5

Bands ( × 10 3 / μ 1) 2.4 0 – 0.3

Monos ( × 10 3 / μ 1) 0.2 0 – 0.8

Lymphs ( × 10 3 / μ 1) 0.9 1.5 – 7.0

Eos ( × 10 3 / μ 1) 0.2 0 – 1.5

Platelets ( × 10 3 / μ 1) 375 200 – 500

TP (P) (g/dL) 11.7 6 – 8

Hemopathology: Slightly toxic neutrophils, many echinocytes.

Biochemical Profi le

Reference Interval

Gluc (mg/dL) 766 (42.7) 67 – 124 (3.7 – 6.8 mmol/L)

BUN (mg/dL) 127 (45.3) 17 – 32 (6.1 – 11.4 mmol/L)

Creat (mg/dL) 6.4 (566) 0.9 – 2.1 (78 – 186 μ mol/L)

Ca (mg/dL) 10.1 8.5 – 11

Phos (mg/dL) 7.9 (10.0) 3.3 – 7.8 (1.1 – 2.5 mmol/L)

TP (g/dL) 9.7 5.9 – 8.1

Alb (g/dL) 4.4 2.3 – 3.9

Glob (g/dL) 5.3 2.9 – 4.4

T. Bili (mg/dL) 0.3 0 – 0.3

Chol (mg/dL) 388 (10.1) 60 – 220 (1.6 – 5.7 mmol/L)

ALT (IU/L) 124 30 – 100

AST (IU/L) 354 14 – 38

ALP (IU/L) 65 6 – 106

GGT (IU/L) 1 0 – 1

Na (mEq/L) 172 146 – 160

K (mEq/L) 5.1 3.7 – 5.4

CL (mEq/L) 132 112 – 129

TCO 2 (mEq/L) 10.9 14 – 23

An. gap (mEq/L) 34 10 – 27

Calc. Osmolarity

(mOsm/L)

417 290 – 310

Urinalysis (cystocentesis)

Color Yellow Urine Sediment

Transparency Cloudy WBCs/hpf 6 – 8

Sp. Gr. 1.034 RBCs/hpf 2 – 3

Protein 2 + Epith cells/hpf 1 – 3 transitional

Gluc 2 + Casts/lpf 0

Bilirubin Negative Crystals 0

Blood 4 + Bacteria 0

pH 5.0 Ketones Negative

Other Small amt of fat

Interpretive discussion

Hematology Leukogram abnormalities include neutrophilia, a left shift, lymphopenia, and slightly toxic neutrophils. This is an infl ammatory leukogram indicating a tissue demand for neutrophils. The lymphopenia suggests concurrent increase in corticosteroid concentrations due to stress. Toxic neutro-phils indicate a rapid rate of neutrophil production.

Echinocyte formation can be an artifact, but in this case, it may have resulted from the marked hyperosmolality and electrolyte abnormalities. These may have caused move-ment of water from the cytoplasm of erythrocytes to the plasma with resulting shrinkage and crenation of erythrocytes.

Biochemical profi le The serum glucose concentration is markedly increased. The most likely cause of hyperglycemia of this magnitude is diabetes mellitus. Severe, acute excitement with release of catecholamines can cause marked hyperglycemia in cats, but serum glucose concentration is seldom greater than 400 mg/dL in such cats. This cat is azotemic, and decreased renal excretion of glucose, secondary to decreased glomerular fi l-tration rate, may have augmented the magnitude of the hyperglycemia. Moreover, the cat does not have an excite-ment leukogram (lymphocytosis).

Both BUN and serum creatinine concentrations are increased. Since the urine specifi c gravity suggests adequate renal concentrating ability (i.e., the specifi c gravity is greater than 1.030), this appears to be a prerenal ozotemia. However, the marked hyperproteinemia and hypernatremia suggest severe dehydration, and an even higher urine specifi c gravity would be expected in this situation. It is, therefore, possible that this cat has some loss of urine concentration ability. Alternatively, osmotic diuresis due to glucosuria may have contributed to the lower than expected urine specifi c gravity. The hyperphosphatemia is a result of a decreased glomerular fi ltration rate. Maintenance of normal serum phosphorus

Page 94: VI Clinical Case Presentations - Wiley · Clinical Case Presentations ... Case 17: Dog with vitamin K antagonist (rodenticide) toxicity, 643 ... PT Prothrombin time

SECTION VI Clinical Case Presentations

708

CA

SES

cause of this abnormality. In this cat, which is apparently not ketotic, increased blood lactate concentration is probably contributing to this gap.

The calculated osmolarity is increased and, in combination with other laboratory changes, suggests this cat has diabetic nonketotic hyperosmolar syndrome (see summary).

Urinalysis This cat has a proteinuria with a mild pyuria. It is possible that the protein exuded into the urine as part of the infl am-matory process; however, the degree of proteinuria appears to be excessive compared to the degree of pyuria. Other causes of proteinuria such as glomerular and tubular disease should be considered in this case. Although glomerular disease has been associated with diabetes mellitus in humans, this has not been documented in animals.

The strongly positive reaction on the chemical test for blood in combination with normal numbers of erythrocytes suggests that the positive reaction is due to either free hemo-globin or myoglobin. It is unlikely that this represents a hematuria with subsequent lysis of erythrocytes since such lysis is unlikely in urine with a high specifi c gravity. Absence of anemia suggests a signifi cant hemolytic problem is not occurring in this cat. Myoglobinuria is a possible explana-tion, and severe muscle hynoxia secondary to hypovolemia may have occurred in this cat. However, the serum AST activity, while increased, does not suggest such massive muscle injury.

Glucosuria is a result of the serum glucose concentration exceeding the renal threshold.

Summary

The clinical diagnosis was diabetic nonketotic hyperosmolar syndrome. This syndrome is characterized by marked hyper-glycemia (blood glucose concentration > 600 mg/dL), hyper-osmolarity ( > 350 mOsm/l), and absence of ketosis in a diabetic animal. Such animals commonly have prerenal or renal azotemia. The hyperosmolarity results in dehydration of neurons and subsequent neurologic signs. This syndrome is associated with a high fatality rate.

After a brief, unsuccessful attempt to decrease serum glucose concentrations with insulin therapy and to improve the cat ’ s electrolyte and fl uid balance by administration of fl uids, the owner elected euthanasia. Necropsy revealed severs islet cell degeneration and amyloidosis and severe hepatocytic vacuolar degeneration. A few mineralized casts were present in renal tubules, but the kidneys were other-wise normal, and the azotemia was probably prerenal in this case. The cause of the infl ammatory leukogram was not determined.

concentrations depends on phosphorus excretion through the kidney.

Hyperproteinemia (both plasma and serum protein) with concurrent hyperalbuminemia and hyperglobulinemai is typical of dehydration. Contraction of plasma water volume results in proportional increases in concentrations of both albumin and globulin. Although other abnormalities can cause hyperglobulinemia, dehydration is the only cause of hyperalbuminemia. Diuresis secondary to glucosuria is common in diabetes mellitus and can result in dehydration.

The serum cholesterol concentration is increased. In this case, this abnormality is probably secondary to diabetes mel-litus and related abnormalities in lipid metabolism.

Serum activities of both ALT and AST are increased. The increased serum ALT activity is due to hepatocyte injury and subsequent leakage of this enzyme. This injury was probably caused by fatty change which developed secondary to the metabolic abnormalities of diabetes mellitus. The increased serum AST activity may also be due to leakage of AST from injured hepatocytes, but the higher activity of AST as com-pared to ALT suggests that there is also an extrahepatic source. This source may be muscle, and may have resulted from muscle injury secondary to hypoperfusion, since the cat is very dehydrated.

Hypernatremia and hyperchloremia are probably due to severe dehydration. Glucosuria causes diuresis resulting in Na and Cl loss through the kidneys in nondehydrated or mildly dehydrated, diabetic animals. This can lead to hypo-natremia and hypochloremia. When such animals become severely dehydrated, however, diuresis no longer occurs, and hypernatremia and hyperchloremia develop. These changes, in combination with hyperglycemia and azotemia, result in severe hyperosmolality.

Decreased serum total CO 2 concentration probably rep-resents a primary metabolic acidosis. Serum total CO 2 con-centration may also decrease as a compensatory reaction in animals with primary respiratory alkalosis, but in animals with diabetes mellitus, metabolic acidosis is more likely to be the primary alteration. Increased serum con-centrations of ketones are a common cause of acidosis in diabetic animals, but the absence of urine ketones suggests that this cat is probably not ketotic. Urine ketone tests that use the nitroprusside reaction do not detect β - hydroxybutyric acid, therefore, the presence of this ketone cannot be ruled out. Increased serum lactate concentration may be contributing to the acidosis in this cat. The cat is markedly dehydrated and is, therefore, probably experi-encing tissue hypoxia which may lead to increased lactate production.

The anion gap is increased. In most diabetic animals, increased ketoacid concentration in the blood is the major

Page 95: VI Clinical Case Presentations - Wiley · Clinical Case Presentations ... Case 17: Dog with vitamin K antagonist (rodenticide) toxicity, 643 ... PT Prothrombin time

Case 54

709

CA

SES

Case 54 Signalment: 10 - year - old MC feline DSH History: Not eating well, lethargic Physical examination: Slightly dehydrated

Hematology Day1 Reference Interval

PCV (%) 38.0 * 24 – 45

Hgb (g/dL) 12.8 8 – 15

RBC ( × 10 6 / μ L) 9.25 5 – 11

MCV (fL) 44.0 39 – 50

MCHC (g/dL) 35.0 33 – 37

Retics ( × 10 3 / μ L) 80,000 0 – 60,000

NCC ( × 10 3 / μ L) 12.9 5.5 – 19.5

Segs ( × 10 3 / μ L) 12.5 2.5 – 12.5

Lymphs ( × 10 3 / μ L) 0.3 1.5 – 7.0

Platelets ( × 10 3 / μ L) Adequate 200 – 500

TP (P) (g/dL) 9.0 6 – 8

Hemopathology: giant platelets, slight increase in polychromasia,

slightly toxic neutrophils, 2 + Heinz bodies

* PCV was 27% on Day 5, and 17% on Day 7

Biochemical Profi le

Day 1 Reference Interval

Gluc (mg/dL) 328 (18.0) 67 – 124 (3.7 – 6.8 mmol/L)

BUN (mg/dL) 29 17 – 32

Creat (mg/dL) 1.5 0.9 – 2.1

Ca (mg/dL) 9.4 8.5 – 11

Phos (mg/dL) 1.9 (0.6) 3.3 – 7.8 (1.1 – 2.5 mmol/L)

TP (g/dL) 8.0 5.9 – 8.1

Alb (g/dL) 4.3 2.3 – 3.9

Glob (g/dL) 3.7 2.9 – 4.4

T. Bili (mg/dL) 2.1 (35.9) 0 – 0.3 (0 – 5.1 mmol/L)

Chol (mg/dL) 512 (13.3) 60 – 220 (1.6 – 5.7 mmol/L)

ALT (IU/L) 282 30 – 100

ALP (IU/L) 99 6 – 106

Na (mEq/L) 130 146 – 160

K (mEq/L) 2.2 3.7 – 5.4

CL (mEq/L) 74 112 – 129

TCO 2 (mEq/L) 10.5 14 – 23

An. gap (mEq/L) 47.7 10 – 27

Lipase (IU/L) 161 3 – 125

Blood Gas Data (arterial)

Reference Interval

pH 7.280 7.33 – 7.44

PCO 2 (mmHg) 20.0 35 – 42

PO 2 (mmHg) 85.5 73 – 92

HCO 3 (mEq/L) 9.2 16 – 22

ionized Ca + + (mg/dL) 4.64 4.8 – 5.3

Urinalysis

Color Yellow Urine Sediment

Transparency Clear WBCs/hpf 0 – 1

Sp. Gr. 1.033 RBCs/hpf 0 – 1

Protein 1 + Epith cells/hpf 0 – 1

Gluc 4 + Casts/lpf 3 – 4 granular

Bilirubin 1 + Crystals Negative

Blood 1 + Bacteria Negative

pH 6.0 Other

Ketones 3 +

Page 96: VI Clinical Case Presentations - Wiley · Clinical Case Presentations ... Case 17: Dog with vitamin K antagonist (rodenticide) toxicity, 643 ... PT Prothrombin time

SECTION VI Clinical Case Presentations

710

CA

SES

Interpretive discussion

Hematology The packed cell volume, hemoglobin, and total RBC count are normal, but given the degree of hemoconcentration represented by the hyperproteinemia, it is possible that the PCV is actually lower. There is a slight increase in polychro-masia and mild reticulocytosis. The anemia is rapidly pro-gressive over a 1 week period of time. The presence of 2 + Heinz bodies indicates signifi cant oxidative damage to the red blood cells, and is commonly observed in cats with dia-betic ketoacidosis; however, the owner should be questioned as to whether the cat has received acetaminophen or other oxidant drugs or chemicals. Another potential cause of hemolytic anemia in this patient is hypophosphatemia. There is a stress leukogram, as indicated by the high normal neutrophil count and lymphopenia.

Biochemical profi le The serum glucose concentration is moderately increased. While a glucose concentration of this magnitude may be encountered due to extreme excitement (sympathetic acti-vation) or stress (glucocorticoid release), diabetes mellitus is more likely. Evidence against excitement - induced hypergly-cemia is the lack of an excitement leukogram (lymphocyto-sis). The BUN and serum creatinine concentrations are normal.

The serum phosphorus concentration is decreased, and given the degree of hyperglycemia, one should consider diabetic ketoacidosis - induced urinary phosphate loss. The serum total calcium concentration is normal, reducing the possibility of an endocrine abnormality causing the change in serum phosphorus. The serum total protein concentration is at the upper end of the reference interval, and serum albumin is increased, indicating hemoconcentration due to dehydration.

The serum cholesterol concentration is moderately increased. While this may be associated with cholestasis, as indicated by the increased total bilirubin, the ALP activity is normal. Given the degree of increase in cholesterol, one should consider metabolic abnormalities including hepatic disease, disorders of lipoprotein metabolism, or endocrinop-athies. If not due to cholestasis, then the increase in bilirubin may be due to hemolysis. The serum ALT activity is increased modestly which indicates hepatocellular damage. ALP is not

induced by steroids in cats, thus, hyperadrenocorticism is a possibility. Serum lipase activity is only slightly increased, possibly reducing the probability for concurrent pancreatitis; however, increased lipase activity is not a reliable marker for feline pancreatitis.

Serum Na, K, and Cl concentration are decreased signifi -cantly. One should consider typical causes for electrolyte depletion, including pathologic losses from the gastrointes-tinal and urinary systems, as well as a shift to third space. The marked hyperglycemia should initiate consideration of diabetic ketoacidosis with subsequent urinary electrolyte loss. There is a marked decrease in serum total CO, suggest-ing metabolic acidosis. The increase in the anion gap is likely due to the presence of ketones, which are unmeasured anions.

Blood gas data The blood gas panel indicates a metabolic acidosis (decreased pH and HCO 3 ) with respiratory compensation (decreased pCO 2 ). Ionized calcium is marginally decreased.

Urinalysis The urinary specifi c gravity is normal. However, with marked increases in the concentration of solutes, such as glucose, not pertinent to urinary concentration capacity, one might question the accuracy of this measure, and consider deter-mining urinary osmolality to address urinary concentration capacity specifi cally. The presence of 1 + protein and coarse granular casts is consistent with renal tubular disease. The absence of more signifi cant proteinuria speaks against the possibility of glomerular protein loss, but a urinary protein : creatinine ratio should be determined to confi rm this. In either case, urinary tract infl ammation is not a likely cause of the observed changes, as there is only a small amount of occult blood and no pyuria. The presence of sig-nifi cant amounts of glucose and ketones supports a diagnosis of diabetic ketoacidosis. The mild bilirubinuria is a result of the increased serum bilirubin and subsequent renal excretion.

Summary

Diabetic ketoacidosis; Heinz body anemia

Page 97: VI Clinical Case Presentations - Wiley · Clinical Case Presentations ... Case 17: Dog with vitamin K antagonist (rodenticide) toxicity, 643 ... PT Prothrombin time

Case 55

711

CA

SES

Interpretive discussion

Hematology The only abnormalities are a mild mature neutrophilia and lymphopenia, consistent with a stress/steroid leukogram. This is not surprising given the history of prednisone administration.

Biochemical profi le Increased ALP and GGT activities are most likely due to steroid induction given the history of prednisone adminis-tration. There is no other evidence to support cholestasis or liver disease.

Hypoglycemia is pronounced. Appropriate sample han-dling procedures were followed, ruling - out artifactual hypo-glycemia due to delayed removal of serum from the red cells, and episodic hypoglycemia fi ts the clinical signs. There is no evidence for sepsis (no infl ammatory leukogram) or liver failure (BUN, cholesterol, albumin are normal).

Serum insulin concentration is increased at the same time this dog is hypoglycemic, which is an inappropriate response. Normally, feedback mechanisms result in low serum insulin concentrations when hypoglycemia exists. Uncontrolled insulin production from a neoplasm is most likely. The most common tumor associated with hypoglycemia in dogs is insulinoma, a neoplasm of pancreatic β - cells.

Summary

An exploratory laparotomy was performed and a small pan-creatic mass was identifi ed and removed. Small nodules were present in the liver, and regional lymph nodes were enlarged. Aspirates from an enlarged node were taken intra-operatively, and a metastatic endocrine tumor was diag-nosed by cytology. Histopathology confi rmed a β - cell carcinoma in the pancreas with metastases to liver and lymph node. It is important to measure serum insulin concentrations at the same time the dog is hypoglycemic, preferably when the blood glucose is < 50 mg/dL. Under these conditions, a serum insulin concentration that is increased or in the upper half of the reference interval indi-cates a relative insulin excess, suggesting uncontrolled insulin production.

Hematology Reference Interval

Packed cell volume (%) 40 37 – 55

Hemoglobin (g/dL) 14 12 – 18

RBC ( × 10 6 / μ L) 5.75 5.5 – 8.5

MCV (fL) 69 60 – 72

MCHC (g/dL) 35 34 – 38

Total nucleated cell count ( × 10 3 / μ L) 14.5 6 – 17

Segmented neutrophils ( × 10 3 / μ L) 12.5 3 – 11.5

Band neutrophils ( × 10 3 / μ L) 0 0 – 0.3

Monocytes ( × 10 3 / μ L) 1.3 0.1 – 1.3

Lymphocytes ( × 10 3 / μ L) 0.7 1 – 4.8

Eosinophils ( × 10 3 / μ L) 0 0.1 – 1.2

Platelets ( × 10 3 / μ L) 463 200 – 500

Plasma protein (g/dL) 7.0 6 – 8

Case 55 Signalment Eight - year - old CM Labrador retriever History Two months of decreased activity, progressing to muscle fasciculations and mild intermittent seizures. Referring DVM prescribed phenobarbital and prednisone 5 days previously. Physical examination Obese, reluctant to move. Normal body temperature, heart rate, respiratory rate. Normal chest radiographs and abdominal ultrasound.

Biochemical Profi le Reference Interval

Glucose (mg/dL) 24 65 – 122

Blood Urea Nitrogen (mg/dL) 16 7 – 28

Creatinine (mg/dL) 1.2 0.9 – 1.7

Calcium (mg/dL) 10.5 9.0 – 11.2

Phosphorus (mg/dL) 4.5 2.8 – 6.1

Total Protein (g/dL) 6.8 5.4 – 7.4

Albumin (g/dL) 3.5 2.7 – 4.5

Globulin (g/dL) 3.3 1.9 – 3.4

Total Bilirubin (mg/dL) 0.3 0 – 0.4

Cholesterol (mg/dL) 256 130 – 370

Alanine aminotransferase (IU/L) 110 10 – 120

Aspartate aminotransferase (IU/L) 32 16 – 40

Alkaline phosphatase (IU/L) 602 13 – 141

Gamma glutamyl transferase (IU/L) 9 0 – 6

Sodium (mEq/L) 151 145 – 158

Potassium (mEq/L) 4.1 4.1 – 5.5

Chloride (mEq/L) 116 106 – 127

Total CO 2 (mEq/L) 17 14 – 27

Anion Gap 22 8 – 25

Other

Serum insulin ( μ U/mL) 46.2 5 – 25

Page 98: VI Clinical Case Presentations - Wiley · Clinical Case Presentations ... Case 17: Dog with vitamin K antagonist (rodenticide) toxicity, 643 ... PT Prothrombin time

SECTION VI Clinical Case Presentations

712

CA

SES

Case 56 Signalment: 6 - day - old female Holstein History: Scours Physical examination: Severe dehydration

Hematology Reference Interval

PCV (%) 58.0 24 – 46

Hgb (g/dL) 19.0 8 – 15

RBC ( × 10 6 / μ L) 17.1 5.0 – 10.0

MCV (fL) 34.0 37 – 53

MCHC (g/dL) 33.0 33 – 38

NCC ( × 10 3 / μ L) 5.0 4.0 – 12.0

Segs ( × 10 3 / μ L) 3.2 0.6 – 4.0

Monos ( × 10 3 / μ L) 1.7 0 – 0.8

Lymphs ( × 10 3 / μ L) 0.1 2.5 – 7.5

Platelets ( × 10 3 / μ L) 288 200 - 800

Fibrinogen (mg/dL) 600 200 – 600

TP (P) (g/dL) 10.9 6 – 8

Hemopathology: many acanthocytes and keratocytes, RBC fragments,

hypochromic RBCs.

Biochemical Profi le

Reference Interval

Gluc (mg/dL) 31 55 – 95

BUN (mg/dL) 87 7 – 20

Creat (mg/dL) 4.6 1.0 – 1.8

Ca (mg/dL) 7.8 8.2 – 9.9

Phos (mg/dL) 6.9 4.3 – 7.0

TP (g/dL) 10.3 6.3 – 7.6

Alb (g/dL) 5.3 2.5 – 4.3

Glob (g/dL) 5.0 2.6 – 5.0

T. Bili (mg/dL) 0.8 0.1 – 0.4

CK (IU/L) 352 57 – 280

AST (IU/L) 286 40 – 130

GGT (IU/L) 14 10 – 26

SDH (IU/L) 17 8 – 23

Na (mEq/L) 129 136 – 147

K (mEq/L) 6.7 3.6 – 5.2

CL (mEq/L) 91 95 – 105

TCO 2 (mEq/L) 17.0 24 – 32

An. gap (mEq/L) 27.7 14 – 26

Blood Gas Data (venous)

Reference Interval

pH 7.140 7.32 – 7.45

pCO 2 (mmHg) 45.7 34 – 44

HCO 3 (mEq/L) 15.3 23 – 31

Urinalysis

Color Yellow Urine Sediment

Transparency Clear WBCs/hpf 0 – 1

Sp. Gr. 1.014 RBCs/hpf 0 – 1

Protein Negative Epith cells/hpf 1 – 2

Gluc Negative Casts/lpf Negative

Bilirubin Negative Crystals Negative

Blood Negative Bacteria Negative

pH 5.0

Page 99: VI Clinical Case Presentations - Wiley · Clinical Case Presentations ... Case 17: Dog with vitamin K antagonist (rodenticide) toxicity, 643 ... PT Prothrombin time

Case 56

713

CA

SES

commonly results from increased sodium chloride loss induced by the enterotoxin that promotes active secretion into the gut lumen. Increased water loss follows this osmotic gradient. Bicarbonate is also lost in the feces, and hypovo-lemia may lead to tissue hypoperfusion, lactic acidosis, and decreased bicarbonate concentration as well. Fecal potas-sium loss is typically increased, but concomitant metabolic acidosis results in exchange of intracellular potassium for extraceliular protons, and a redistributional hyperkalemia.

Blood gas data There is a combined metabolic (decreased bicarbonate) and respiratory (increased pCO 2 ) acidosis. The metabolic acidosis results from bicarbonate loss in the diarrhea and from lactic acidosis due to tissue hypoperfusion. The increased anion gap refl ects the accumulation of unmeasured anions such as lactate. The mild respiratory acidosis indicates pulmonary dysfunction. Early pneumonia or decreased pulmonary per-fusion secondary to dehydration are possible explanations.

Urinalysis The only signifi cant abnormality is a urine specifi c gravity of 1.014. Six - day - old calves, unlike neonates of many other species, should have mature capacity to concentrate urine. Dehydration should stimulate antidiuretic hormone release from the hypothalamus, and increased water reclamation by the renal tubules. However, electrolyte loss in this type of hypotonic dehydration often leads to medullary solute depletion and a loss of the renal concentration gradient. Another alternative is that there is renal disease, due to renal hypoperfusion, sepsis, etc., resulting in both azotemia and loss of concentrating ability.

Summary

Secretory diarrhea and hypotonic dehydration in a neonatal calf.

Interpretive discussion

Hematology There is a monocytosis and a lymphopenia that represent the effects of stress. The plasma protein concentration is increased, most probably due to dehydration. Erythrocyte indices refl ect hemoconcentration as well, as evidenced by the increased RBC count, hemoglobin concentration, and PCV. The MCV is decreased, which may be due to an under-lying iron - defi ciency anemia of the newborn that is obscured by hemoconcentration. The presence of several erythrocyte morphologic abnormalities supports this. Iron defi ciency is frequently associated not only with a microcytic anemia, but also with oxidative damage to the erythrocytes, resulting in membrane abnormalities and fragmentation changes.

Biochemical profi le There is a profound hypoglycemia, which in a neonatal calf with diarrhea is most probably related to decreased food intake, as well as the possibility of sepsis. Sepsis is unlikely, considering the normal neutrophil concentration.

The BUN and serum creatinine concentrations are increased, but the origin of this azotemia cannot be dis-cerned from this data alone. Refer to the discussion in the urinalysis section below.

Serum calcum is mildly decreased, possibly due to decreased milk intake. The serum total protein and albumin concentrations are increased, further refl ecting hemocon-centration due to dehydration. The serum CK and AST activities are modestly increased, which may be related to muscle damage subsequent to prolonged recumbency or hypoperfusion. The total bilirubin is increased. Together with the increased AST activity, this may indicate hepatocel-lular damage. Alternatively, there may be cholestasis due to dehydration or prehepatic icterus due to increased destruc-tion of oxidatively - damaged iron - defi cient erythrocytes.

The serum sodium and chloride concentrations are decreased, refl ecting decreased intake and/or increased loss from the body. E. coli - associated diarrhea in neonatal calves

Page 100: VI Clinical Case Presentations - Wiley · Clinical Case Presentations ... Case 17: Dog with vitamin K antagonist (rodenticide) toxicity, 643 ... PT Prothrombin time

SECTION VI Clinical Case Presentations

714

CA

SES

Case 57 Signalment: 9 - month - old bull History: Anorexia, depression Physical examination: Enlarged abdomen

Interpretive discussion

Hematology PCV is decreased, indicating anemia.

MCV is decreased, suggesting iron defi ciency anemia sec-ondary to chronic blood loss.

Neutrophilia, increased band neutrophils, and monocyto-sis are indicative of chronic infl ammation.

Increased fi brinogen also suggests infl ammation. Keratocytes and schistocytes are commonly seen with iron

defi ciency anemia.

Biochemical profi le Glucose is markedly increased, perhaps a sympathoadrenal response that can be seen in severely ill cattle. Other pos-sibilities include prior treatment with glucose - containing fl uids, diabetes mellitus, or acute pancreatitis. Other lab data supports proximal duodenal obstruction, in which marked hyperglycemia is a consistent fi nding. This may be due to a combination of stress - induced hyperglycemia and poor peripheral perfusion, so that the glucose isn ’ t used. Also low K may result in decreased cell uptake of glucose.

BUN, creatinine, and phosphorus are increased. Urine specifi c gravity would help determine if renal or pre - renal. Because of severe dehydration as indicated by increased albumin, at least a pre - renal component is likely. Phospho-rus may also be increased due to high GI obstruction, which is likely the diagnosis.

Calcium is slightly decreased. Phosphorus is excreted in the saliva of ruminants; with GI obstruction, elimination of phosphorus via the GI tract is decreased. Mild hypocalcemia has been reported with abomasal and forestomach disease.

Total protein and albumin are increased, indicating dehy-dration. Globulin is increased, which may be due to dehy-dration or antigenic stimulation.

Bilirubin is increased, which in this patient may be due to cholestasis or anorexia.

Serum creatine kinase activity is increased, probably indicative of myopathy. AST is mildly increased, either from myopathy or hepatocellular damage.

Marked hypochloremia is probably due to abomasal acid secretion into the lumen. Obstruction of abomasal out fl ow and distention exacerbates. Chloride is decreased more than would be expected with abomasal displacement or volvulus; this degree of hypochloremia is indicative of high GI obstruc-tion. Potassium is likely decreased for the same reason.

Sodium is low and is perhaps being lost in urine. This may be due to hyperglycemia resulting in osmotic diuresis and thus increasing urinary losses of electrolytes.

Hyperosmolality may also be contributing to hyponatre-mia, as a result of cellular water moving into extracellular

Hematology Reference Interval

PCV (%) 19 24 – 46

MCV (fL) 31 37 – 53

NCC ( × 10 3 / μ L) 18.0 4.0 – 12.0

Segs ( × 10 3 / μ L) 10.5 0.6 – 4.0

Bands ( × 10 3 / μ L) 2.5 0 – 0.1

Monos ( × 10 3 / μ L) 1.0 0 – 0.8

Lymphs ( × 10 3 / μ L) 3.5 2.5 – 7.5

Eos ( × 10 3 / μ L) 0.5 0 – 2.4

Platelets ( × 10 3 / μ L) Adequate 200 – 800

Fibrinogen (mg/dL) 1000 200 – 600

Hemopathology: Numerous schistocytes, keratocytes

Biochemical Profi le

Reference Interval

Gluc (mg/dL) 618 55 – 95

BUN (mg/dL) 90 7 – 20

Creat (mg/dL) 6.1 1.0 – 1.8

Ca (mg/dL) 7.8 8.2 – 9.9

Phos (mg/dL) 14.1 4.3 – 7.0

TP (g/dL) 10.1 6.3 – 7.6

Alb (g/dL) 4.5 2.5 – 4.3

Glob (g/dL) 5.6 2.6 – 5.0

T. Bili (mg/dL) 0.8 0.1 – 0.4

CK (IU/L) 1100 57 – 280

AST (IU/L) 350 40 – 130

Na (mEq/L) 130 136 – 147

K (mEq/L) 3.1 3.6 – 5.2

CL (mEq/L) 47 95 – 105

TCO 2 (mEq/L) 50 24 – 32

Blood Gas Data (Venous)

Reference Interval

HCO 3 (mEq/L) 49.3 23 – 31

pH 7.412 7.32 – 7.45

pCO 2 (mmHg) 80 34 – 44

Page 101: VI Clinical Case Presentations - Wiley · Clinical Case Presentations ... Case 17: Dog with vitamin K antagonist (rodenticide) toxicity, 643 ... PT Prothrombin time

Case 57

715

CA

SES

fl uid compartment, diluting serum sodium (1.6 mEq/L decrease in Na for every 100 mg/dL increase in glucose)

Total CO 2 and HCO 3 are increased, indicating marked hypochloremic metabolic alkalosis. pH is in the high normal range as a result of compensatory respiratory acidosis (increased pCO 2 ). Remarkable hypochloremia and alkalosis indicated that there is obstruction of abomasal out fl ow, preventing re - exchange of chloride and bicarbonate.

Increased anion gap (36 mEq/L) also indicates increase in unmeasured anions. Most of the anions contributing to this are not truly “ unmeasured, ” but are the increased phos-phates and protein. Additionally, there may be increased lactate due to decreased tissue perfusion, or increased sulfates due to tissue breakdown.

Summary

This animal had a high GI obstruction (foreign body), thus explaining many of the abnormalities.

Azotemia was probably prerenal due to dehydration, although there are abnormalities in distal tubular transport which may be due to hypochloremia; osmotic diuresis may be also contributing to these abnormalities.

Infl ammation is present, perhaps associated with the GI obstruction.

Iron defi ciency anemia from chronic blood loss is present (perhaps abomasal ulcer, GI parasites).

Other tests that should be performed include urinalysis, especially specifi c gravity, and fecal occult blood.

Page 102: VI Clinical Case Presentations - Wiley · Clinical Case Presentations ... Case 17: Dog with vitamin K antagonist (rodenticide) toxicity, 643 ... PT Prothrombin time

SECTION VI Clinical Case Presentations

716

CA

SES

Case 58

Hematology Reference Interval

PCV (%) 51 24 – 46

NCC ( × 10 3 / μ L) 19.7 4.0 – 12.0

Segs ( × 10 3 / μ L) 11.4 0.6 – 4.0

Monos ( × 10 3 / μ L) 2.0 0 – 0.8

Lymphs ( × 10 3 / μ L) 6.3 2.5 – 7.5

Platelets ( × 10 3 / μ L) Adequate 200 – 800

Signalment: 9 - day - old female Holstein History: Several days duration of diarrhea, anorexia, extreme weakness Physical examination: Hypothermic, 12% dehydrated

Interpretive discussion

Hematology There is a neutrophilia and monocytosis, indicating an infl ammatory leukogram. The PCV is increased, refl ecting hemoconcentration due to dehydration.

Biochemical profi le There is a profound hypoglycemia, which in a neonatal calf with diarrhea is most probably related to decreased food intake, as well as the possibility of sepsis. Considering the increased neutrophil concentration, sepsis is unlikely.

The BUN and serum creatinine concentrations are increased, but the origin of this azotemia cannot be dis-cerned without a urinalysis. However, given the other evi-dence of hemoconcentration, prerenal azotemia is the most likely cause. Although higher serum phosphorus concentra-tions are common in young animals, this degree of hyper-phosphatemia is more likely related to decreased glomerular fi ltration rate. There is a marked hypocalcemia, but this may be due solely to the hypoalbuminemia; i.e., the ionized calcium concentration may be normal, but the protein - bound fraction is decreased.

There is marked hypoproteinemia, despite the severe degree of dehydration. This is due both to hypoalbuminemia and hypoglobulinemia. The former may be due to liver disease, inanition, or intestinal loss associated with the diar-rhea. The latter is very likely due to lack of passive transfer, which would have subsequently predisposed this neonate to infections, resulting in diarrhea and sepsis.

The increased serum CK and AST activities may be due to muscle damage, to prolonged recumbency, or hypoperfu-sion. The very slight increase in GGT activity may be due to absorption of a small amount of colostrum, which is high in GGT activity in ruminants.

The increased serum sodium and chloride indicate that this calf is hypertonically dehydrated. One typically expects hypotonic dehydration to develop in a neonatal calf with scours, owing to electrolyte loss in the secretory diarrhea. Thus, it is more likely that this is not a secretory diarrhea, but rather another infectious cause of diarrhea, with or without septicemia. Water, loss in excess of solute may be compounded by reduced water consumption, increased insensible losses due to fever, and/or exudation (along with albumin) across a damaged intestinal mucosa. Although there may have been signifi cant potassium loss in

Biochemical Profi le

Reference Interval

Gluc (mg/dL) 46 55 – 95

BUN (mg/dL) 63 7 – 20

Great (mg/dL) 3.7 1.0 – 1.8

Ca (mg/dL) 5.9 8.2 – 9.9

Phos (mg/dL) 14.5 4.3 – 7.0

TP (g/dL) 3.0 6.3 – 7.6

Alb (g/dL) 1.9 2.5 – 4.3

Glob (g/dL) 1.1 2.6 – 5.0

T. Bili (mg/dL) 0.2 0.1 – 0.4

CK (IU/L) 7819 57 – 280

AST (IU/L) 177 40 – 130

. GGT (IU/L) 28 10 – 26

Na (mEq/L) 158 136 – 147

K (mEq/L) 7.9 3.6 – 5.2

CL (mEq/L) 117 95 – 105

TCO 2 (mEq/L) 15 24 – 32

An. gap (mEq/L) 33.9 14 – 26

Blood Gas Data (Venous)

Reference Interval

pH 7.140 7.32 – 7.45

pCO 2 (mmHg) 45.7 34 – 44

HCO 3 (mEq/L) 15.3 23 – 31

Page 103: VI Clinical Case Presentations - Wiley · Clinical Case Presentations ... Case 17: Dog with vitamin K antagonist (rodenticide) toxicity, 643 ... PT Prothrombin time

Case 58

717

CA

SES

the diarrhea, redistributional hyperkalemia is commonly observed in cases like this owing to exchange of intracellular potassium for extracellular protons (H + ) in response to the metabolic acidosis. The respiratory acidosis suggests inade-quate pulmonary perfusion.

Blood gas data There is a combined metabolic (decreased bicarbonate) and respiratory (increased pCO 2 ) acidosis. The metabolic acidosis results from bicarbonate loss in the diarrhea and from lactic

acidosis due to tissue hypoperfusion. The increased anion gap refl ects the accumulation of unmeasured anions like lactate.

Summary

Nonsecretory diarrhea and hypertonic dehydration in a neo-natal calf following failure of passive transfer.

Page 104: VI Clinical Case Presentations - Wiley · Clinical Case Presentations ... Case 17: Dog with vitamin K antagonist (rodenticide) toxicity, 643 ... PT Prothrombin time

SECTION VI Clinical Case Presentations

718

CA

SES

Hematology 8/8 8/17 Reference Interval

Packed cell volume (%) 28 22 40 – 55

Hgb (g/dL) 8.9 7.0 13 – 20

RBC (106/ μ L) 3.81 2.95 5.5 – 8.5

MCV (fL) 73 73 62 – 73

MCHC (g/dL) 32 32 33 – 36

Total nucleated cell count

( × 10 3 / μ L)

17.3 17.9 4.5 – 15

Segmented neutrophils

( × 10 3 / μ L)

14.4 (83%) 16.3 2.6 – 11

Band neutrophils ( × 10 3 / μ L) 0.3 (2%) 0 – 0.2

Monocytes ( × 10 3 / μ L) 0.3 (2%) 1.3 0.2 – 1.0

Lymphocytes ( × 10 3 / μ L) 1.9 (11%) 0.2 1 – 4.8

Eosinophils( × 10 3 / μ L) 0.2 0.1 – 1.2

Platelets ( × 10 3 / μ L) 302 323 200 – 500

Plasma protein (g/dL) 2.5 2.9 6 – 8

Reticulocytes ( × 10 3 ) 80,000

(2.1%)

209,450

(7.1%)

0 – 60,000

Hemopathology: slight polychromasia, slt toxic neuts

Case 59 Signalment Six - year - old male castrated male fox terrier. History Intermittent vomiting and diarrhea for past seven weeks, seizured for a few minutes a few hours before presentation on 8/8. Physical examination Lethargic, weak, “ bloated ”

Biochemical Profi le slightly hemolyzed

Reference Interval

Glucose (mg/dL) 123 99 75 – 130

Blood Urea Nitrogen (mg/dL) 8 9 7 – 28

Creatinine (mg/dL) 0.5 0.4 0.7 – 1.9

Calcium (mg/dL) 4.4 5.1 9.0 – 11.7

Phosphorus (mg/dL) 1.7 1.5 2.1 – 6.0

Magnesium (mg/dL) 0.9 0.7 1.8 – 2.5

Total Protein (g/dL) 2.1 2.3 5.4 – 7.4

Albumin (g/dL) 1.2 1.2 2.7 – 4.5

Globulin (g/dL) 0.9 1.1 2.0 – 3.8

Total Bilirubin (mg/dL) 0.1 0.1 0 – 0.3

Cholesterol (mg/dL) 69 71 130 – 300

Alanine aminotransferase (IU/L) 600 274 10 – 110

Aspartate aminotransferase (IU/L) 540 163 16 – 50

Alkaline phosphatase (IU/L) 660 405 20 – 142

Creatine kinase (IU/L) 1343 356 50 – 275

GGT (IU/L) 77 108 0 – 9

Sodium (mEq/L) 136 140 142 – 152

Potassium (mEq/L) 2.9 3.6 3.5 – 5.2

Chloride (mEq/L) 106 109 108 – 120

Bicarbonate (mEq/L) 17.6 20.6 16 – 25

Anion Gap (mEq/L) 15 14 13 – 22

Calc. osmolality (mosm/L) 267 275 284 – 304

Iron 100 140 75 – 280

TIBC 110 153

Sat percent 91 92

UIBC < 10 13

Ionized calcium (mmol/L) 0.96 0.80 1.30 – 1.46

Coagulation Reference Interval

Protime (sec) 17.5 7.5 – 10.5

APTT (sec) 52.9 10.5 – 16.5

Antithrombin 45% of normal pooled sera

Page 105: VI Clinical Case Presentations - Wiley · Clinical Case Presentations ... Case 17: Dog with vitamin K antagonist (rodenticide) toxicity, 643 ... PT Prothrombin time

Case 59

719

CA

SES

enteropathy is usually because of hypoalbuminemia, decreased ionized calcium may also occur in these patients as a result of decreased GI calcium absorbtion and also because of decreased vitamin D absorption. Hypomagnese-mia may also result in hypocalcemia by leading to decreased synthesis or release of PTH or by leading to decreased responsiveness to PTH by skeletal and renal tissue. More-over, hypomagnesemia can cause decreased activation of vitamin D by kidneys.

The hypomagnesemia is severe and in this case is likely due to loss through the GI tract. Decreased vitamin D may result in decreased magnesium absorption.

(Other possible causes of hypomagnesemia include loss through the kidney or shifts of magnesium from extracel-lular to intracellular, but there is no evidence for renal disease. Other disorders associated with hypomagnesemia include diuresis, diabetic ketoacidosis, pancreatitis, sepsis, and primary hyperparathyroidism, but again, there is no history or evidence of these.)

Hypocholesterolemia is usually due to either decreased intake through the GI tract, or decreased production as a result of liver failure. In this patient, it is likely due to increased intake, secondary to protein losing enteropathy.

Increased ALT and AST in later profi les are indicative of hepatocellular damage, although the AST may be increased due to muscle damage, as CK is increased. ALT may be induced by steroids, as well.

Increased GGT and ALP may be due to cholestasis or induced by corticosteroids. The serum alkaline phosphatase and GGT were not increased in the initial biochemical profi le performed by the referring veterinarian. Additional history revealed that the dog had been receiving injectable gluco-corticosteroids, which are likely responsible for the increase in these enzymes. The dog likely has a steroid hepatopathy at this time.

Hyponatremia and hypokalemia are also likely due to losses through the GI tract.

The increase in APPT and PT are likely due to decreased vitamin K absorption, as it is also a fat soluble vitamin.

The decreased antithrobin III is likely due to concurrent albumin losses.

Additional tests needed Parathyroid hormone assay, vitamin D assay.

Summary

The most likely diagnosis in this dog is protein losing enter-opathy with resulting severe hypocalcemia and hypomag-nesemia, and likely secondary hypoparathyroidism.

Interpretive discussion

Hematology Regenerative anemia is present as evidenced by reticulocy-tosis. This may be due to blood loss or blood destruction. Hypomagnesemia has been reported to cause hemolytic anemia in humans. The total protein is not useful to dif-ferentiate hemolysis from loss in this case, as it is likely decreased due to other causes. Blood loss could be though the GI tract.

An infl ammatory leukogram is present as evidenced by neutrophilia and increased bands on 8/8.

A stress leukogram is present on 8/17 as evidenced by mature neutrophilia and lymphopenia. A monocytosis may be a component of a stress leukogram.

Plasma protein by refractometry is markedly decreased (see biochemical profi le interpretation).

Biochemical profi le Numerous biochemical profi les were performed over several weeks. The common abnormalities are marked hypopro-teinemia, hypoalbuminemia, hypoglobulinemia, and hypo-calcemia, Hypocholesterolemia is present in three of the profi les, as well. All but the fi rst biochemical profi le also showed increased ALT, AST, ALP, and CK. Sodium and potassium are decreased in the last three profi les, and serum magnesium is markedly decreased. Serum creatinine is also decreased on the last two profi les. These abnormalities are discussed below.

Panhypoproteinemia The most likely differential for a decrease of this magnitude in both serum albumin and globulin, particularly in a dog with a history of diarrhea, is protein - losing enteropathy. Other possible causes would be blood loss or loss of protein into the abdominal cavity due to infl ammation. The PCV is not low enough to explain this degree of hypoproteinemia.

Hypocalcemia While some of the hypocalcemia can be explained by hypo-albuminemia, this cannot account for this degree of hypocalcemia. When corrected for hypoalbuminemia, the calcium is still low. (For example, in the profi le on 8/17, 5.1 − 1.2 + 3.5 = 7.4.) The decreased ionized calcium further substantiates that both ionized and bound calcium are decreased.

There is probably no differential for this degree of hypo-calcemia other than the hypocalcemia that may occur in patients with protein losing enteropathy, since primary hypoparathyroidism should result in increased phosphorus concentration. While hypocalcemia with protein losing

Page 106: VI Clinical Case Presentations - Wiley · Clinical Case Presentations ... Case 17: Dog with vitamin K antagonist (rodenticide) toxicity, 643 ... PT Prothrombin time

SECTION VI Clinical Case Presentations

720

CA

SES

Yorkshire trriers : fi ve cases (1992 – 1998) . J Am Vet Med Assoc

217 : 703 – 6 .

Bush WW , Kimmel Se , Wosar MA , et al . ( 2001 ) Secondary hypo-

parathyroidism attributed to hypomagnesemia in a dog with

protein - losing enteropathy . J Am Vet Med Assoc 219 : 1732 – 4 .

Explains in detail how hypomagnesemia results in hypoparathy-

roidism (decreases cAMP generation and blunts release of

PTH) and also results in blunted end organ response to PTH by

decreasing phosphoinositide - specifi c phospholipase C in cell

membranes.

Outcome PTH was low normal and vitamin D was decreased. Small bowel biopsy showed mixed infl ammation, dilated crypts, necrosis, bacterial overgrowth, and mild lymphangiectasia. Dog had positive occult blood in feces. It was treated by changing diet (Eukanuba low - residue dry), Prednisone (1.3 mg/kg bid), Tums, and enrofl oxacin (Baytril), and coconut oil (10 – 20 mL per day added to food).

Further reading

Kimmel SE , Waddel LS , Michel KE ( 2000 ) Hypomagnesemia and

hypocalcemia associated with protein - losing enteropathy in

Page 107: VI Clinical Case Presentations - Wiley · Clinical Case Presentations ... Case 17: Dog with vitamin K antagonist (rodenticide) toxicity, 643 ... PT Prothrombin time

Case 60

721

CA

SES

Case 60

Hematology Reference Interval

Packed cell volume (%) 37 36 – 54

Plasma protein (g/dL) 5.2 5.4 – 7.2

Signalment Ten - year - old, spayed female, mixed breed dog History Chronic weight loss, chronic voluminous diarrhea, ravenous appetite Physical Bright and alert, thin with a 1/5 body condition score, 5% dehydrated.

Biochemical Profi le Reference Interval

Glucose (mg/dL) 94 77 – 126

Blood Urea Nitrogen (mg/dL) 17 5 – 20

Creatinine (mg/dL) 1.1 0.6 – 1.6

Calcium (mg/dL) 9.7 9.3 – 11.6

Phosphorus (mg/dL) 4.1 3.2 – 8.1

Total Protein (g/dL) 5.3 5.1 – 7.1

Albumin (g/dL) 3.0 2.9 – 4.2

Globulin (g/dL) 2.3 2.2 – 2.9

Total Bilirubin (mg/dL) 0.1 0.1 – 0.4

Cholesterol (mg/dL) 49 80 – 315

ALT (IU/L) 44 10 – 55

AST( IU/L) 23 12 – 40

ALP (IU/L) 66 15 – 120

Creatine Kinase (IU/L) 81 50 – 400

Sodium (mEq/L) 145 143 – 153

Potassium (mEq/L) 4.1 4.1 – 5.4

Chloride (mEq/L) 112 109 – 120

TCO 2 (meq/L) 22 16 – 25

Anion Gap 15 15 – 25

Other Data

Bile acids – fasting ( μ mol/L) 1.0 < 15.5

Bile acids – post prandial ( μ mol/L) 7 5 – 20

Folate ( μ g/L) 20.4 7.7 – 24.4

Cobalamine - B12 (ng/L) 154 251 – 908

Trypsin - Like Immunoreactivity

(TLI) – fasting ( μ g/L) 0.2 5.7 – 45.2

Fecal fl oat negative

Page 108: VI Clinical Case Presentations - Wiley · Clinical Case Presentations ... Case 17: Dog with vitamin K antagonist (rodenticide) toxicity, 643 ... PT Prothrombin time

SECTION VI Clinical Case Presentations

722

CA

SES

Interpretive discussion

The decrease in the plasma protein may be compatible with loss of protein or decreased protein production. On the bio-chemical profi le, total protein, albumin and globulins are at the low end of the reference interval. Because the dog is dehydrated, it is possible that these values will decrease below the reference interval when the animal is rehydrated. Given the history of chronic diarrhea and the low body condition score, protein losing enteropathy or exocrine pancreatic insuffi ciency (EPI) are primary differential diagnoses.

The hypocholesterolemia can be the result of malabsorp-tion, maldigestion, protein losing enteropathy, or liver failure. Given the normal liver enzymes and total bilirubin, there is no evidence of hepatocellular damage or cholestasis. However, liver enzymes do not measure liver function: fasting and postprandial bile acids were done to evaluate for hepatic insuffi ciency. Given the normal fasting and post-prandial bile acids, decreased production of cholesterol sec-ondary to hepatic insuffi ciency is unlikely.

The low normal total protein, albumin, and globulin in conjunction with hypocholesterolemia is compatible with protein losing enteropathy or EPI resulting in malabsorption and maldigestion, respectively. Both of these conditions are associated with diarrhea and weight loss. Differentiating these two conditions requires additional testing. The normal serum folate in conjunction with decreased cobalamnie - B12

is compatible with distal small intestinal disease or EPI. The low TLI is diagnostic for EPI.

Summary

This dog was diagnosed with exocrine pancreatic insuffi -ciency (EPI), and she responded to dietary supplementation with pancreatic enzymes and cobalamine injections. The syndrome of EPI results from inadequate production and release of pancreatic enzymes into the intestinal tract. Mal-digestion and malabsorption of nutrients results in diarrhea that is often voluminous. Steatorrhea and hypocholesterol-emia are consequences of maldigestion and malabsorption of fats. Unlike patients with protein losing enteropathy, many patients with EPI maintain their serum protein within the reference interval. Intrinsic factor is required for absorp-tion of dietary cobalamine/vitamin B12. Because the pan-creas is the source of intrinsic factor in the dog, cobalamine defi ciency may develop as a result of malabsorption second-ary to EPI and is refl ected in the low serum cobalamine levels seen in this patient. Pancreatic ascinar atrophy is the most common cause of EPI in younger dogs and is seen most commonly in the German shepherd breed. When pancreatic ascinar atrophy occurs in an older dog such as this patient, other causes of EPI such as pancreatitis or neoplasia should be explored.

Page 109: VI Clinical Case Presentations - Wiley · Clinical Case Presentations ... Case 17: Dog with vitamin K antagonist (rodenticide) toxicity, 643 ... PT Prothrombin time

Case 61

723

CA

SES

Interpretive discussion

Hematology A mild nonregenerative, normocytic, normochromic anernia is the only abnormality in the CRC. “ Target cells ” are common and are not very diagnostically useful. They com-monly present in animals with hypercholesterolemia.

Biochemical profi le The only abnormalities present are hypercholesterolemia and mild hyponatremia. Hypercholesterolemia is marked, and in conjunction with the history, physical examination, and mild anemia, is very suggestive of hypothyroidism. Mild hyponatremia has been reported in approximately 30% of dogs with hypothyroidism.

Endocrine data Total T4 is within the reference interval. However, since many variables affect TT4, and this dog has clinical and laboratory fi ndings that are suggestive of hypothyroidism a free T4 and endogenous TSH are indicated. The decreased FT4 and increased endogenous TSH are diagnostic for hypothyroidism.

Summary

Early primary hypothyroidism.

Case 61 Signalment: 3 - year - old castrated male golden retriever History: Lethargic, heat seeking Physical examination: Obese, poor hair coat, tailhead alopecia

Hematology Reference Interval

PCV (%) 34 37 – 55

MCV (fL) 65 60 – 72

MCHC (g/dL) 35 34 – 38

Retics ( × 10 3 / μ L) 2 < 60

NCC ( × 10 3 / μ 1) 12.5 6 – 17

Segs ( × 10 3 / μ 1) 9.3 3 – 11.5

Monos ( × 10 3 / μ 1) 1.0 0.1 – 1.3

Lymphs ( × 10 3 / μ 1) 2.2 1 – 4.8

Platelets ( × 10 3 / μ 1) Adequate 200 – 500

TP (P) (g/dL) 7.5 6 – 8

Hemopathology: Numerous leptocytes ( “ target cells ” ) present

Biochemical Profi le

Reference Interval

Gluc (mg/dL) 105 65 – 122

BUN (mg/dL) 20 7 – 28

Creat (mg/dL) 1.2 0.9 – 1.7

Ca (mg/dL) 10.5 9.0 – 11.2

Phos (mg/dL) 4.0 2.8 – 6.1

TP (g/dL) 7.0 5.4 – 7.4

Alb (g/dL) 3.7 2.7 – 4.5

Glob (g/dL) 3.3 1.9 – 3.4

T. Bili (mg/dL) 0.2 0 – 0.4

Chol (mg/dL) 720 (18.7) 130 – 370 (3.4 – 9.6 mmol/L)

ALT (IU/L) 110 10 – 120

AST (IU/L) 35 16 – 40

ALP (IU/L) 220 35 – 280

Na (mEq/L) 143 145 – 158

K (mEq/L) 4.5 4.1 – 5.5

CL (mEq/L) 107 106 – 127

TCO 2 (mEq/L) 20 14 – 27

Endocrine Data Reference Interval

TT4 ( μ g/dL) 1.6 1.4 – 4.0

Free T4 (ng/dL) 0.24 (3.0) 1.2 – 3.4 (15.4 – 4.8 pmol/L)

Endogenous TSH

(ng/mL) (Immulite)

0.5 0.1 – 0.45

Page 110: VI Clinical Case Presentations - Wiley · Clinical Case Presentations ... Case 17: Dog with vitamin K antagonist (rodenticide) toxicity, 643 ... PT Prothrombin time

SECTION VI Clinical Case Presentations

724

CA

SES

Case 62 Signalment: 13 - year - old MC dog History: Polyuria, frequent urination with small volumes Physical examination: Overweight

Hematology Reference Interval

PCV (%) 36.0 37 – 55

Hgb (g/dL) 13.4 12 – 18

RBC ( × 10 6 / μ L) 5.26 5.5 – 8.5

MCV (fL) 69.0 60 – 72

MCHC (g/dL) 37.0 34 – 38

NCC ( × 10 3 / μ L) 18.1 6 – 17

Segs ( × 10 3 / μ L) 16.7 3 – 11.5

Monos ( × 10 3 / μ L) 1.3 0.1 – 1.3

Lymphs ( × 10 3 / μ L) 0.2 1.0 – 4.8

Platelets ( × 10 3 / μ L) 452 200 – 500

TP (P) (g/dL) 8.2 6 – 8

Hemopathology: few Howell - Jolly bodies

Biochemical Profi le Reference Interval

Gluc (mg/dL) 806 (44.3) 65 – 122 (3.5 – 6.7 mmol/L)

BUN (mg/dL) 81 (28.9) 7 – 28 (2.5 – 10.0 mmol/L)

Creat (mg/dL) 1.6 0.9 – 1.7

Ca (mg/dL) 8.4 (2.1) 9.0 – 11.2 (2.25 – 2.80 mmol/L)

ionized Ca + + (mg/dL) 3.56 4.5 – 5.6

Phos (mg/dL) 7.2 (2.3) 2.8 – 6.1 (0.9 – 2.0 mmol/L)

TP (g/dL) 6.0 5.4 – 7.4

Alb (g/dL) 3.3 2.7 – 4.5

Glob (g/dL) 2.7 1.9 – 3.4

T. Bili (mg/dL) 1.3 (22.2) 0 – 0.4 (0 – 6.8 μ mol/L)

Chol (mg/dL) 467 (12.1) 130 – 370 (3.4 – 9.6 mmol/L)

ALT (IU/L) 1355 0 – 120

AST (IU/L) 341 16 – 40

ALP (IU/L) 4660 35 – 280

GGT (IU/L) 373 0 – 6

CK (IU/L) 266 50 – 250

Na (mEq/L) 144 145 – 158

K (mEq/L) 3.8 4.1 – 5.5

CL (mEq/L) 98 106 – 127

TCO 2 (mEq/L) 18.5 14 – 27

An. gap (mEq/L) 31.3 8 – 25

Amylase (IU/L) 1687 50 – 1250

Lipase (IU/L) 3746 30 – 560

Urinalysis

Color Yellow Urine Sediment

Transparency Cloudy WBCs/hpf 50 – 100

Sp. Gr. 1.014 RBCs/hpf > 100

Protein 2 + Epith cells/hpf Negative

Gluc 4 + Casts/lpf Negative

Bilirubin Negative Crystals Negative

Blood 4 + Bacteria 3 + rods

pH 5.0

ketones Negative

Coagulation Data

Reference Interval

PT (seconds) 7.5 7.5 – 10.5

aPPT (seconds) 18.2 10.5 – 16.5

FDPs ( μ g/mL) 1:12 < 1:10

Endocrine Data Reference Interval

Free T4 (ng/dL) < 0.15 1.2 – 3.4

total T4 ( μ g/dL) 0.85 1.5 – 3.5

endog TSH (ng/mL) 0.05 0.1 – 0.45

Interpretive discussion

Hematology The PCV and total RBC count are marginally decreased, with no abnormalities in red blood cell size, hemogiobin content, or morphology. One should consider recent blood loss (par-ticularly GI hemorrhage) even though plasma protein con-centration is normal. Alternatively, there may be a mild normochromic, normocytic anemia associated with renal failure. The leukocyte count is increased, with a mature neutrophilia and lymphopenia. This is a stress leukogram, and may support the possibility of hyperadrenocorticism as part of the disease process.

Page 111: VI Clinical Case Presentations - Wiley · Clinical Case Presentations ... Case 17: Dog with vitamin K antagonist (rodenticide) toxicity, 643 ... PT Prothrombin time

Case 62

725

CA

SES

hyperglycemia should initiate consideration of diabetic keto-acidosis with subsequent urinary electrolyte loss. However, although the anion gap is increased, the serum total CO 2 is normal. It is possible that there are concurrent causes for metabolic acidosis (ketoacidosis) and metabolic alkalosis (vomiting and/or gastrointestinal stasis).

Urinalysis The urinary specifi c gravity is in the isosthenuric range, despite azotemia and hyperphosphatemia. This may be the result of renal disease, or impaired concentrating ability due to electrolyte depletion and loss of the medullary concentra-tion gradient. There is signifi cant proteinuria, pyuria, hema-turia, and bacteriuria which most likely indicate a bacterial infection and infl ammatory response in the urinary tract. In the absence of tubular casts or information regarding enzy-muria or urinary fractional excretion of electrolytes, it is diffi cult to specify the anatomic location of this disorder. There is signifi cant glucosuria corresponding to the marked hyperglycemia rioted earlier. The absence of ketones on the dipstick speaks against the possibility of prominent ketoaci-dosis (and ketonuria) noted above. However, this test does not detectg one of the ketones, β - hydroxybutyric acid. One can anticipate that detectable ketosis would develop if untreated.

Coagulation data The coagulation profi le indicates a slightly prolonged APTT and mildly increased FDP concentration. This may be the result of liver disease (although one may expect a change in PT prior to one in the APTT), or incipient DIC (although platelet concentration is usually decreased with DIC). If liver disease was severe enough to impair coagulation factor syn-thesis, one would fi rst expect to see hypoalburninemia and/or hypocholesterolemia. It is not possible to draw conclu-sions with these borderline abnormalities.

Endocrine data Low free T4, low total T4, and low endogenous TSH are diagnostic for secondary hypothyroidism. Secondary hypo-thyroidism as a result of decreased endogenous TSH is com-monly associated with diabetes mellitus.

Summary

Diabetes mellitus and secondary hypothyroidism.

Biochemical profi le The serum glucose concentration is markedly increased. This is well beyond the level encountered due to excitement (sympathetic activation) or stress (glucocorticoid release), and should immediately suggest diabetes mellitus.

The BUN is disproportionately increased relative to the mild increase in serum creatinine concentration. The BUN : creatinine ratio is 50.6, which should suggest gastro-intestinal hemorrhage, leading to an increase in hepatic urea production. Nevertheless, some degree of azotemia (prere-nal, renal, or postrenal) is probably also present (refer to discussion of urinalysis below). The serum phosphorus is moderately increased, and may be associated with the impaired glomerular fi ltration and the azotemia. Because the serum total calcium concentration is also decreased, one should consider dietary imbalance or renal disease as causes of secondary hyperparathyroidism. See discussion of ionized Ca below.

The serum total protein and albumin concentrations are normal. Unless there is a concomitant cause for hypopro-teinemia, the absence of hyperproteinemia decreases the probability for hemoconcentration and prerenal azotemia due to dehydration.

The serum cholesterol is moderately increased. This may be related to cholestasis, as indicated by the moderate increase in serum to biliruhin concentration and serum ALP and GGT activities. However, the degree of increase in cho-lesterol is suffi cient to warrant consideration of abnormali-ties in lipoprotein metabolism owing to hepatic disease or an endocrine abnormality. Likewise, the degree of increase in ALP and GGT activities suggests other means for their induction beyond cholestasis, such as hyperadrenocorticism. Marked increases in serum ALT and AST activities indicate hepatocellular damage, which may have contributed to the increases in ALP and GGT activities. The serum CK activity is essentially normal, and rules out the potential contribu-tion of muscle damage to serum AST and ALT increases. Hepatic lipidosis associated with diabetes should be consid-ered as a cause of hepatocellular injury and cholestasis.

A marginal increase in serum amylase and marked increase in serum lipase activities may indicate the presence of pancreatitis. However, concurrent azotemia may impair renal extraction of these enzymes from the serum, leading to increases in their activities.

Serum Na, K, and Cl concentrations are decreased. One should consider typical causes for electrolyte depletion, including pathologic losses from the gastrointestinal and urinary systems, as well as a shift to third space. The marked

Page 112: VI Clinical Case Presentations - Wiley · Clinical Case Presentations ... Case 17: Dog with vitamin K antagonist (rodenticide) toxicity, 643 ... PT Prothrombin time

SECTION VI Clinical Case Presentations

726

CA

SES

Case 63 Signalment: 3 - year - old MC English springer spaniel History: Anorexia, occasional vomiting Physical examination: Lethargic, thin, approximately 8% dehydrated

Hematology Reference Interval

PCV (%) 32 37 – 55

Hgb (g/dL) 11.1 12 – 18

RBC ( × 10 6 / μ 1) 4.47 5.5 – 8.5

MCV (fL) 72 60 – 72

MCHC (g/dL) 35 34 – 38

Retics ( × 10 3 / μ L) ND * < 60

NCC ( × 10 3 / μ 1) 9.8 6 – 17

Segs ( × 10 3 / μ 1) 5.6 3 – 11.5

Monos ( × 10 3 / μ 1) 0.8 0.1 – 1.3

Lymphs ( × 10 3 / μ 1) 2.2 1.0 – 4.8

Eos ( × 10 3 / μ 1) 1.2 0.1 – 1.2

Platelets ( × 10 3 / μ 1) Adequate 200 – 500

TP (P) (g/dL) 8.5 6 – 8

* Not Determined

Biochemical Profi le

Reference Interval

Gluc (mg/dL) 83 65 – 122

BUN (mg/dL) 47 (16.8) 7 – 28 (2.5 – 10.0 mmol/L)

Creat (mg/dL) 1.6 0.9 – 1.7

Ca (mg/dL) 13.8 (3.45) 9.0 – 11.2 (2.25 – 2.80 mmol/L)

Phos (mg/dL) 6.2 (2.0) 2.8 – 6.1 (0.9 – 2.0 mmol/L)

TP (g/dL) 7.5 5.4 – 7.4

Alb (g/dL) 5.0 2.7 – 4.5

Glob (g/dL) 2.5 1.9 – 3.4

T. Bili (mg/dL) 0.2 0 – 0.4

Chol (mg/dL) 135 130 – 370

ALT (IU/L) 49 10 – 120

AST (IU/L) 19 16 – 40

ALP (IU/L) 98 35 – 280

Na (mEq/L) 132 145 – 158

K (mEq/L) 5.5 4.1 – 5.5

CL (mEq/L) 97 106 – 127

TCO 2 (mEq/L) 10 14 – 27

An. gap (mEq/L) 30 8 – 25

Amylase (IU/L) 1300 50 – 1250

Lipase (IU/L) 570 30 – 560

Endocrine Data Reference Interval

ACTH stimulation

serum cortisol ( μ g/dL)(pre) < 0.1 ( < 2.8) 1 – 4 (28 – 100 nmol/L)

serum cortisol ( μ g/dL)

(post)

< 0.1 ( < 2.8) < 10.5 ( < 290 nmol/L)

Urinalysis

Urine specifi c gravity 1.020

Page 113: VI Clinical Case Presentations - Wiley · Clinical Case Presentations ... Case 17: Dog with vitamin K antagonist (rodenticide) toxicity, 643 ... PT Prothrombin time

Case 63

727

CA

SES

Interpretive discussion

Hematology A mild anemia is present. Reticulocyte concentration was not determined, thus the degree of regeneration is unknown. Increased polychromasia is not mentioned, suggesting that the anemia is nonregenerative; however, the MCV is at the upper end of the reference interval, suggesting the presence of large immature erythrocytes. Considering the degree of dehydration, anemia is likely more severe than is apparent.

While the leukogram is normal, a patient that is ill and vomiting would be expected to have a stress leukogram. The absence of a stress leukogram should prompt consideration of hypoadrenocorticism.

Plasma protein is increased, probably as a result of dehydration.

Biochemical profi le Azotemia is evidenced by increased BUN, creatinine, and phosphorus concentrations. While azotemia may be pre - renal, since the dog is dehydrated, one would expect the urine specifi c gravity to be greater than 1.030, if this were the case. However, the serum sodium concentration is decreased, and ability to concentrate is impaired by medul-lary washout of sodium. Refer to the discussion on sodium and potassium for further interpretation.

Hypercalcemia, in light of hyponatremia and hyperkale-mia, is likely due to hypoadrenocorticism. The pathophysiol-ogy may be related to decreased glucocorticoids and

subsequent increased GI calcium uptake, calcium retention by the kidney, as related to sodium loss, or increased albumin - bound calcium. Other causes of hypercalcemia, such as hypercalcemia of malignancy, primary hyperpara-thyroidism, and vitamin D toxicosis are much less likely in this patient.

Mild hyperproteinemia, due to hyperalbuminemia, is due to dehydration.

Hyponatremia and high normal potassium should cause suspicion of Addison ’ s disease. While these electrolyte abnormalities are not marked, and result in a Na : K ratio of 24, they should prompt an ACTH stimulation test. Hypona-tremia and hyperkalemia in this patient, on the other hand, could be a result of renal disease. Hypochloridemia is con-sistent with hyponatremia. Low total CO 2 is consistent with metabolic acidosis, and the anion gap is increased due to increased unmeasured anions, which in this dehydrated hypovolemic patient are probably lactic acids.

Mild increase in serum amylase and lipase activities are probably secondary to decreased glomerular fi ltration.

Endocrine data The immeasurably low cortisol concentration with a “ fl at - line ” response to ACTH confi rms hypoadrenocorticism

Summary

Hypoadrenocorticism

Page 114: VI Clinical Case Presentations - Wiley · Clinical Case Presentations ... Case 17: Dog with vitamin K antagonist (rodenticide) toxicity, 643 ... PT Prothrombin time

SECTION VI Clinical Case Presentations

728

CA

SES

Case 64 Signalment: 8 - month - old intact male dog History: Suddenly collapsed during grooming; bloody diarrhea Physical examination: Extreme weakness, bradycardia, and cool extremities

Hematology Reference Interval

PCV (%) 42 37 – 55

Hgb (g/dL) 13.3 12 – 18

RBC ( × 10 6 / μ L) 6.6 5.5 – 8.5

MCV (fL) 64 60 – 72

MCHC (g/dL) 32 34 – 38

NCC ( × 10 3 / μ L) 12.0 6 – 17

Segs ( × 10 3 / μ L) 7.2 3 – 11.5

Monos ( × 10 3 / μ L) 0.6 0.1 – 0.3

Lymphs ( × 10 3 / μ L) 3.6 1 – 4.8

Eos ( × 10 3 / μ L) 0.6 0.1 – 1.2

Platelets ( × 10 3 / μ L) 410 200 – 500

TP (P) (g/dL) 6.9 6 – 8

Hemopathology: Normal

Biochemical Profi le

Reference Interval

Gluc (mg/dL) 87 65 – 122

BUN (mg/dL) 63 (22.5) 7 – 28 (2.5 – 10.0 mmol/L)

Creat (mg/dL) 1.6 0.9 – 1.7

Ca (mg/dL) 10.3 9.0 – 11.2

Phos (mg/dL) 5.6 2.8 – 6.1

TP (g/dL) 6.8 5.7 – 7.4

Alb (g/dL) 3.9 2.7 – 4.5

Glob (g/dL) 2.9 1.9 – 3.4

T. Bili (mg/dL) 0.3 0 – 0.4

Chol (mg/dL) 230 130 – 370

ALT (IU/L) 80 10 – 120

AST (IU/L) 32 16 – 40

ALP (IU/L) 90 35 – 280

Na (mEq/L) 127 145 – 158

K (mEq/L) 7.5 4.1 – 5.5

CL (mEq/L) 99 106 – 127

TCO 2 (mEq/L) 12 14 – 27

An. gap (mEq/L) 2.3 8 – 25

Urinalysis

Color Yellow Urine Sediment

Transparency Cloudy WBCs/hpf 0 – 1

Sp. Gr. 1.019 RBCs/hpf 2 – 3

Protein Negative Epith cells/hpf 1 – 2 transitional

Gluc Negative Casts/lpf 0

Bilirubin Negative Crystals 0

Blood Negative Bacteria 0

pH 6.0

Endocrine Data Reference Interval

ACTH stimulation:

serum cortisol

( μ g/dL)(pre)

1.1 1 – 4

serum cortisol

( μ g/dL)(post)

1.3 (36) 10 – 20 (276 – 552 nmol/L)

Page 115: VI Clinical Case Presentations - Wiley · Clinical Case Presentations ... Case 17: Dog with vitamin K antagonist (rodenticide) toxicity, 643 ... PT Prothrombin time

Case 64

729

CA

SES

Decreased serum total CO 2 concentration suggests meta-bolic acidosis. Metabolic acidosis is common in hypoadreno-corticism and results from decreased tissue perfusion secondary to hypotension and from decreased renal tubular excretion of H + secondary to mineralocorticoid defi ciency.

Urinalysis Except for the evidence of inadequate urine concentrating ability (see the discussion of azotemia above), the urinalysis is normal.

Endocrine data The inadequate response to ACTH stimulation in com-bination with hyponatremia and hyperkalemia confi rms hypoadrenocorticism. Dogs with hypoadrenocorticism com-monly have decreased basal plasma cortisol concentrations which fail to increase or increase only slightly after ACTH stimulation. If these values do increase after ACTH stimu-lation, they are usually well below normal post - ACTH stimulation values, especially in dogs with primary hypoadrenocorticism.

Summary

Hyponatremia, hyperkalemia, and a Na : K ratio of 17 : 1 strongly suggest hypoadrenocorticism. An inadequate response to ACTH stimulation confi rms this disease. The azotemia with evidence of inadequate urine concentrating ability, while typical of primary renal failure, is more likely due to a combination of prerenal azotemia and decreased renal concentrating ability resulting from the effects of mineralocorticoid defi ciency. It is typical of ill animals to have a stress leukogram (Iymphopenia); absence of stress leukogram in this ill animal is compatible with hypoadrenocorticism.

Interpretive discussion

Hematology The CBC reveals no signifi cant abnormalities.

Biochemical profi le This dog is azotemic. Since urine concentration is not ade-quate (i.e., it is not > 1.030), this may be a renal azotemia, but prerenal azotemia with inadequate renal concentrating ability may occur in hypoadrenocorticism. The hypotension and dehydration that accompany hypoadrenocortism may result in azoternia while hyponatremia and solute diuresis may result in medullary washout which, in turn, limits renal concentrating ability. The result is azotemia with a urine specifi c gravity indicating inadequate renal concentrating ability.

Hyponatremia and hyperkalemia, in combination with the abnormal response to ACTH stimulation, confi rms the diagnosis of hypoadrenocorticism (see discussion of the ACTH stimulation test below). While a Na : K ratio < 23 : 1 is suggestive of hypoadrenocorticism, hyponatremia and hyperkalemia are not specifi c for this disease. Oliguric or anuric renal failure are common causes of hyponatremia and hyperkalemia and should be considered when these abnormalities are observed, but the response to ACTH stimulation should he adequate to distinguish these diseases.

Hypochloremia is common in animals with hypoadreno-corticism. In renal tubules, Cl is reabsorbed with Na in both the proximal tubule and the loop of Henle. After hypona-tremia develops, the concentration of Na in the ultrafi ltrate is decreased, and this, in turn, decreases the amount of Na available for reabsorption in these portions of the nephron. The decreased Na absorption results in decreased Cl absorp-tion and hypochloremia.

Page 116: VI Clinical Case Presentations - Wiley · Clinical Case Presentations ... Case 17: Dog with vitamin K antagonist (rodenticide) toxicity, 643 ... PT Prothrombin time

SECTION VI Clinical Case Presentations

730

CA

SES

Case 65 Signalment: 6 - year - old M canine History: Lethargic, quit eating Physical examination: Depressed, weak pulse, apparent weakness

Hematology Reference Interval

PCV (%) 46.0 37 – 55

Hgb (g/dL) 16.2 12 – 18

PBC ( × 10 6 / μ L) 7.10 5.5 – 8.5

MCV (f1) 65.0 60 – 72

MCHC (g/d1) 35.0 34 – 38

NCC ( × 10 3 / μ L) 20.4 6 – 17

Segs ( × 10 3 / μ L) 11.4 3 – 11.5

Monos ( × 10 3 / μ L) 1.8 0.1 – 1.3

Lymphs ( × 10 3 / μ L) 5.5 1 – 4.8

Eos ( × 10 3 / μ L) 1.6 0.1 – 1.2

Platelets ( × 10 3 / μ L) 574 200 – 500

TP (P) (g/dL) 5.9 6 – 8

Biochemical Profi le

Reference Interval

Gluc (mg/dL) 79 65 – 122

BUN (mg/dL) 95 (33.9) 7 – 28 (2.5 – 10.0 mmol/L)

Creat (mg/dL) 3.8 (334) 0.9 – 1.7 (80 – 150 μ mol/L)

Ca (mg/dL) 14.3 (3.57) 9.0 – 11.2 (2.25 – 2.80 mmol/L)

Phos (mg/dL) 9.9 (3.2) 2.8 – 6.1 (0.9 – 2.0 mmol/L)

TP (g/dL) 5.8 5.4 – 7.4

Alb (g/dL) 3.0 2.7 – 4.5

Glob (g/dL) 2.8 1.9 – 3.4

T. Bili (mg/dL 0.3 0 – 0.4

Chol (mg/dL) 130 130 – 370

ALT (IU/L) 62 10 – 120

AST (IU/L) 108 16 – 40

ALP (IU/L) 38 35 – 280

GGT (IU/L) 3 0 – 6

Na (mEq/L) 124 145 – 158

K (mEq/L) 7.1 4.1 – 5.5

CL (mEq/L) 8.9 106 – 127

TCO 2 (mEq/L) 10.1 14 – 27

An. gap (mEq/L) 32 8 – 25

Amylase (IU/L) 1490 50 – 1250

Lipase (IU/L) 130 30 – 560

Blood Gas Data (arterial)

Reference Interval

pH 7.213 7.33 – 7.45

PO 2 (mmHg) 101.0 67 – 92

PCO 2 (mmHg) 27.6 24 – 39

HCO 3 (meq/L) 10.4 14 – 24

ionized Ca + + (mg/dL) 6.40 4.5 – 5.6

Urinalysis

Color Yellow Urine Sediment

Transparency Cloudy WBCs/hpf 1 – 4

Sp. Gr. 1.018 RBCs/hpf 1 – 2

Protein Negative Epith cells/hpf 1 – 2

Gluc Negative Casts/lpf Negative

Bilirubin Trace Crystals Negative

Blood Negative Bacteria Negative

pH 6.0 Other

UPC 0.93

Endocrine Data Reference Interval

ACTH stimulation:

serum cortisol

( μ g/dL)(pre)

0.04 (1.1) 1 – 4 (28 – 110 nmol/L)

Serum cortisol

( μ g/dL)(post)

0.09 (2.5) < 20 ( < 552 nmol/L)

Page 117: VI Clinical Case Presentations - Wiley · Clinical Case Presentations ... Case 17: Dog with vitamin K antagonist (rodenticide) toxicity, 643 ... PT Prothrombin time

Case 65

731

CA

SES

accumulation of unmeasured anions such as lactic acids or phosphates.

Blood gas data The blood gas data indicate an uncompensated metabolic acidosis (decreased pH and HCO 3 , normal pCO 2 ). The ionized calcium concentration is increased, further supporting a fi nding of hypercalcemia. One should consider the possibili-ties of either primary hypoadrenocorticism or renal disease resulting in a functional defi cit in response to corticosteroids and calcium retention.

Urinalysis The urinary specifi c gravity revels only marginal concentrat-ing ability, which may result from either renal disease, or loss of the medullary concentration gradient due to electro-lyte depletion. This is a common fi nding in hypoadrenocorti-cism that should prompt further diagnostics to rule out primary renal disease. The absence of nonregenerative anemia is evidence counter to chronic renal disease. The dipstick protein was negative, and the UPC is < 1.0, support-ing no signifi cant urinary protein loss.

Endocrine data The pre and post ACTH cortisol concentrations are both low, and there is an inadequate response. This confi rms hypoadrenocorticism.

Summary

Hypoadrenocorticism with typical azotemia secondary to hypovolemia. While there is no biochemical evidence of hemoconcentration, hypovolemia is a consistent event in the pathogenesis of azotemia associated with hypoadrenocorticism.

Interpretive discussion

Hematology There are no erythrocyte abnormalities. There is a lym-phocytosis, which should prompt brief consideration of lymphoma (note the hypercalcemia), or which could be explained by a corticosteroid defi ciency. Whenever an ill animal does not have a stress leukogram, one should con-sider the possibility of hypoadrenocorticism.

Biochemical profi le The BUN, serum creatinine, and phosphorus concentrations are moderately increased. These fi ndings indicate decreased glomerular fi ltration rate. However, one cannot differentiate the nature of the azotemia (prerenal, renal, or postrenal) based on these fi ndings alone. Refer to the discussion of urinalysis results for further interpretation.

The serum total calcium concentration is moderately increased. The most common causes for this would be malignancy - associated hypercalcemia, hypoadrenocorti-cism, or renal failure. One might also consider primary hyperparathyroidism and vitamin D toxicosis.

The serum total protein, albumin, and globulin concentra-tions are normal. The absence of hemoconcentration decreases the probability for prerenal azotemia associated with dehydration.

There are no signifi cant changes in indices of liver disease, with the exception of a mild increase in serum AST activity. This may be due to mild hepatocellular damage or muscle damage, but is small enough that further consideration may not be necessary.

There are signifi cant decrease in the serum concentrations of Na and Cl, as well as a signifi cant increase in serum K concentration. The Na : K ratio is 17.5, which is strongly suggestive of hypoadrenocorticism. The presence of a meta-bolic acidosis (low total CO 2 ) is consistent with that possibil-ity, and the anion gap may be increased owing to

Page 118: VI Clinical Case Presentations - Wiley · Clinical Case Presentations ... Case 17: Dog with vitamin K antagonist (rodenticide) toxicity, 643 ... PT Prothrombin time

SECTION VI Clinical Case Presentations

732

CA

SES

Interpretive discussion

Hematology The PCV, hemoglobin, and total RBC count are within refer-ence limits but plasma protein is increased indicating dehy-dration. If the animal is dehydrated, it is likely that it is mildly anemic. There are no abnormalities in leukogram. Ill animals usually have a stress leukogram.

Biochemical profi le The most signifi cant abnormalities are Na, K, Na : K ratio, and Cl. There are three possible differential diagnoses. The most likely differentials are Addison ’ s disease or chronic renal failure, possibly with a ruptured urinary bladder third. Chronic renal failure is not as probable considering that the creatinine is normal and the animal is capable of concentrat-ing urine. The BUN : Ct ratio is 50, therefore one should suspect dehydration or GI bleeding. Dehydration is deter-mined to be present by increases in both albumin and total serum and plasma protein and can be further confi rmed by physical examination. Creatinine will increase in dehydra-tion due to decreased excretion; however, the BUN increases earlier because there will be decreased excretion and increased reabsorption from tubules. The slow transit time of glomerular fi ltrate through the tubules due to dehydra-tion allows for increased reabsorption of BUN, hence it increases more than creatinine. The urine specifi c gravity would be expected to be > 1.035 if the dog is dehydrated and has normal renal function. The inability to concentrate beyond 1.020 is likely attributable to medullary washout due to the low sodium. Ruptured urinary bladder is unlikely since the dog is urinating and there is no history of trauma. To confi rm Addison ’ s disease one should perform an ACTH stimulation test following the determination of baseline cor-tisol concentrations.

Further support for Addisonian is: hypercalcemia. Hyper-calcemia is seen in one third of dogs with hypoadrenocoriti-cism. However, hypercalcemia can also be seen in a small percentage of dogs with renal failure. It would not be expected in a dog with ruptured bladder. The presence of hypercalcemia in this dog helps prioritize Addison ’ s before renal failure, ruptured urinary bladder and other differen-tials. Hyperphosphatemia is attributed to decreased glomer-ular fi ltration rate due to dehydration in this case.

Hematology Reference Interval

PCV (%) 35.0 34 – 55

Hgb (g/dL) 11.8 11 – 18

RBC (10 6 / μ L) 5.6 5.5 – 8.5

MCV (fL) 63 60 – 72

MCHC (g/dL) 34 34 – 38

NCC ( × 10 3 / μ L) 7.7 6 – 17

Segs ( × 10 3 / μ L) 3.6 3 – 11.5

Bands ( × 10 3 / μ L) 0 0 – 0.3

Monos ( × 10 3 / μ L) 1.2 0.1 – 1.3

Lymphs ( × 10 3 / μ L) 2.4 1 – 4.8

Eos ( × 10 3 / μ L) 0.5 0.1 – 1.2

Platelets ( × 10 3 / μ L) 299 200 – 500

TP (P) (g/dL) 8.5 6 – 8

Case 66 Signalment Six - month - old FS Norwegian elkhound History Poor appetite, small, and has not grown well Physical examination Quiet, unhappy thin dog

Biochemical Profi le Reference Interval

Gluc (mg/dL) 67 65 – 122

BUN (mg/dL) 54 7 – 28

Creat (mg/dL) 0.9 0.9 – 1.7

Ca (mg/dL) 12.7 9.0 – 11.2

Phos (mg/dL) 10.2 2.8 – 6.1

TP (g/dL) 7.8 5.4 – 7.4

Alb (g/dL) 4.9 2.7 – 4.5

Glob (g/dL) 2.9 1.9 – 3.4

T. Bili (mg/dL) 0.2 0 – 0.4

Chol (mg/dL) 211 130 – 370

ALT (IU/L) 92 10 – 120

AST (IU/L) 22 16 – 40

ALP (IU/L) 155 35 – 280

Na (mEq/L) 130 145 – 158

K (mEq/L) 7.7 4.1 – 5.5

CL (mEq/L) 98 106 – 127

Na : K ratio 17 > 25

TCO 2 (mEq/L) 11 14 – 27

An. gap (mEq/L) 28.7 8 – 25

Lipase (IU/L) 175 < 500

Amylase 1895 220 – 800

Urine Analysis

Urine s.g. voided 1.022

Endocrine tests requested Reference Interval

ACTH stimulation

serum cortisol ( μ g/dL)(Pre – basal) 0.4 1 – 4.5

serum cortisol ( μ g/dL)(Post) 0.5 5.5 – 20

Page 119: VI Clinical Case Presentations - Wiley · Clinical Case Presentations ... Case 17: Dog with vitamin K antagonist (rodenticide) toxicity, 643 ... PT Prothrombin time

Case 66

733

CA

SES

Special testing ACTH stimulation confi rmed hypoadrenocorticism. The basal sample is less than 1 µ g/dL which strongly implicates Addison ’ s especially given a Na : K ratio of 17. The post sample of 0.5 is not an increase over basal and is therefore fl atline which confi rms hypoadrenocorticism in this dog.

Summary

Hypoadrenocorticism (Addison ’ s disease), with prerenal azotemia and probable medullary washout.

The most likely lesion is lymphyocytic adenalitis which destroys all three cortical zones of both adrenal glands. Regeneration will not occur therefore recommend treat-ment with glucocorticoids and mineralocorticoids for life. Dog was treated successfully, gained weight, and lived for 7 years.

The serum glucose concentration is at the low end of the reference interval. Hypoglycemia is sometimes seen in patients with hypoadrenocorticism, likely due to a lack of glucocorticoids as well as mineralocorticoids.

The decreased bicarbonate (TCO 2 ) is indicative of a meta-bolic acidosis. Amylase is increased and this is attributed due to decreased glomerular fi ltration rate. Amylase and lipase are excreted through the urine and any cause of decreased GFR may result in one or both enzymes being increased.

Urine specifi c gravity of 1.022 in a dog with dehydration indicates inadequate concentrating ability which could be due to renal disease or medullary washout of sodium. The latter is more likely as azotemia is considered to be prerenal and medullary washout fi ts with Addison ’ s and chronic hyponatremia. The two most important solutes that produce a concentration gradient in the medullary interstitium are UN and sodium. The decreased sodium in the medullary interstitium means glomerular fi ltrate (forming urine) can only be partially concentrated.

Page 120: VI Clinical Case Presentations - Wiley · Clinical Case Presentations ... Case 17: Dog with vitamin K antagonist (rodenticide) toxicity, 643 ... PT Prothrombin time

SECTION VI Clinical Case Presentations

734

CA

SES

Case 67 Signalment: 11 - year - old spayed female beagle History: Polyuria, polydipsia, polyphagia, and bilateral symmetrical alopecia for 5 months Physical examination: “ Pot - bellied, ” comedones in inguinal region, panting

Hematology Reference Interval

PCV (%) 50 37 – 55

NCC ( × 10 3 / μ 1) 22.6 6 – 17

Segs ( × 10 3 / μ 1) 20.0 3 – 11.5

Monos ( × 10 3 / μ 1) 2.3 0.1 – 1.3

Lymphs ( × 10 3 / μ 1) 0 1 – 4.8

Eos ( × 10 3 / μ 1) 0 0.1 – 1.2

NRBC ( × 10 3 / μ 1) 0.3 0

Platelets ( × 10 3 / μ 1) Adequate 200 – 500

TP (P) (g/dL) 7.6 6 – 8

Biochemical Profi le

Reference Interval

Gluc (mg/dL) 140 (7.7) 65 – 122 (3.5 – 6.7

mmol/L)

BUN (mg/dL) 6 (2.1) 7 – 28 (2.5 – 10.0

mmol/L)

Creat (mg/dL) 1.0 0.9 – 1.7

Ca (mg/dL) 10.2 9.0 – 11.2

Phos (mg/dL) 2.7 (0.9) 2.8 – 6.1 (0.9 – 2.0)

TP (g/dL) 7.2 5.4 – 7.4

Alb (g/dL) 4.1 2.7 – 4.5

Glob (g/dL) 3.1 1.9 – 3.4

T. Bili (mg/dL) 0.2 0 – 0.4

Chol (mg/dL) 460 (12.0) 130 – 370 (3.4 – 9.6)

ALT (IU/L) 400 10 – 120

ALP (IU/L) 4500 35 – 280

Na (mEq/L) 159 145 – 158

K (mEq/L) 3.9 4.1 – 5.5

CL (mEq/L) 127 106 – 127

TCO 2 (mEq/L) 20 14 – 27

An. gap (mEq/L) 16 8 – 25

Urinalysis

Specifi c Gravity 1.005

Endocrine Data Reference Interval

ACTH stimulation

serum cortisol

( μ g/dL)(pre)

12 (331) 1 – 4 (28 – 110)

serum cortisol

( μ g/dL)(post)

15.5 < 20

Low dose dexamethasone suppression test

serum cortisol

( μ g/dL)(Pre)

9.0 (248) 1 – 4 (28 – 110)

serum cortisol

( μ g/dL)(3 hour post)

8.0 (221) < 1.5 (41)

serum cortisol

( μ g/dL)(8 hour post)

6.0 (166) < 1.5 (41)

High dose dexamethasone suppression test

serum cortisol

( μ g/dL)(Pre)

10 (276) 1 – 4 (28 – 110)

serum cortisol

( μ g/dL)(post)

8 (221) < 1.5 (41)

endogenous

ACTH (pg/mL)

10 (2.2) 20 – 100

(4.4 – 22.0)

Page 121: VI Clinical Case Presentations - Wiley · Clinical Case Presentations ... Case 17: Dog with vitamin K antagonist (rodenticide) toxicity, 643 ... PT Prothrombin time

Case 67

735

CA

SES

Interpretive discussion

Hematology Mature neutrophilia, monocytosis, and lymphopenia are typically seen with increased endogenous or exogenous cor-ticosteroids. Increased concentration of nucleated RBCs are seen with a variety of conditions; in this case they are likely secondary to hyperadrenocorticism.

Biochemical profi le Mild hyperglycemia is consistent with increased endogenous or exogenous corticosteroids. Glucocorticoids increase glu-coneogenesis and decrease peripheral utilization of glucose by antagonizing the effects of insulin.

The BUN concentration is below the reference interval. While decreased BUN may be associated with liver failure or inadequate protein intake, diuresis will also result in increased urinary loss of urea nitrogen. In this case, diuresis is probably stimulated by glucocorticoids.

Hypercholesterolemia is associated with numerous condi-tions, including hypothyroidism, diabetes mellitus, hyperad-renocorticism, and cholestasis. In this patient, the increase is probably due to hyperadrenocorticism.

Alanine aminotransferase activity is mildly increased, indicating glucocorticoid - induced increase in ALT produc-tion or hepatocellular damage. Hepatocellular damage is an important feature of steroid hepatopathy, which may be occurring in this dog. Alkaline phosphatase activity is mark-edly increased. While cholestasis may result in an increase of this magnitude, bilirubin is not increased, suggesting that the increase is likely due to corticosteroid induction of alka-line phosphatase. Activities of this magnitude are almost always related to steroid effect. Determination of steroid - induced alkaline phosphatase isoenzyme would be helpful.

Mild hypernatremia and hypokalemia are commonly seen in approximately 50% of dogs with hyperadrenocorticism.

Urinalysis Urine specifi c gravity is low, and is likely due to hyperadre-nocorticism. Glucocorticoids are thought to interfere with ADH receptors, resulting in isosthenuria or hyposthenuria, and polyuria and polydipsia.

Endocrine data ACTH stimulation: The baseline cortisol concentration is well above normal and the post - stimulation cortisol concen-

tration is within the reference interval. While most dogs with hyperadrenocorticism have normal basal cortisol con-centrations, this increase is very suggestive of hyperadreno-corticism. While dogs with pituitary dependent hyperplasia (PDH) have hyperplastic adrenals and dogs with functional adrenocortical tumors have the potential to respond to ACTH stimulation by increasing cortisol production and release, not all do so. Cortisol increases above the reference interval following ACTH stimulation in approximately 85% of dogs with pituitary dependent disease, and in approxi-mately 50% of dogs with adrenal tumors. In summary, while ACTH stimulation is a useful screening test for PDH and adrenal tumors, cortisol concentrations do not exceed the high end of the reference interval in many dogs. Thus, this dog may have pituitary dependent disease or an adrenal tumor, based on the ACTH stimulation results.

Low and High Dose Dexamethasone Suppression: Dexa-methasone screening tests are diagnostically useful because in patients with pituitary dependent disease, the abnormal pituitary is somewhat resistant to the negative feedback action of cortisol. Moreover, while dexamethasone may inhibit endogenous ACTH production in dogs with adrenal tumors, endogenous ACTH production is probably already maximally suppressed, and at any rate, these tumors usually autonomously secrete cortisol, independent of ACTH. In normal dogs, endogenous ACTH is suppressed by dexameth-asone, resulting in a rapid decline in plasma cortisol concen-trations which remain suppressed for up to 48 hours. Thus, since this dog ’ s cortisol concentration did not decrease, either pituitary dependent disease resulting in adrenocorti-cal hyperplasia, or adrenal neoplasia, is present.

Endogenous ACTH: Endogenous ACTH is below the refer-ence interval in this dog, indicating that the dog has a func-tional adrenal tumor, rather than pituitary disease.

Summary

Hyperadrenocorticism due to functional adrenal tumor. On abdominal radiographs, a calcifi ed mass cranial to the right kidney was observed. On ultrasound examination, a large right adrenal mass was seen. The left adrenal was not detect-able. A CT scan of the brain was normal.

Page 122: VI Clinical Case Presentations - Wiley · Clinical Case Presentations ... Case 17: Dog with vitamin K antagonist (rodenticide) toxicity, 643 ... PT Prothrombin time

SECTION VI Clinical Case Presentations

736

CA

SES

Case 68 Signalment: 4 - year - old MC golden retriever History: Polyuria, polydipsia for several months, on medication for fl ea allergy dermatitis Physical examination: Exudative, erythematous plaques in inguinal area, “ pot - bellied ” appearance

Hematology Reference Interval

PCV (%) 40 37 – 55

NCC ( × 10 3 / μ L) 25.9 6 – 17

Segs ( × 10 3 / μ L) 23.4 3 – 11.5

Monos ( × 10 3 / μ L) 2.0 0.1 – 1.3

Lymphs ( × 10 3 / μ L) 0.4 1 – 4.8

Eos ( × 10 3 / μ L) 0.1 0.1 – 1.2

Platelets ( × 10 3 / μ L) Adequate 200 – 500

TP (P) (g/dL) 7.5 6 – 8

Biochemical Profi le

Reference Interval

Gluc (mg/dL) 140 (7.7) 65 – 112 (3.5 – 6.7 mmol/L)

BUN (mg/dL) 18 7 – 28

Creat (mg/dL) 1.2 0.9 – 1.7

Ca (mg/dL) 10.5 9.0 – 11.2

Phos (mg/dL) 4.0 2.8 – 6.1

TP (g/dL) 7.0 5.4 – 7.4

Alb (g/dL) 4.0 2.7 – 4.5

Glob (g/dL) 3.0 1.9 – 3.4

T. Bili (mg/dL) 0.2 0 – 0.4

Chol (mg/dL) 350 130 – 370

ALT (IU/L) 110 10 – 120

AST (IU/L) 30 16 – 40

ALP (IU/L) 5500 35 – 280

GGT (IU/L) 260 0 – 6

Na (mEq/L) 148 145 – 158

K (mEq/L) 5.0 4.1 – 5.5

CL (mEq/L) 112 106 – 127

TCO 2 (mEq/L) 16 14 – 27

An. gap (mEq/L) 25 8 – 25

Urinalysis

Color Yellow Urine Sediment

Transparency Cloudy WBCs/hpf 2

Sp. Gr. 1.002 RBCs/hpf 2

Protein Negative Epith cells/hpf 0

Gluc Negative Casts/lpf 0

Bilirubin Negative Crystals 0

Blood Negative Bacteria 4 +

pH 6.5

Endocrine Data Reference Interval

ACTH stimulation

serum cortisol

( μ g/dL)(pre)

1.2 1 – 4

serum cortisol

( μ g/dL)(post)

1.2 (33) > 10.5; < 20 ( > 290; < 550 nmol/L)

Low dose dexamethasone suppression test

serum cortisol

( μ g/dL)(pre)

2.0 1 – 4

serum cortisol

( μ g/dL)(3 - hour post)

2.0 (55) < 1.5 ( < 41 nmol/L)

serum cortisol

( μ g/dL)(8 - hour post)

1.7 (47) < 1.5 ( < 41 nmol/L)

Page 123: VI Clinical Case Presentations - Wiley · Clinical Case Presentations ... Case 17: Dog with vitamin K antagonist (rodenticide) toxicity, 643 ... PT Prothrombin time

Case 68

737

CA

SES

Interpretive discussion

Hematology Mature neutrophilia, monocytosis, and lymphopenia are indicative of a corticosteroid (stress) leukogram.

Biochemical profi le Mild hyperglycemia is consistent with increased endogenous or exogenous corticosteroids.

Alkaline phosphatase activity is markedly increased. While cholestasis may result in an increase of this magni-tude, bilirubin is not increased, suggesting that the increase is likely due to corticosteroid induction of alkaline phospha-tase. Determination of steroid - induced alkaline phosphatase isoenzyme would be helpful.

Gamma glutamyl transferase activity is also markedly increased, and with the lack of increase in ALT and AST activities, as well as bilirubin concentration, corticosteroid induction is likely.

The combination of mild hyperglycemia and increased ALP and GGT activities, with no other evidence of cholesta-sis, should trigger further endocrine testing.

Urinalysis Low urine specifi c gravity (often hyposthenuria) is commonly seen in patients with hyperadrenocorticism.

Glucocorticoids are thought to interfere with ADH receptors, resulting in isosthenuria or hyposthenuria, and polyuria and polydipsia. Bacteriuria without signifi cant pyuria may also occur with hyperadrenocorticism.

Endocrine data ACTH stimulation: Patients with iatrogenic hyperadrenocor-ticism have a “ fl at - line ” response to ACTH stimulation (much like a patient with hypoadrenocorticism) due to feed-back to the pituitary and secondary adrenal atrophy. While some corticosteroid drugs cross - react on the cortisol assay, the post - ACTH response will not be higher than the pre - ACTH response.

Low Dose Dexamethasone Suppression: LDDS is not helpful in diagnosing iatrogenic hyperadrenocorticism. The pituitary is already responding to feedback from iatrogenic corticosteroids, and adrenal glands are atrophied.

Summary

Iatrogenic Cushing disease which resulted from Vetalog injections for fl ea allergy dermatitis. Fleas were eliminated, and the dog was slowly withdrawn from corticosteroids by treating on alternate days with decreasing doses over several months.

Page 124: VI Clinical Case Presentations - Wiley · Clinical Case Presentations ... Case 17: Dog with vitamin K antagonist (rodenticide) toxicity, 643 ... PT Prothrombin time

SECTION VI Clinical Case Presentations

738

CA

SES

Case 69 Signalment: 10 - year - old spayed female Airedale History: “ Leaking ” urine, polydipsia, limping Physical examination: Ruptured anterior cruciate ligament, “ pot - bellied, ” mild truncal alopecia

Hematology Reference Interval

PCV (%) 58 37 – 55

NCC ( × 10 3 / μ L) 24.4 6 – 17

Segs ( × 10 3 / μ L) 21.5 3 – 11.5

Monos ( × 10 3 / μ L) 2.4 0.1 – 1.3

Lymphs ( × 10 3 / μ L) 0 1 – 4.8

NRBC ( × 10 3 / μ L) 0.5 0

Platelets ( × 10 3 / μ L) Adequate 200 – 500

Biochemical Profi le

Reference Interval

Gluc (mg/dL) 130 (7.1) 65 – 122 (3.5 – 6.7 mmol/L)

BUN (mg/dL) 18 7 – 28

Creat (mg/dL) 1.2 0.9 – 1.7

Ca (mg/dL) 10.2 9.0 – 11.2

Phos (mg/dL) 4.9 2.8 – 6.1

TP (g/dL) 5.7 5.7 – 7.4

Alb (g/dL) 2.7 2.7 – 4.5

Glob (g/dL) 3.0 1.9 – 3.4

T. Bili 0.3 0 – 0.4

Chol (mg/dL) 350 130 – 370

ALT (IU/L) 65 10 – 120

AST (IU/L) 60 16 – 40

ALP (IU/L) 4000 35 – 280

Urinalysis

Sp. Gr. 1.008

Bacteria Many

Endocrine Data Reference Interval

ACTH stimulation:

serum cortisol

( μ g/dL)(pre)

8 (221) 1 – 4 (28 – 110 nmol/L)

serum cortisol

( μ g/dL)(post)

20 (552) < 20 ( < 552 nmol/L)

Low dose dexamethasone suppression test

serum cortisol

( μ g/dL)(pre)

6 (166) 1 – 4 (28 – 110 nmol/L)

serum cortisol

( μ g/dL)(3 - hour post)

0.9 < 1.5

serum cortisol

( μ g/dL)(8 - hour post)

1.7 (47) < 1.5 ( < 41 nmol/L)

High dose dexamethasone suppression test

serum cortisol

( μ g/dL)(Pre)

9 (248) 1 – 4 (28 – 110 nmol/L)

serum cortisol

( μ g/dL)(post)

3 (83) < 1.5 ( < 41 nmol/L)

endogenous ACTH

(pg/mL)

350 (77) 20 – 100

(4.4 – 22.0 pmol/L)

Page 125: VI Clinical Case Presentations - Wiley · Clinical Case Presentations ... Case 17: Dog with vitamin K antagonist (rodenticide) toxicity, 643 ... PT Prothrombin time

Case 69

739

CA

SES

Interpretive discussion

Hematology The PCV is mildly increased, with increased nucleated eryth-rocyte concentration. Possibilities for this combination might include hypoxia or other causes of increased erythropoietin concentration. Dogs with hyperadrenocorticism will some-times have increased erythropoiesis. Additionally, corticoste-roids may inhibit removal of NRBC by macrophages in spleen. Mature neutrophilia, monoctosis, and lymphopenia are indicative of a stress leukogram.

Biochemical profi le Mild hyperglycemia is consistent with a stress leukogram, and may be a result of increased endogenous or exogenous glucocorticoids.

Alkaline phosphatase is markedly increased, AST is mildly increased, and cholesterol is borderline high. No other abnormalities are present. Increased alkaline phosphatase activity and mild hypercholesterolemia may be secondary to cholestasis; however, serum bilirubin is not increased. Alka-line phosphatase activity may also increase secondary to steroid induction. This is most likely given the magnitude of increase. The slight increase in serum AST activity may be due to mild steroid hepatopathy or steroid induction.

Urinalysis Urine specifi c gravity is quite low, and while it is not neces-sarily abnormal, it is consistent with decreased urinary con-centrating ability in dogs with hyperadrenocorticism, related to decreased responsiveness to ADH. Bacteriuria without pyuria may be seen in dogs with hyperadrenocorticism.

Physical exam, history, stress leukogram, hyperglycemia, and increased serum alkaline phosphatase activity should trigger screening tests for hyperadrenocorticism.

Endocrine data ACTH stimulation: Baseline cortisol is above normal and poststimulation is “ borderline. ” Stimulation of above 20 is

consistent with hyperadrenocorticism. Eighty - fi ve percent of dogs with pituitary dependent hyperplasia stimulate, as do approximately 50% of dogs with adrenal tumors. Thus, the ACTH stimulation is not diagnostic in this dog, but is suspicious.

Low dose dexamethasone suppression: Baseline cortisol is above normal. The dog suppressed at 3 hours, with escape from suppression at 8 hours. In normal dogs, endogenous ACTH is suppressed by dexamethasone, resulting in a rapid decline in plasma cortisol concentrations which remain sup-pressed for up to 48 hours. Most dogs with adrenal tumors show no suppression at 3 or 8 hours. If a dog suppresses at 3 hours, but does not remain suppressed at 8 hours, it is likely that the dog has PDH, rather than an adrenal tumor. This “ escape ” is thought to be due to rapid clearance of dexamethasone.

High dose dexamethasone suppression: Baseline cortisol is above normal. The dog did not suppress to the range for normal dogs. Dogs with adrenal disease do not suppress, and most dogs with pituitary dependent adrenal hyperplasia (PPH) do suppress. Very high dose steroids will suppress ACTH production, and hence cortisol secretion, even with PPD. However, most dogs with pituitary macroadenomas do not suppress; an endogenous ACTH serum concentration is indicated.

Endogenous ACTH: Dogs with PDH have normal to increased endogenous ACTH, while dogs with adrenal tumors have decreased endogenous ACTH. Thus, this dog has pituitary dependent disease.

Summary

A large pituitary macroadenoma was present in this dog. Note that multiple endocrine tests were required to make this diagnosis.

Page 126: VI Clinical Case Presentations - Wiley · Clinical Case Presentations ... Case 17: Dog with vitamin K antagonist (rodenticide) toxicity, 643 ... PT Prothrombin time

SECTION VI Clinical Case Presentations

740

CA

SES

Hematology Reference Interval

PCV (%) 42.0 37 – 55

Hgb (g/dL) 13.8 12 – 18

RBC (10 6 / μ L) 5.8 5.5 – 8.5

MCV (fL) 72.0 60 – 72

MCHC (g/dL) 33 34 – 38

NCC ( × 10 3 / μ L) 23.4 6 – 17

Segs ( × 10 3 / μ L) 20.1 3 – 11.5

Bands ( × 10 3 / μ L) 0 0 – 0.3

Monos ( × 10 3 / μ L) 2.7 0.1 – 1.3

Lymphs ( × 10 3 / μ L) 0.6 1 – 4.8

Eos ( × 10 3 / μ L) 0 0.1 – 1.2

Platelets ( × 10 3 / μ L) 455 200 – 500

TP (P) (g/dL) 6.5 6 – 8

Hemopathology: few nucleated red blood cells noted

Case 70 Signalment Eight - year - old MC, poodle History Hair loss, PUPD Physical exam Hair loss along abdomen and legs, pendulous abdomen

Urinalysis

Color yellow Urine Sediment

Transparency hazy WBCs/hpf 20 – 30

Specifi c Gravity 1.008 RBCs/hpf 50 – 100

Protein 2 + Epithelial cells/hpf few

Glucose 1 + Casts/lpf neg

Ketones neg

Bilirubin neg Crystals triple phosphate

Blood 3 + Bacteria 1 +

Biochemical Profi le Reference Interval

Gluc (mg/dL) 289 65 – 122

BUN (mg/dL) 22 7 – 28

Creat (mg/dL) 0.8 0.9 – 1.7

Ca (mg/dL) 10.1 9.0 – 11.2

Phos (mg/dL) 5.2 2.8 – 6.1

TP (g/dL) 6.7 5.4 – 7.4

Alb (g/dL) 3.3 2.7 – 4.5

Glob (g/dL) 3.4 1.9 – 3.4

T. Bili (mg/dL) 0.2 0 – 0.4

Chol (mg/dL) 411 130 – 370

ALT (IU/L) 420 10 – 120

AST (IU/L) 122 16 – 40

ALP (IU/L) 6855 35 – 280

Na (mEq/L) 146 145 – 158

K (mEq/L) 4.3 4.1 – 5.5

CL (mEq/L) 115 106 – 127

Na : K ratio 34 > 25

TCO 2 (mEq/L) 20 14 – 27

An. gap (mEq/L) 15.3 8 – 25

Lipase (IU/L) 175 < 500

Amylase 441 220 – 800

Endocrine data Cortisol ( µ g/dL) Reference Interval

Basal cortisol 3.6 1 – 4

ACTH stim 28 8 – 16

Basal cortisol 4.1 1 – 4

LDDS 8 h post 4.4 < 1.4

Basal cortisol 3.8

HDDS 4 h 4.6 < 1.4

HDDS 8 h 2.2 < 1.4

Endogenous ACTH pg/mL 264 10 – 80

Two weeks later

Basal cortisol 0.3 1 – 4

ACTH stim 0.4 8 – 16

Page 127: VI Clinical Case Presentations - Wiley · Clinical Case Presentations ... Case 17: Dog with vitamin K antagonist (rodenticide) toxicity, 643 ... PT Prothrombin time

Case 70

741

CA

SES

disease from an adrenal tumor. The basal concentration of cortisol in the LDDS is just above reference interval but the 8 hour value is well above 1.4 µ g/dL therefore there was failure to suppress and Cushing ’ s disease is ruled in when all the other data fi ts, as in this dog. (False positive rate for LDDS, however, is as high as 50% therefore all the other lab and historical and physical exam results need to fi t with HAC.) Unfortunately, a 4 hour sample was not collected which could have proven helpful to distinguish pituitary and adrenal dependent HAC. If the 4 hour sample exhibited suppression ( < 1.4) with the present value for the 8 hour sample of no suppression it would have indicated a rebound which is consistent with pituitary dependent Cushing ’ s disease, the more common cause of HAC. The HDDS is confusing or at least not very helpful. The basal cortisol is normal, the 4 hour sample clearly failed to suppress and the 8 hour sample is above 1.4, but it is almost 50% less than the basal and the 4 hour samples. The interpretation is the HDDS failed to suppress, indicating AT or PD HAC (suppression would indicate the dog had PD HAC). Although the endocrine testing to this point indicates an adrenal tumor is possible, adrenal tumors only account for 10 – 20% of dogs with HAC. Furthermore there seemed to be some suppression by the HDDS but even using a decrease of cor-tisol by 50% from basal it still did not clearly suppress. Another way to defi ne suppression is if the 8 hour sample is less than 50% of basal even if it is still greater than 1.4 µ g/dL. In this dog the 8 hour sample is 57% of basal, and the 8 hour sample is 47% of the 4 hour sample. Because dis-tinction of AT and PD is needed for treatment and the HDDS was equivocal, abdominal ultrasonography (US) and an endogenous ACTH concentration were performed. Abdomi-nal US did not identify an adrenal tumor and the endog-enous ACTH clearly indicates this dog has PD HAC.

Summary

The e ACTH is markedly increased, and therefore the dog has a pituitary neoplasm secreting ACTH. The dog was given mitotane. The ACTH stim 2 weeks post diagnosis indicates a fl at line response. The dog was clinically normal, and the electrolytes were normal, and therefore this indicated degeneration to necrosis of zona fasiculata by the mitotane. When dosed correctly the cortex will eventually regenerate. Under the stimulation of the ACTH secreting pituitary tumor the adrenal cortex will regenerate and is the reason repeated ACTH stims will be required during maintenance therapy. The results of the ACTH stim are the same pattern seen with hypoadrenocorticism, spontaneous disease, or from mito-tane or steroid therapy.

Interpretive discussion

Hematology A few nucleated red blood cells are present in the absence of anemia. This may indicate a disruption in the endothelial barrier in centers of hematopoiesis, or is possibly a result of immunosuppression and lack of removal of nuclei by mac-rophages. Increased NRBCs can be seen with hemangiosar-coma, some leukemias, lead toxicity, hyperadrenocorticism (HAC) and DIC. The leukogram is characteristic of a stress or steroid response: mature neutorphilia, lymphopenia, eosinopenia and monocytosis.

Biochemical profi le Marked increase in ALP with only mild increases in ALT and AST indicate possible cholestasis and or hyperadrenocorti-cism. An ALP over 5000 IU/L without bilirubinemia and only mild increases in ALT and AST is most consistent with HAC. Further support for this diagnosis is history of alopecia and PUPD coupled with dilute urine, cystitis and nucleated red blood cells. Over 90% of Cushingoid dogs will have mild to marked ALP. If ALP is not increased it is very unlikely that a dog has HAC. Cholesterol is increased which in this dog could be cholestasis, hypothyroidism, or HAC. Increases in ALT and AST are attributed to glycogen (steroid) hepa-topathy induced by hyperadrenocorticism. Hyperglycemia is moderate, which is consistent with hyperadrenocorticism.

The urine analysis has abundant evidence for infectious cystitis; numerous leukocytes and bacteria are present. Col-lection method is not specifi ed, therefore infl ammation could be present anywhere in the urogenital tract. Protein-uria may be due to the infl ammatory response and increased capillary permeability. Absence of casts and no azotemia support cystitis over pyleonephritis. Dilute urine is likely a result of a failure to concentrate due to glucocorticoid interference with ADH, subsequent polyuria, and respon-sive polydipsia. Cystitis is fairly common in dogs with hyperadrenocorticism.

At this point, laboratory evaluation of the endocrine system should be done. Initially, a low dose dexamethasone suppression test (LDDS) should be performed. If LLDS indi-cates hyperadrenocorticism, one should then perform an endogenous ACTH to distinguish pituitary dependent hyper-adrenocorticism (PD HAC) from an adrenal tumor.

Endocrine testing In this patient, the ACTH stimulation was done and although the basal cortisol is normal the post stim sample is > 22 µ g/dL which is excessive and supports hyperadreno-corticism. One must now differentiate pituitary dependent

Page 128: VI Clinical Case Presentations - Wiley · Clinical Case Presentations ... Case 17: Dog with vitamin K antagonist (rodenticide) toxicity, 643 ... PT Prothrombin time

SECTION VI Clinical Case Presentations

742

CA

SES

Case 71 Signalment: 6 - year - old male dog History: Change in temperament from docile to irritable. Severe constipation for several days. Physical examination: No abnormalities detected

Hematology Reference Interval

PCV (%) 44 37 – 55

Hgb (g/dL) 14.5 12 – 18

RBC ( × 10 6 / μ L) 6.7 5.5 – 8.5

MCV (fL) 66 60 – 72

MCHC (g/dL) 33 34 – 38

NCC ( × 10 3 / μ L) 15.6 6 – 17

Segs ( × 10 3 / μ L) 12.7 3 – 11.5

Monos ( × 10 3 / μ L) 0.2 0.1 – 1.3

Lymphs ( × 10 3 / μ L) 2.4 1 – 4.8

Eos ( × 10 3 / μ L) 0.3 0.1 – 1.2

Platelets ( × 10 3 / μ L) 440 200 – 500

TP (P) (g/dL) 6.8 6 – 8

Hemopathology: Normal

Biochemical Profi le

Reference Interval

Gluc (mg/dL) 80 65 – 122

BUN (mg/dL) 28 7 – 28

Creat (mg/dL) 1.5 0.9 – 1.7

Ca (mg/dL) 14.3 (3.57) 9.0 – 11.2 (2.25 – 2.80 mmol/L)

Phos (mg/dL) 1.7 (0.5) 2.8 – 6.1 (0.9 – 2.0 mmol/L)

TP (g/dL) 6.1 5.4 – 7.4

Alb (g/dL) 3.4 2.7 – 4.5

Glob (g/dL) 2.7 1.9 – 3.4

T. Bili (mg/dL) 0.4 0 – 0.4

Chol (mg/dL) 235 130 – 370

ALT (IU/L) 100 10 – 120

AST (IU/L) 33 16 – 40

ALP (IU/L) 285 35 – 280

Na (mEq/L) 145 145 – 158

K (mEq/L) 5.3 4.1 – 5.5

CL (mEq/L) 115 106 – 127

TCO 2 (mEq/L) 21 14 – 27

An. gap (mEq/L) 14 8 – 25

Urinalysis (catheterized)

Color Yellow Urine Sediment

Transparency Clear WBCs/hpf 0 – 2

Sp. Gr. 1.011 RBCs/hpf 0

Protein Negative Epith cells/hpf 0

Gluc Negative Casts/lpf 0

Bilirubin Negative Crystals 0

Blood Negative Bacteria 0

pH 6.5

Endocrine Data Reference Interval

Intact parathormone 22 2 – 13 (pmol/L)

PTHrp Undetectable < 0.2 (pmol/L)

Page 129: VI Clinical Case Presentations - Wiley · Clinical Case Presentations ... Case 17: Dog with vitamin K antagonist (rodenticide) toxicity, 643 ... PT Prothrombin time

Case 71

743

CA

SES

Interpretive discussion

Hematology In light of normal results for other erythrocyte measure-ments, the decreased MCHC is marginal and not important.

The mild mature neutrophilia, in the absence of lympho-penia, suggests normal variability or very mild infl ammation. This dog ’ s irritability may have predisposed it to epinephrine release when the venipuncture was performed, although excitement leukograms are quite rare in dogs.

Biochemical profi le Hypercalcemia and hypophosphatemia can occur with primary hyperparathyroidism and pseudohyperparathyroid-ism (hypercalcemia of malignancy). In this case, the increased intact parathormone (iPTH) and normal parathormone - related protein (PTHrp) concentrations are most suggestive of primary hyperparathyroidism (see discussion of hormone assays below). Other causes of hypercalcemia include vitamin D toxicosis, excessive bone resorption, and renal failure (5 – 10% of these cases in dogs), but serum phospho-rus concentration is typically normal to increased in these cases.

The slightly increased serum alkaline phosphatase activity is not signifi cant. There is no evidence suggesting either cholestasis or increased corticosteroid levels. Since this dog has an abnormality of calcium and phosphorus metabolism, it is possible that altered bone metabolism is occurring. Although the net effect in this animal is probably bone demineralization, increased osteoblastic activity, as part of effort to regenerate bone, may have resulted in this slight increase in activity.

Urinalysis Low urine specifi c gravity may refl ect this dog ’ s hydration status and, therefore, may be normal in this patient. Hyper-

calcemia can, however, interfere with renal concentrating ability and can result in decreased urine specifi c gravity with subsequent polyuria and polydipsia. Nephrocalcinosis, other toxic effects of calcium on renal tubules, and interference with the action of antidiuretic hormone are possible mecha-nisms for decreased concentrating ability in hypercalcemic animals. The absence of polyuria and polydipsia in this dog suggests that calcium interference with renal concentrating ability is not a major factor.

Endocrine data Increased intact parathormone (iPTH) concentration and undetectable PTH - related protein (PTHrp) concentration indicate that this dog has primary hyperparathyroidism, rather than hypercalcemia of malignancy. The iPTH concen-trations are increased due to overproduction of PTH by hyperplastic or neoplastic parathyroid glands. Parathormone - related protein is synthesized by malignant cells of neoplasms such as lymphoma and apocrine gland adenocar-cinoma of the anal sac, but not by the parathyroid glands, and concentrations of PTHrp are, therefore, not increased in animals with primary hyperparathyroidism.

Summary

The combination of hypercalcemia, hypophosphatemia, increased iPTH concentration, and undetectable PTHrp concentration indicate primary hyperparathyroidism in this case. A mass in the neck region compatible in location with the parathyroid gland was found during a more thor-ough physical examination. Surgical removal and histo-pathologic examination revealed a parathyroid adenoma. This dog ’ s clinical signs and serum calcium and phospho-rus concentrations returned to normal after surgery. Irrita-bility is unusual in hypercalcemic dogs; dullness is more common.

Page 130: VI Clinical Case Presentations - Wiley · Clinical Case Presentations ... Case 17: Dog with vitamin K antagonist (rodenticide) toxicity, 643 ... PT Prothrombin time

SECTION VI Clinical Case Presentations

744

CA

SES

Interpretive discussion

Hematology Unremarkable; absence of a stress leukogram may prove informative.

Biochemical profi le Mild hypercalcemia is the only abnormality. The increased total serum calcium explains the dilute hyposthenuria, as hypercalcemia affects the function of ADH on tubules. Serum phosphorus is normal which is somewhat helpful to shorten the list of differentials for hypercalcemia. Recom-mend rechecking calcium, total and ionized, and if both are increased pursue possible causes of hypercalcemia. The calcium is likely to be increased again given the urine spe-cifi c gravity. The two most likely differentials are hypercal-cemia of malignancy and primary hyperparathyroidism because the serum phosphorus is normal and there are no other signifi cant biochemical abnormalities. Unlikely differ-entials are hypoadrenocorticism, renal failure, vitamin D toxicity, and granulomatous diseases, all of which usually have increased serum phosphorus and produce other bio-chemical disturbances. The only two diseases that produce hypercalcemia and hypophosphatemia in dogs are hypercal-cemia of malignancy and primary hyperparathyroidism.

Summary and follow - up

Both total serum calcium and ionized calcium were increased on a recheck. On physical examination, no evidence of lym-phoma or perirectal apocrine gland adenocarcinoma was found. Serum was sent for PTH and PTH - rp.

Hematology Reference Interval

PCV (%) 53 39 – 58

Hgb (g/dL) 19.7 13.8 – 20.3

RBC (10 6 / μ L) 7.67 5.7 – 8.0

MCV (fL) 75 61 – 75

MCHC (g/dL) 34.3 30.8 – 35.4

NCC ( × 103/ μ L) 6.91 4.4 – 11.6

Segs ( × 103/ μ L) 4.9 2.8 – 9.1

Bands ( × 103/ μ L) 0 0 – 0.3

Monos ( × 103/ μ L) 0.9 0.07 – 1.0

Lymphs ( × 103/ μ L) 2.4 0.6 – 3.3

Eos ( × 103/ μ L) 0.2 0 – 1.2

Platelets ( × 103/ μ L) 366 200 – 500

TP (P) (g/dL) 7.4 6.1 – 7.5

Case 72 Signalment Eleven - year old FS Australian cattle dog History Poor appetite Physical examination Quiet, adequate body condition

Biochemical Profi le

Reference Interval

Gluc (mg/dL) 91 70 – 131

BUN (mg/dL) 14 6 – 26

Creat (mg/dL) 0.7 0.7 – 1.5

Ca (mg/dL) 12.3 9.3 – 11.5

Phos (mg/dL) 3.3 2.5 – 5.6

Magnesium (mg/dL) 2.0 1.8 – 2.5

TP (g/dL) 6.7 5.2 – 7.4

Alb (g/dL) 3.9 3 – 3.9

Glob (g/dL) 2.8 1.7 – 3.8

T. Bili (mg/dL) 0.1 0 – 0.3

Chol (mg/dL) 274 124 – 344

ALS (IU/L) 72 12 – 54

ALP (IU/L) 62 16 – 140

GGT (IU/L) 5 0 – 6

CK (IU/L) 176 43 – 234

Na (mEq/L) 145 140 – 156

K (mEq/L) 4.4 4 – 5.3

CL (mEq/L) 111 108 – 122

Na : K ratio 32.6 > 25

TCO 2 (mEq/L) 22 18 – 26

An. gap (mEq/L) 16.1 11.2 – 19

Lipase (IU/L) 210 12 – 147

Amylase 600 236 – 1337

Urine Analysis

Urine s.g. voided 1.007

Endocrine Data Reference Interval

PTH (pmol/L) 35.5 3 – 17

PTHrp (pmol/L) 0 0 – 0.9

iCa 1.65 1.25 – 1.45

Additional Endocrine Data Reference Interval

PTH Sample 1, pre, Turbo intact PTh (pg/mL) 98 11.2 – 72.8

PTH Sample 2, post, Turbo intact PTh (pg/mL) 9.0

These results confi rm hypercalcemia and rule in primary hyperparathyroidism.

The neck region was explored and a small mass was found in the region of one thyroid lobe. During surgery a STAT methodology to measure PTH pre and post removal of any mass was employed and the results were:

Page 131: VI Clinical Case Presentations - Wiley · Clinical Case Presentations ... Case 17: Dog with vitamin K antagonist (rodenticide) toxicity, 643 ... PT Prothrombin time

Case 72

745

CA

SES

The decrease in PTH in the second sample, after the para-thyroid mass was removed is dramatic (less than 50% of previous sample) indicating that the offending lesion was removed.

Comment

The serum concentration of PTH was increased in this dog when it was hypercalcemic which makes the diagnosis of primary hyperparathyroidism (HPTH) quite simple. However, increased PTH is only present in about 25% of dogs and the remaining dogs (75%) with primary HPTH will have a concentration of PTH within reference interval. Moreover, 45% of dogs with primary hyperparathyroidsm will have serum PTH concentrations in the low to middle range. Increased concentrations of PTH are the exception

in dogs, but if PTH is detectable in an animal that is hyper-calcemic and not azotemic then this combination is inap-propriately abnormal because PTH should be decreased or undetectable in response to nonparathyroid induced hyper-calcemia. If PTH is within the reference interval, it is inap-propriately high in the face of hypercalcemia, and therefore diagnostic for primary HPTH. It indicates the parathyroid gland is secreting PTH at a time when secretion should be suppressed. It is critical to measure PTHrp concurrently as many dogs with hypercalcemia of malignancy will have measureable PTH.

Ultrasonography of the neck region is just as accurate at identifying a parathyroid mass as is measuring serum PTH and if positive will localize the side of the neck to look for the adenoma during surgery. More than one mass is possi-ble, especially in Keeshonds.

Page 132: VI Clinical Case Presentations - Wiley · Clinical Case Presentations ... Case 17: Dog with vitamin K antagonist (rodenticide) toxicity, 643 ... PT Prothrombin time

SECTION VI Clinical Case Presentations

746

CA

SES

Hematology Reference Interval

Packed cell volume (%) 32 39 – 58

Hemoglobin (g/dL) 10.8 13.8 – 20.3

RBC (10 6 / μ L) 4.27 5.7 – 8.01

MCV (fL) 74 61 – 75

MCHC (g/dL) 34 30.8 – 35.4

Total nucleated cell count ( × 10 3 / μ L) 10.2 4.4 – 11.6

Segmented neutrophils ( × 10 3 / μ L) 7.9 2.84 – 9.11

Band neutrophils ( × 10 3 / μ L) 0 0 – 0.3

Monocytes ( × 10 3 / μ L) 1.9 0.075 – 1.0

Lymphocytes ( × 10 3 / μ L) 0.4 0.59 – 3.3

Eosinophils ( × 10 3 / μ L) 0 0.03 – 1.2

Platelets ( × 10 3 / μ L) 386 190 – 468

Plasma protein (g/dL) 7.1 6.1 – 7.5

Case 73 Signalment Eleven - year - old FS mixed breed dog History Weight loss, poor appetite and lethargy Physical examination Thin body condition, depressed attitude

Interpretive discussion

Hematology There is a mild anemia. Although a reticulocyte count is not present the indices are normocytic normochromic which indicate it may be nonregenerative. The azotemia could explain this anemia if the azotemia is due to chronic renal disease. Anemia of infl ammatory disease is another possible cause of the anemia in this dog. There is a stress leukogram as evidenced by the lymphopenia, eosinopenia, and monocytosis.

Biochemical profi le The dog has renal failure based on mild azotemia combined with isosthenuria. However, any time the serum calcium is increased the kidneys may not be able to concentrate urine adequately (interference with ADH) which in this case clouds the interpretation that primary renal disease is present. If this dog is dehydrated the azotemia may all or partially be due to prerenal and the dilute urine in the face of dehydration is caused by hypercalcemia. Hypercalcemia is severe, hyperphosphatemia is mild to moderate. The Ca × P product is 126, which indicates mineralization of soft tissues is occurring. Mineralization may have caused the renal failure or at least enhanced it. The diagnostic dilemma is to determine which came fi rst, the renal failure or the hypercalcemia. It is often diffi cult to distinguish and there may be two diseases occurring, renal failure and a disease other than renal disease that caused hypercalcemia (e.g., a malignancy, vitamin D toxicity, etc.). In this dog it seems more likely the hypercalcemia came fi rst or that there is a second disease causing hypercalcemia. This is based on mild hyperphosphatemia and marked hypercalcemia. Rules to help make this distinction are: the greater the serum P the more likely the primary disease is renal and the lower the serum P the more likely there is hypercalcemia of malig-nancy (HCM). The higher the serum calcium the more likely there is a malignancy and the lower the calcium the more likely renal disease is causing hypercalcemia. The greater the

Biochemical Profi le Reference Interval

Gluc (mg/dL) 98 70 – 131

BUN (mg/dL) 74 6 – 26

Creat (mg/dL) 3.7 0.7 – 1.5

Ca (mg/dL) 17.3 9.3 – 11.5

Phos (mg/dL) 7.3 2.5 – 5.6

Magnesium (mg/dL) 2.0 1.8 – 2.5

TP (g/dL) 6.9 5.2 – 7.4

Alb (g/dL) 3.9 3 – 3.9

Glob (g/dL) 3.0 1.7 – 3.8

T. Bili (mg/dL) 0.1 0 – 0.3

Chol (mg/dL) 254 124 – 344

ALS (IU/L) 372 12 – 54

ASt (IU/L) 388 42 – 175

ALP (IU/L) 662 16 – 140

GGT (IU/L) 15 0 – 6

CK (IU/L) 111 43 – 234

Na (mEq/L) 141 140 – 156

K (mEq/L) 4.9 4 – 5.3

CL (mEq/L) 110 108 – 122

Na : K ratio 28.7 > 25

TCO 2 (mEq/L) 12 18 – 26

An. gap (mEq/L) 23.9 11.2 – 19

Lipase (IU/L) 510 12 – 147

Amylase 1724 236 – 1337

Urinalysis cystocentesis

Color yellow Urine Sediment

Transparency clear WBCs/hpf 0 – 3

Specifi c Gravity 1.010 RBCs/hpf 10 – 30

Protein 1 + Epithelial cells/hpf none

Glucose Casts/lpf neg

Ketones neg Crystals none

Bilirubin neg

Blood 2 +

Page 133: VI Clinical Case Presentations - Wiley · Clinical Case Presentations ... Case 17: Dog with vitamin K antagonist (rodenticide) toxicity, 643 ... PT Prothrombin time

Case 73

747

CA

SES

overweight with an acute onset of problems, none of which this dog has. The dog has a metabolic acidosis probably caused by dehydration and or renal disease.

A reasonable plan is to search for lymphoma (palpate lymph nodes, search for an anterior thoracic mass, abdomi-nal organ evaluation, etc.) and do a rectal and perirectal exam to evaluate if a carcinoma of the anal sacs is present. If a malignant tumor is found there is no need to perform PTH and PTH rp assays.

Summary

A mass was found in the pelvic vault, and aspirational cytol-ogy indicated it was a tumor of the anal sacs. These are invariably malignant, but repeated excisions and or chemo-therapy can extend a dog ’ s life for months or years. These owners declined treatments. Intravenous and subcutaneous fl uid therapy decreased the azotemia, but it did not return to normal. An autopsy was not performed so it is not known if or what type of renal disease was present and whether mineralization played a role. The anemia was due to anemia of chronic disease (cancer) and or concurrent renal disease. The most common substance secreted by tumors is PTHrp, which stimulates phosphaturia and results in hypophospha-temia, absolute or as in this dog relative for the degree of azotemia. Primary hyperparathyroidism could cause the hypercalcemia and phosphaturia, but rarely is there concur-rent azotemia with primary hyperparathyroidism.

azotemia the more likely it is primary renal and the lower the azotemia the more likely the renal problems are caused by the mineralization or it is pre renal. The easiest way to distinguish is to either fi nd the malignancy or iden-tify the type of renal failure (chronic, acute, glomerular, pyelonephritis etc.). In this dog it seems clear the primary disease is one that is causing hypercalcemia because the azotemia is mild, the hypercalcemia is severe and hyper-phosphatemia is mild. This distinction can be more diffi cult in other cases. A reasonable plan is to search for cancer and give the dog fl uids to see if the azotemia can be reversed. Measuring ionized calcium may also be diagnostically helpful. If the ionized calcium is within the reference inter-val then primary renal failure is more likely, but if ionized calcium is increased it could still be either primary renal or a malignancy.

The urine is not concentrated, which could be primary renal or secondary to hypercalcemia. There is some blood likely due to cystocentesis, which may also be responsible for the 1 + protein. The rest of the analysis is unremarkable.

The hepatic leakage enzymes are increased (ALT and AST) and the enzymes associated with cholestasis are increased (ALP, GGT). There are many possible causes of cholestasis in this dog, one of which is an infi ltrative disease in the liver such as lymphoma, another possiblity is pancreatitis. The increases in lipase and amylase are mild and seem more likely due to decreased GFR (azotemia) and therefore decreased excretion than pancreatitis. If the dog had pan-creatitis it may have hypocalcemia, tender abdomen and be

Page 134: VI Clinical Case Presentations - Wiley · Clinical Case Presentations ... Case 17: Dog with vitamin K antagonist (rodenticide) toxicity, 643 ... PT Prothrombin time

SECTION VI Clinical Case Presentations

748

CA

SES

Case 74 Signalment: 9 - year - old intact male dog History: One seizure. Occasional tremors observed. Physical examination: Physical abnormalities, but had seizure during examination

Hematology Reference Interval

PCV (%) 44 37 – 55

Hgb (g/dL) 15.2 12 – 18

RBC ( × 10 6 / μ L) 7.1 5.5 – 8.5

MCV (fL) 62 60 – 72

MCHC (g/dL) 35 34 – 38

NCC ( × 10 3 / μ L) 20.2 6 – 17

Segs ( × 10 3 / μ L) 17.2 3 – 11.5

Monos ( × 10 3 / μ L) 2.4 0.1 – 1.3

Lymphs ( × 10 3 / μ L) 0.6 1 – 4.8

Platelets ( × 10 3 / μ L) 470 200 – 500

TP (P) (g/dL) 7.2 6 – 8

Hemopathology: Normal

Biochemical Profi le

Reference Interval

Gluc (mg/dL) 138 (7.6) 65 – 122 (3.5 – 6.7 mmol/L)

BUN (mg/dL) 14 7 – 28

Creat (mg/dL) 0.5 0.9 – 1.7

Ca (mg/dL) 4.0 (1.0) 9.0 – 11.2 (2.25 – 2.80 mmol/L)

Phos (mg/dL) 7.0 (2.3) 2.8 – 6.1 (0.9 – 2.9 mmol/L)

TP (g/dL) 7.0 5.4 – 7.4

Alb (g/dL) 3.6 2.7 – 4.5

Glob (g/dL) 3.4 1.9 – 3.4

T. Bili (mg/dL) 0.4 0 – 0.4

Chol (mg/dL) 161 130 – 370

ALT (IU/L) 38 10 – 120

AST (IU/L) 18 16 – 40

ALP (IU/L) 176 35 – 280

Na (mEq/L) 145 145 – 158

K (mEq/L) 4.4 4.1 – 5.5

CL (mEq/L) 103 106 – 127

TCO 2 (mEq/L) 22 14 – 27

An. gap (mEq/L) 24 8 – 25

Urinalysis

Color Yellow Urine Sediment

Transparency Clear WBCs/hpf 0

Sp. Gr. 1.032 RBCs/hpf 0

Protein Negative Epith cells/hpf 0

Gluc Negative Casts/lpf 0

Bilirubin Trace Crystals 0

Blood Negative Bacteria 0

pH 6.0

Endocrine Data Reference Interval

iPTH (pmol/L) 2 2 – 13

Page 135: VI Clinical Case Presentations - Wiley · Clinical Case Presentations ... Case 17: Dog with vitamin K antagonist (rodenticide) toxicity, 643 ... PT Prothrombin time

Case 74

749

CA

SES

may also result in hypocalcemia, but hypophosphatemia rather than hyperphosphatemia is typical of such a defi -ciency. Hypoparathyroidism can be confi rmed by measuring the serum intact parathormone concentration (see below).

Slight hypochloremia, in the absence of abnormalities in Na, K, or total CO 2 , is probably insignifi cant.

Urinalysis The urinalysis is normal.

Endocrine data The serum intact parathyroid hormone (iPTH) concentration is at the low end of the reference interval. The normal response of the parathyroid glands to hypocalcemia is pro-duction of PTH. Low normal iPTH concentration in a hypo-calcemic animal strongly suggests inability of the parathyroid glands to respond to hypocalcemia and, therefore, hypo-parathyroidism. Other possible causes of hypocalcemia (dis-cussed above) should result in high normal to increased iPTH concentrations.

Summary

The combination of hypocalcemia with low normal iPTH concentration indicate hypoparathyroidism. Other diseases can result in hypocalcemia and hyperphosphatemia, but iPTH concentration in these diseases is typically high normal to increased.

Interpretive discussion

Hematology Mature neutrophilia, lymphopenia, and monocytosis are typical of a stress leukogram.

Biochemical profi le The serum glucose concentration is in the range typical for glucocorticoid - induced hyperglycemia. Stress is the most likely cause in this case, particularly in light of the leukogram.

Decreased serum creatinine concentration is meaningless in most cases. This abnormality can result from diuresis, but, if this is the cause, the BUN concentration is usually also decreased. The absence of a history of polyuria and the normal BUN concentration make diuresis unlikely in this case.

Hypocalcemia and hyperphosphatemia can occur in renal failure, pancreatitis with prerenal azotemia, eating a diet containing excessive phosphorus, or hypoparathyroidism. Hypoparathyroidism is most likely in this case. Normal BUN concentration and decreased serum creatinine concentration indicate that renal function is normal. Clinical signs are not typical of pancreatitis, and there is no evidence of a prerenal azotemia. This dog may be receiving a diet with excessive phosphorus, but this is very unlikely if the dog is receiving a commercial diet. Hypoalbuminenmia is another cause of hypocalcemia, but the absence of hypoalbuminemia indi-cates that this is not a consideration. Vitamin D defi ciency